kỷ yếu hội thảo các trường chuyên dh db bắc bộ lần iii

132
HI CÁC TRƯỜNG THPT CHUYÊN KHU VC DUYÊN HI VÀ ðỒNG BNG BC BKYU HI THO KHOA HC, LN THIII MÔN TOÁN HC (TÀI LIU LƯU HÀNH NI B) HÀ NAM, THÁNG 11 NĂM 2010

Upload: chu-thi-thu-hien

Post on 29-Jul-2015

58 views

Category:

Documents


8 download

TRANSCRIPT

Page 1: Kỷ yếu hội thảo các trường chuyên dh db bắc bộ lần III

HỘI CÁC TRƯỜNG THPT CHUYÊN

KHU VỰC DUYÊN HẢI VÀ ðỒNG BẰNG BẮC BỘ

KỶ YẾU

HỘI THẢO KHOA HỌC, LẦN THỨ III

MÔN TOÁN HỌC (TÀI LIỆU LƯU HÀNH NỘI BỘ)

HÀ NAM, THÁNG 11 NĂM 2010

Page 2: Kỷ yếu hội thảo các trường chuyên dh db bắc bộ lần III

=========================================================== 4

HỘI CÁC TRƯỜNG THTP CHUYÊN KHU VỰC DUYÊN HẢI VÀ ðỒNG BẰNG BẮC BỘ

Hội thảo khoa học môn Toán học lần thứ III - 2010

MỤC LỤC

STT NỘI DUNG TRANG

1 LỜI NÓI ðẦU 5

2

MỘT SỐ DẠNG PHƯƠNG TRÌNH VÔ TỈ CHO HỌC SINH GIỎI

Nguyễn Anh Tuấn (THPT chuyên Bắc Giang) 6

3 LÀM NGƯỢC BẤT ðẲNG THỨC

Nguyễn ðức Vang (THPT chuyên Bắc Ninh) 27

4

CHỨNG MINH BẤT ðẲNG THỨC BẰNG CÁCH SỬ DỤNG BẤT

ðẲNG THỨC SẮP XẾP LẠI VÀ BẤT ðẲNG THỨC CHEBYSHEV

ðào Quốc Huy, Tổ Toán – Tin, Trường THPT Chuyên Biên Hòa – Hà Nam

31

5 TÍNH TUẦN HOÀN TRONG DÃY SỐ NGUYÊN

Ngô Thị Hải, trường THPT chuyên Nguyễn Trãi, Hải Dương 43

6 ðỊNH LÝ PASCAL VÀ ỨNG DỤNG

Lê ðức Thịnh, THPT Chuyên Trần Phú – Hải Phòng 47

7 HÀM SỐ HỌC VÀ MỘT SỐ BÀI TOÁN VỀ HÀM SỐ HỌC

Trường THPT Chuyên Hưng Yên 56

8 MỘT SỐ BÀI TOÁN SỐ HỌC TRONG CÁC KÌ THI OLYMPIC TOÁN

Trần Xuân ðáng (THPT Chuyên Lê Hồng Phong – Nam ðịnh) 67

9 ðỊNH LÍ LAGRANGE VÀ ỨNG DỤNG

ðặng ðình Sơn, Chuyên Lương Văn Tụy – Ninh Bình 73

10 TỈ SỐ KÉP VÀ PHÉP CHIẾU XUYÊN TÂM

Trường THPT chuyên Thái Bình – Thái Bình 93

11 MỘT SỐ DẠNG TOÁN VỀ DÃY SỐ VÀ GIỚI HẠN

Trần Ngọc Thắng - THPT Chuyên Vĩnh Phúc 105

12

SỬ DỤNG CÔNG CỤ SỐ PHỨC ðỂ GIẢI CÁC BÀI TOÁN HÌNH HỌC

PHẲNG

Trường THPT chuyên Hạ Long 123

13 BẤT BIẾN TRONG CÁC BÀI TOÁN LÝ THUYẾT TRÒ CHƠI

Phạm Minh Phương, trường THPT chuyên ðại học Sư phạm Hà Nội 130

Page 3: Kỷ yếu hội thảo các trường chuyên dh db bắc bộ lần III

=========================================================== 5

HỘI CÁC TRƯỜNG THTP CHUYÊN KHU VỰC DUYÊN HẢI VÀ ðỒNG BẰNG BẮC BỘ

Hội thảo khoa học môn Toán học lần thứ III - 2010

DI TRUYỀN HỌC

LỜI NÓI ðẦU

Hội các trường chuyên vùng Duyên Hải Bắc Bộ ñến nay ñã có 12 trường

tham gia. Trong ñó có nhiều trường có truyền thống lâu năm, có thành tích

cao trong các kỳ thi học sinh giỏi Quốc gia và Quốc tế môn Toán.

Năm nay, lần thứ 3 hội thảo khoa học. Với cương vị là ñơn vị ñằng cai,

chúng tôi ñã nhận ñược 12 bài viết về các chuyên ñề chuyên sâu cho học

sinh giỏi Toán. ðó là các chuyên ñề tâm huyết của các thày cô dạy chuyên

Toán của các trường chuyên trong hội.

Xin trân trọng giới thiệu các bài viết của các thày cô trong kỷ yếu môn

Toán của hội trong dịp hội thảo khoa học lần thứ 3. Hy vọng rằng cuốn kỷ

yếu này sẽ một tài liệu tham khảo cho các thày cô!

TỔ TOÁN - TIN

TRƯỜNG THPT CHUYÊN BIÊN HOÀ - HÀ NAM

Page 4: Kỷ yếu hội thảo các trường chuyên dh db bắc bộ lần III

=========================================================== 6

HỘI CÁC TRƯỜNG THTP CHUYÊN KHU VỰC DUYÊN HẢI VÀ ðỒNG BẰNG BẮC BỘ

Hội thảo khoa học môn Toán học lần thứ III - 2010

MỘT SỐ DẠNG PHƯƠNG TRÌNH VÔ TỈ CHO HỌC SINH GIỎI Nguyễn Anh Tuấn (THPT chuyên Bắc Giang)

Lời mở ñầu Toán học có một vẻ ñẹp lôi cuốn và quyến rũ, ai ñã ñam mê thì mãi mãi ñam mê…

Trong vẻ ñẹp ñầy huyền bí ñó thì các bài toán liên quan ñến Phương trình vô tỷ (chứa căn thức) - có nét ñẹp thật sự xao xuyến và quyến rũ.

Có lẽ vì lý do ñó mà trong các kì thi HSG các nước, thi HSG Quốc gia (VMO) của

chúng ta, bài toán liên quan ñến Phương trình vô tỷ thường có mặt ñể thách thức các nhà Toán học tương lai với dung nhan muôn hình, muôn vẻ. Rồi thì còn trong các kì thi HSG cấp tỉnh, thi HSG cấp thành phố, thi ðại học, thi …

Thật là ñiều thú vị ! Chuyên ñề: “ Một số dạng phương trình vô tỷ cho học sinh giỏi ” tôi viết với mong

muốn phần nào giúp các Thầy cô giáo dạy Toán, các em học sinh phổ thông trong các ñội tuyển thi học sinh giỏi Toán có thể tìm thấy nhiều ñiều bổ ích và nhiều ñiều thú vị ñối với dạng toán này. Trong Chuyên ñề có cả những bài với cấp ñộ giải trí cho học sinh giỏi (rèn luyện phản xạ nhanh).

ðối với việc giải phương trình vô tỷ thì hầu hết các phương pháp giải, các phương

pháp biến ñổi hay ñều có trong cuốn Chuyên ñề này. Cách phân tích ñể nhận dạng một phương trình và chọn lựa phương pháp giải thích hợp là khó và ña dạng. ðể có khả năng này chúng ta phải giải quyết nhiều phương trình và tự rút ra những nhận xét, kinh nghiệm và hay hơn nữa là một vài thuật giải toán, cũng như lưu ý rằng một bài toán có thể có nhiều cách giải khác nhau.

Tôi viết Chuyên ñề này với một tinh thần trách nhiệm cao. Tôi hy vọng rằng Chuyên

ñề sẽ ñể lại trong lòng Thầy cô và các em học sinh một ấn tượng tốt ñẹp. Với mỗi ví dụ trong từng phương pháp giải, người ñọc có thể tự sáng tác cho mình

những bài toán với những con số mà mình yêu thích. Tuy nhiên Chuyên ñề chắc chắn sẽ không thể tránh khỏi những ñiều không mong muốn. Tôi rất mong nhận ñược sự ñộng viên và những ý kiến ñóng góp chân thành của Quý Thầy cô và các em học sinh ñể Chuyên ñề tiếp tục ñược hoàn thiện hơn.

Tôi xin chân thành cảm ơn!

Page 5: Kỷ yếu hội thảo các trường chuyên dh db bắc bộ lần III

=========================================================== 7

HỘI CÁC TRƯỜNG THTP CHUYÊN KHU VỰC DUYÊN HẢI VÀ ðỒNG BẰNG BẮC BỘ

Hội thảo khoa học môn Toán học lần thứ III - 2010

§1. MỘT SỐ PHƯƠNG PHÁP GIẢI PHƯƠNG TRÌNH VÔ TỶ 1. MỘT SỐ QUY ƯỚC KHI ðỌC CHUYÊN ðỀ

1.1 Vt: Vế trái của phương trình. Vt 2 : Bình phương của vế trái phương trình. 1.2 Vp: Vế phải của phương trình. Vp 2 : Bình phương của vế phải phương trình. 1.3 Vt (1) : Vế trái của phương trình (1) . 1.4 Vp (1) : Vế phải của phương trình (1) . 1.5 ðk, ñk: ðiều kiện. 1.6 BðT: Bất ñẳng thức. HSG, HSG: Học sinh giỏi. 1.7 VMO, VMO: Thi học sinh giỏi Việt Nam, CMO: Thi học sinh giỏi Canada. 2. PHƯƠNG PHÁP ðẶT ẨN PHỤ 2.1 Một số lưu ý

Khi giải phương trình vô tỷ bằng phương pháp ñặt ẩn phụ ta có thể gặp các dạng như:

2.1.1 ðặt ẩn phụ ñưa phương trình ñã cho về phương trình ñại số không còn chứa căn thức với ẩn mới là ẩn phụ.

2.1.2 ðặt ẩn phụ mà vẫn còn ẩn chính, ta có thể tính ẩn này theo ẩn kia. 2.1.3 ðặt ẩn phụ ñể ñưa phương trình về hệ hai phương trình với hai ẩn là hai ẩn phụ,

cũng có thể hai ẩn gồm một ẩn chính và một ẩn phụ, thường khi ñó ta ñược một hệ ñối xứng. 2.1.4 ðặt ẩn phụ ñể ñược phương trình có hai ẩn phụ, ta biến ñổi về phương trình

tích với vế phải bằng 0. Thường giải phương trình ta hay biến ñổi tương ñương, nếu biến ñổi hệ quả thì nhớ

phải thử lại nghiệm. 2.2 Một số ví dụ

Ví dụ 1. Giải các phương trình sau:

1) 218 18 17 8 2 0x x x x x− − − − = .

2) 2 4 233 1 1

3x x x x− + = − + + .

3) 22

1 12 2 4x x

x x

− + − = − +

.

4) 2 22 1 2 1 1x x x x+ − + − = .

Hướng dẫn (HD): 1) ðặt x y= với 0y ≥ . Khi ñó phương trình ñã cho trở thành

2 2(3 4 2)(6 2 1) 0y y y y− − + + = , suy ra 2(3 4 2) 0y y− − = , ta ñược 2 10

3y

+= . Từ ñó

phương trình có nghiệm là 14 4 10

9x

+= .

2) Ta có 4 2 2 2 2 2 21 ( 1) ( 1)( 1) 0x x x x x x x x+ + = + − = + + − + > , với mọi x.

Mặt khác 2 2 23 1 2( 1) ( 1)x x x x x x− + = − + − + + .

ðặt 2

2

1

1

x xy

x x

− +=

+ + (có thể viết ñk 0y ≥ hoặc chính xác hơn là

33

3y≤ ≤ ), ta ñược

Page 6: Kỷ yếu hội thảo các trường chuyên dh db bắc bộ lần III

=========================================================== 8

HỘI CÁC TRƯỜNG THTP CHUYÊN KHU VỰC DUYÊN HẢI VÀ ðỒNG BẰNG BẮC BỘ

Hội thảo khoa học môn Toán học lần thứ III - 2010

2 232 1 0 6 3 3 0

3y y y y− = − = ⇔ + − = , ta ñược

3

3y = (loại

3

2y = − ).

Từ ñó phương trình có nghiệm là 1x = . 3) Ta thấy 0x < không thỏa mãn.

Khi ñó phương trình tương ñương với hệ

2 2

22

0

14 0

1 12 2 4

1

x

xx

x xx

>

− + >

− + − = − +

.

ðặt1

x yx

+ = , ta ñược 2 2 2

2 4(1)

4 ( 2) 2 5 2( 2) (4 ) (2)

y

y y y

≤ <

− − + − − = −.

Xét 2 2(2) 9 2 4 5y y y⇔ − = − + 4 3 28 28 40 16 0y y y y⇔ − + − + = (do hai vế không

âm).

3 2

2

( 2)( 6 16 8) 0

( 2)(( 2)( 4 8) 8) 0

y y y y

y y y y

⇔ − − + − =

⇔ − − − + + =

Dẫn ñến 2y = (do 2(( 2)( 4 8) 8) 0y y y− − + + > với mọi y thỏa mãn (1)). Từ ñó phương trình có nghiệm là 1x = .

Nhận xét: Bài toán này ta có thể giải bằng Phương pháp ñánh giá trong phần sau. 4) Ta có phương trình tương ñương với

2 21 1 2 2 1x x x x− = − − −

4 2 2 2 2 3 21 1 4 4 (1 ) 4 4 1 8 1x x x x x x x x x⇒ − = + + − − − − + −

2 2 2

2 2 2

(1 4 1 8 1 ) 0

0

1 4 1 8 1 0(1)

x x x x

x

x x x

⇔ − − + − =

=⇔

− − + − =

Xét (1), ñặt 21y x= − , suy ra 0y ≥ và 2 21x y= − .

Ta ñược 2 31 4 8 (1 ) 0 8 4 1 0y y y y y− + − = ⇔ − − =

2(2 1)(4 2 1) 0y y y⇔ + − − =

1 5

4y

+⇔ = . Từ ñó suy ra

5 5

8x

−= ± .

Thử lại ta ñược nghiệm của phương trình là 0x = và 5 5

8x

−= − .

Nhận xét: Bài toán này ta có thể giải bằng Phương pháp lượng giác trong phần sau.

Ví dụ 2. Giải phương trình 2 23 1 ( 3) 1x x x x+ + = + + .

HD: ðặt 2 1x y+ = , với 1y ≥ . Khi ñó ta ñược 2 3 ( 3)y x x y+ = + ⇔ ( 3)( ) 0y y x− − = .

Dẫn ñến 3y = và y x= . Từ ñó phương trình có nghiệm là 2x = ± .

Page 7: Kỷ yếu hội thảo các trường chuyên dh db bắc bộ lần III

=========================================================== 9

HỘI CÁC TRƯỜNG THTP CHUYÊN KHU VỰC DUYÊN HẢI VÀ ðỒNG BẰNG BẮC BỘ

Hội thảo khoa học môn Toán học lần thứ III - 2010

Ví dụ 3. Giải phương trình 8 3 84 17 2 1 1x x− − − = .

HD: ðặt 84 17 x y− = với 0y ≥ và 3 82 1x z− = . Khi ñó ta ñược hệ

4 3 4 3

1 1

2 33 2 ( 1) 33

y z z y

y z y y

− = = − ⇔

+ = + − = .

Xét 4 3 3 22 ( 1) 33 ( 2)(2 5 7 17) 0y y y y y y+ − = ⇔ − + + + = . Suy ra ñược y - 2 = 0. Từ ñó nghiệm của phương trình là x = 1 và x = -1. Ví dụ 4. Giải các phương trình sau:

1) 2 24 2 3 4x x x x+ − = + − .

2) 3 23 481 8 2 2

3x x x x− = − + − .

HD: 1) ðặt 24 x y− = , với 0 2y≤ ≤ .

Khi ñó ta ñược hệ 2 2

2 3

4

x y xy

x y

+ = +

+ =.

Thế hoặc lại ñặt ;x y S xy P+ = = rồi giải tiếp ta ñược nghiệm của phương trình là

0x = ; 2x = và 2 14

3x

− −= .

2) ðặt 3 23 481 8 2 3 3 2

3x y x y y y− + = ⇒ = − + .

Khi ñó ta ñược hệ

3 2

3 2

43 2

34

3 23

x y y y

y x x x

= − +

= − +

.

Xét hiệu hai phương trình dẫn ñến x y= (do 2 2 21 1 1 1( ) ( 2) ( 2) 0

2 2 2 3x y x y+ + − + − + > ).

Thay vào hệ và giải phương trình ta ñược 3 2 6

0;3

x x±

= = .

Ví dụ 5. Giải phương trình 2 25 14 9 20 5 1x x x x x+ + − − − = + . HD: ðk 5x ≥ . Với ñiều kiện ñó ta biến ñổi phương trình ñã cho như sau:

2 2

2 2

5 14 9 20 5 1

5 14 9 20 25( 1) 10 ( 1)( 4)( 5)

+ + = − − + +

⇔ + + = − − + + + + + −

x x x x x

x x x x x x x x

22 5 2 5 ( 1)( 5) 4⇔ − + = + − +x x x x x

2( 1)( 5) 3( 4) 5 ( 1)( 5) 4⇔ + − + + = + − +x x x x x x

ðặt ( 1)( 5) ; 4x x y x z+ − = + = , với 0; 3y z≥ ≥ .

Page 8: Kỷ yếu hội thảo các trường chuyên dh db bắc bộ lần III

=========================================================== 10

HỘI CÁC TRƯỜNG THTP CHUYÊN KHU VỰC DUYÊN HẢI VÀ ðỒNG BẰNG BẮC BỘ

Hội thảo khoa học môn Toán học lần thứ III - 2010

Ta ñược 2 22 3 5 ( )(2 3 ) 0y z yz y z y z+ = ⇔ − − = , từ ñó ta ñược 3

2

y z

y z

= =

.

Nếu y z= thì ta ñược 5 61

2x

+= (do 5x ≥ ).

Nếu 3

2y z= thì ta ñược

78;

4x x= = − . Vậy phương trình có ba nghiệm trên.

Ví dụ 6. Giải phương trình 2 4 97 7

28

xx x

++ = , với 0x > .

Nhận xét: Dạng phương trình này ta thường ñặt 4 9

28

xay b

+= + , sau ñó bình

phương lên rồi ta “cố ý” biến ñổi về hệ ñối xứng với hai ẩn ,x y . Từ ñó ta sẽ biết ñược giá

trị của a, b. Với bài toán này ta tìm ñược 1

1;2

a b= = . (Nếu a = 1 và b = 0 mà giải ñược thì

ñó là phương trình quá ñơn giản, ta không xét ở ñây).

HD: ðặt 4 9 1

28 2

xy

+= + , do 0x > nên

4 9 9 1

28 28 2

x +> > , từ ñó 0y > .

Ta ñược hệ

2

2

17 7

21

7 72

, 0

x x y

y y x

x y

+ = +

+ = +

>

. Giải hệ bình thường theo dạng ta ñược 6 50

14x

− += .

Ví dụ 7. Giải phương trình 3 2 32 2x x− = − .

Nhận xét: Khi giải một phương trình không phải lúc nào cũng có nghiệm thực, có những phương trình vô nghiệm nhưng khi cho học sinh làm bài ta cũng kiểm tra ñược năng lực của học sinh khi trình bầy lời giải bài toán ñó. Chẳng hạn như bài toán trong ví dụ này.

HD: ðặt 3 2 32 2x x− = − = y với 0y ≥ . Khi ñó ta ñược hệ 2 3

3 2

2

2

x y

x y

= +

= − và từ

phương trình ban ñầu ta có 2x ≤ − . Xét hiệu hai phương trình của hệ ta ñược phương trình 2 2( )( ) 0x y x xy y x y+ − + − + = .

Với x y= − thì 3 2 2x x= − − , dẫn ñến vô nghiệm.

Còn 2 2 2( )(1 ) 0x xy y x y y x x y− + − + = − − + > với mọi 0y ≥ và 2x ≤ − . Do ñó hệ vô nghiệm hay phương trình ñã cho vô nghiệm.

2.3 Một số bài tập tương tự Bài 1. Giải các phương trình sau:

1) 2 22 2 2x x x x+ − = − .

(HD: ðặt 2 ; 0y x y= − ≥ , ta ñược 2 2( 1)( 1)(2 4) 0y y y y y− + − − − = .

Page 9: Kỷ yếu hội thảo các trường chuyên dh db bắc bộ lần III

=========================================================== 11

HỘI CÁC TRƯỜNG THTP CHUYÊN KHU VỰC DUYÊN HẢI VÀ ðỒNG BẰNG BẮC BỘ

Hội thảo khoa học môn Toán học lần thứ III - 2010

Từ ñó 5 1 33 1

1; ;2 8

y y y− +

= = = và ñược nghiệm của phương trình là

5 1 33 11; ;

2 8x x x

+ += = = − ).

2) 2 32 5 1 7 1x x x+ − = − .

(HD: Từ phương trình suy ra 1x ≠ . ðặt 2 1

1

x xy

x

+ +=

−, bình phương dẫn ñến

3 2 3y ≥ + . Phương trình trở thành 22 7 3 0y y− + = , ta ñược 3y = . Từ ñó 4 6x = ± ).

Bài 2. Giải phương trình 2 2(4 1) 1 2 2 1x x x x− + = + + .

(HD: ðặt 2 1x y+ = , với 1y ≥ . Từ ñó ta ñược 1

2 12

y y x= ∨ = − . Phương trình có

nghiệm 4

3x = ).

Bài 3. Giải các phương trình sau:

1) 3(2 2) 2 6x x x+ − = + + .

(HD: ðặt 3 2 , 6x y x z− = + = , với 0; 0y z≥ ≥ .

Ta ñược 3 4x y z= ∨ + = . Từ ñó phương trình có 2 nghiệm 11 3 5

3;2

x x−

= = ).

2) 42 2(1 ) 2 1x x− + + = .

(HD: ðk 0 2 1x≤ ≤ − . ðặt 42 2(1 ) 2 2 1x y y x− + = ⇔ = − −

và 44 42 2x z z x= ⇔ = với 0; 0y z≥ ≥ .

Suy ra 4

2 4

2( ) 1(1)

2 1(2)

y z

y z

+ =

+ = −. Từ (1) thay

4

1

2y z= − vào (2) ta ñược

2 2 2

4

1( 1) ( ) 0

2z z+ − + = . Xét hiệu hai bình phương suy ra

44 3 21

4 22

z

−±

= .

Từ ñó ta ñược nghiệm của phương trình là

44

4

4 3 21

22

x

− ±

=

).

Bài 4. Giải phương trình 2 1000 1 8000 1000x x x− − + = .

(HD: ðặt 1 1 8000x+ + = 2y , ta ñược 2

2

2000(*)

2000

x x y

y y x

− =

− =.

Page 10: Kỷ yếu hội thảo các trường chuyên dh db bắc bộ lần III

=========================================================== 12

HỘI CÁC TRƯỜNG THTP CHUYÊN KHU VỰC DUYÊN HẢI VÀ ðỒNG BẰNG BẮC BỘ

Hội thảo khoa học môn Toán học lần thứ III - 2010

Từ (*) suy ra ( )( 1999) 0x y x y− + + = và , do ñó 1999 0x y+ + > . Suy ra x y= , ta ñược nghiệm 2001x = , loại 0x = ). Bài 5. Giải các phương trình sau:

1) 3

2

1 2

2 5

x

x

+=

+.

(HD: ðặt 21 0; 1y x z x x= + ≥ = − + , ta ñược 2

2 2 55 2( ) 2 2

y yyz y z

z z

= + ⇔ = +

25 1

2 2 0 22

y y y y

z z z z

⇔ − + = ⇔ = ∨ =

.

Nếu 2y

z= ta ñược 21 2 1x x x+ = − +

2

1

4 5 3 0

x

x x

≥ −⇔

− + = (vô nghiệm).

Nếu 1

2

y

z= ta ñược 22 1 1x x x+ = − +

15 37

5 37 22

x

xx

≥ −±

⇔ ⇔ = ±=

(thỏa mãn)).

2) 2 32 5 2 4 2( 21 20x x x x− + = − − .

(HD: ðk 4 1

5

x

x

− ≤ ≤ − ≥

. ðặt 22 8 10x x y− − = và 4x z+ = , với 0; 0y z≥ ≥ .

Khi ñó ta ñược ( )( 3 ) 0y z y z− − = . Từ ñó phương trình có bốn nghiệm là 9 193

4x

±=

và 17 3 73

4x

±= ).

Bài 6. Giải các phương trình sau:

1) 2 4 3 5x x x− − = + .

(HD: ðặt 5 2x y+ = − , ta ñược 5 29

1;2

x x+

= − = ).

2) 2 32 4

2

xx x

++ = , với 1x ≥ .

(HD: ðặt 3

12

xy

+= + ,ñược

3 171

4x

− += < (loại), nếu 1x ≥ − thì

3 17

4x

− += ).

3) 2 427 18

3x x x+ = + , với 0x > .

(HD: Tương tự, ta ñược 5 37

18x

− += ).

3. PHƯƠNG PHÁP ðÁNH GIÁ 3.1 Một số lưu ý

Khi giải phương trình vô tỷ (chẳng hạn ( ) ( )f x g x= ) bằng phương pháp ñánh giá, thường là ñể ta chỉ ra phương trình chỉ có một nghiệm (nghiệm duy nhất).Ta thường sử dụng

Page 11: Kỷ yếu hội thảo các trường chuyên dh db bắc bộ lần III

=========================================================== 13

HỘI CÁC TRƯỜNG THTP CHUYÊN KHU VỰC DUYÊN HẢI VÀ ðỒNG BẰNG BẮC BỘ

Hội thảo khoa học môn Toán học lần thứ III - 2010

các bất ñẳng thức cổ ñiển Cô si, Bunhiacopxki, ñưa vế trái về tổng bình phương các biểu thức, ñồng thời vế phải bằng 0. Ta cũng có thể sử dụng tính ñơn ñiệu của hàm số (có thể thấy ngay hoặc sử dụng ñạo hàm xét sự biến thiên của hàm số) ñể ñánh giá một cách hợp lý.

Thường ta ñánh giá như sau:

( ) ( )

( ) ( ) ( ) ( )

( ) ( )

f x g x

f x C C f x g x C

g x C C

=

≥ ≤ ⇔ = = ≤ ≥

, hoặc ñánh giá

( ) ( )f x g x≥ cũng như là ( ) ( )f x g x≤ … Ngoài ra ñối với bài cụ thể nào ñó ta sẽ có cách ñánh giá khác. Cũng có một số phương trình vô tỷ có nhiều hơn một ẩn mà ta giải bằng phương pháp

ñánh giá.

3.2 Một số ví dụ

Ví dụ 1. Giải phương trình 24 1 4 1 1x x− + − = .

HD: Bài toán này có trong ñề thi vào ðại học Bách Khoa và ðHQG năm 2001. Bài này có nhiều cách giải, ñáp án sử dụng ñạo hàm.

Ta có thể làm ñơn giản như sau: Ta thấy 1

2x = là nghiệm của phương trình.

Nếu 1

2x > thì Vt > 1 = Vp.

Nếu 1

2x < thì Vt < 1 = Vp.

Do ñó phương trình không có nghiệm trong hai trường hợp này.

Vậy phương trình có một nghiệm là 1

2x = .

Ví dụ 2. Giải phương trình 2 2 23 6 7 5 10 14 4 2x x x x x x+ + + + + = − − . HD: Bài này quá ñơn giản, ñánh giá Vt 5≥ còn Vp 5≤ , do ñó hai vế cùng bằng 5.

Ta ñược phương trình có nghiệm duy nhất là 1x = − .

Ví dụ 3. Giải phương trình 2 2 219 7 8 13 13 17 7 3 3( 2)x x x x x x x− + + + + + + + = + . HD: Bài này cách giải có vẻ hơi mất tự nhiên bởi cách “cố ý” cho như vậy. Giáo viên

và học sinh có thể sáng tác những bài kiểu ñó. ðk 2x ≥ − . Với ñk ñó Vt =

2 2 2 2 21 75 1 3( ) (2 1) 3( 2) (2 1) (4 3)

2 4 4 4x x x x x− + + − + + + − + +

75 3

3 2 4 34 2

x x≥ + + + +

5 3

3 3( 2) (4 3)2 2

x x≥ + + + +

3 3.( 2)x≥ + = Vp.

Dấu ñẳng thức xảy ra khi 1

2x = . Vậy phương trình có nghiệm duy nhất là

1

2x = .

Page 12: Kỷ yếu hội thảo các trường chuyên dh db bắc bộ lần III

=========================================================== 14

HỘI CÁC TRƯỜNG THTP CHUYÊN KHU VỰC DUYÊN HẢI VÀ ðỒNG BẰNG BẮC BỘ

Hội thảo khoa học môn Toán học lần thứ III - 2010

Ví dụ 4. Giải phương trình 2428 27

2 27 24 1 63 2

x x x+ + = + + .

HD: Phương trình ñã cho tương ñương với phương trình

2

4(9 4) 3(9 4)

2 4 13 2

x x+ ++ = + , ñk

4

9x ≥ − . ðặt (9 4)x y+ = , suy ra 0y ≥ .

Khi ñó ta ñược 2 2

43 3

2 4 1 4 4 1 63 2 3 2

y y y yy+ = + ⇔ + = + + (bình phương hai vế).

Theo BðT Cô-si ta ñược 6

62

yy

+≤ , do ñó

2 224 4 2 4 4 4 ( 2)

3 3

y yy y

+ ≤ + ⇔ + ≤ +

2 2

2

2

4 48 3 12 12

12 36 0

( 6) 0.

y y y

y y

y

⇔ + ≤ + +

⇔ − + ≤

⇔ − ≤

Từ ñó ta ñược 6y = , suy ra 2

9x = thỏa mãn ñk.

Vậy phương trình có nghiệm duy nhất là 2

9x = .

Ví dụ 5. Giải phương trình 2

4 3 232 7 3 3 2

2

x xx x x x

−+ − + − + = .

HD: Phương trình ñã cho tương ñương với

2 2 22 2 3 4 (2 1) ( 3)

(2 1)( 3) (1)2 2

x x x x xx x x

− + − + + +− + + = = . Phương trình xác ñịnh

với mọi x là số thực. Theo BðT Cô-si cho hai số dương ta ñược Vt(1) ≤ Vp(1). Do ñó (1) ⇔ 2 2 22 1 3 2 0x x x x x− + = + ⇔ − − = . Từ ñó phương trình có nghiệm là

1x = − và 2x = .

Ví dụ 6. Giải phương trình 22

1 12 2 4x x

x x

− + − = − +

.

HD: ðk

22

2

22

2

x

x

− ≤ ≤ −

≤ ≤

. Với ñk ñó, phương trình ñã cho tương ñương với

phương trình 22

1 12 2 4(1)x x

x x− + − + + = .

Page 13: Kỷ yếu hội thảo các trường chuyên dh db bắc bộ lần III

=========================================================== 15

HỘI CÁC TRƯỜNG THTP CHUYÊN KHU VỰC DUYÊN HẢI VÀ ðỒNG BẰNG BẮC BỘ

Hội thảo khoa học môn Toán học lần thứ III - 2010

Theo BðT Bunhiacopxki, ta ñược

2 2 2 2

2 2

2 2

( 2 ) ( 2 .1 .1) 4

1 1 1 12 2 .1 .1 4

x x x x

x x x x

− + = − + ≤

− + = − + ≤

.

Suy ra Vt (1) 4≤ = Vp (1) . Do ñó

2

2

2 2(1) 1 1

2 2

x x

x x

− + =

⇔ − + =

, nghĩa là dấu bằng trong hệ

xảy ra. Từ ñó phương trình có nghiệm duy nhất là 1x = .

Ví dụ 7. Giải phương trình 2 2

91

x xx

+ = ++

.

HD: ðk 0x ≥ .

Theo BðT Bunhiacopxki, ta ñược

2Vt =

2

1 12 2 1 ( 9)

1 11 1

x xx x

x xx x

+ + ≤ + + = + ++ +

2Vp .

Phương trình có nghiệm khi dấu ñẳng thức xảy ra hay

12 2 1

1

1

x

x x

x

+=+

+

1

7x⇔ = .

Vậy phương trình có nghiệm duy nhất là 1

7x = .

Ví dụ 8. Giải phương trình 2 4 2 413 9 16x x x x− + + = .

HD: ðk 1 1x− ≤ ≤ . Với ñk ñó phương trình tương ñương với

2 2 2 2 2 2(13 1 9 1 ) 16 (13 1 9 1 ) 256(1)x x x x x x− + + = ⇔ − + + =

Theo BðT Bunhiacopxki, ta ñược 2 2 2 2 2 2(13 1 9 1 ) ( 13. 13 1 3. 3. 3 1 )x x x x− + + = − + +

2 2

2

(13 27)(13(1 ) 3(1 ))

40(16 10 ).

x x

x

≤ + − + +

= −

Theo BðT Cô-si cho hai số dương ta ñược 22 2

2 2 10 (16 10 )10 (16 10 ) 64

2

x xx x

+ −− ≤ =

.

Do ñó Vt(1) ≤ 4 64 256. = , ta ñược

(1)

22 22

22 2

1 9 9 113

20 1610 16 10

xx xx

xx x

+ − = +− = ⇔ ⇔

= = −

. Từ ñó dẫn ñến 2 5

5x = ± .

Vậy phương trình có hai nghiệm là 2 5

5x = ± .

Page 14: Kỷ yếu hội thảo các trường chuyên dh db bắc bộ lần III

=========================================================== 16

HỘI CÁC TRƯỜNG THTP CHUYÊN KHU VỰC DUYÊN HẢI VÀ ðỒNG BẰNG BẮC BỘ

Hội thảo khoa học môn Toán học lần thứ III - 2010

Ví dụ 9. Giải phương trình 3 2 32 2x x− = − .

Nhận xét: Trong phần giải phương trình vô tỷ bằng Phương pháp ñặt ẩn phụ ta ñã giải bài toán này, ta cũng có thể giải nó bằng phương pháp ñánh giá như sau.

HD: ðk 3 32 0 2x x− ≥ ⇔ ≤ .

Giả sử x là nghiệm của phương trình. Khi ñó 2 2 0x − ≥ ⇔2

2

x

x

≤ −, ta ñược 2x ≤ − .

Mũ 6 hai vế suy ra 9 6 4 3 26 12 4 4 0x x x x x− + + − − = (*). Cách thứ nhất ta biến ñổi Vt thành 9 6 2 4 2 3 25 ( 1) 12 3 4x x x x x x x− − − + + − − là một biểu

thức âm khi 2x ≤ − . Cách thứ hai ta biến ñổi Vt thành 9 4 2 3 2(6 1) 12 4 4x x x x x− − + − − cũng là một biểu thức

âm khi 2x ≤ − … Ta có thể biến ñổi tiếp phương trình (*) sau khi chia hai vế cho 1 0x − ≠ , ta ñược 8 7 6 5 4 3 25 5 4 8 4 4 0x x x x x x x x+ + − − − + + + =

6 2 4 2 2( 1) 5 ( 1) 4 ( 1) 4(2 1) 0x x x x x x x x⇔ + + − + − − + + = vô nghiệm vì Vt luôn dương

khi 2x ≤ − . Vậy phương trình vô nghiệm.

Ví dụ 10. Giải phương trình ( 2)(2 1) 3 6 4 ( 6)(2 1) 3 2x x x x x x+ − − + = − + − + + .

HD: Biến ñổi phương trình thành ( 6 2)( 2 1 3) 4x x x+ + + − − = , suy ra 5x ≥ .

Vt là hàm số ñồng biến trên ñoạn [ )5;+∞ . Từ ñó dẫn ñến 7x = là nghiệm duy nhất của

phương trình ñã cho.

Ví dụ 11. Giải phương trình 2 32 11 21 3 4 4 0x x x− + − − = .

HD: Phương trình tương ñương với

2 33

12( 3)( 3)(2 5)

(4 4) 2 4 4 4

xx x

x x

−− − =

− + − +.

Ta thấy 3x = là nghiệm của phương trình.

Nếu 3x ≠ thì phương trình tương ñương với 2 33

12(2 5) (1)

(4 4) 2 4 4 4x

x x− =

− + − +

Nếu 3x > thì Vt(1) > 1 > Vp(1). Nếu 3x < thì Vt(1) < 1 < Vp(1). Vậy phương trình có nghiệm duy nhất là 3x = .

Ví dụ 12. Giải phương trình 2 2 2 22 1 3 2 2 2 3 6x x x x x x x− + − + = + + + − + .

Nhận xét: Với bài toán này ta sử dụng một ñánh giá ít gặp sau ñây: ( ) 0; ( ) 0

( ) ( ) ( ) ( ) ( ) ( )( ) 0

f x g xf x g x f x ah x g x bh x

h x

≥ ≥+ = + + + ⇔

=, với a, b là hai

số thực dương. HD: Biến ñổi phương trình

Page 15: Kỷ yếu hội thảo các trường chuyên dh db bắc bộ lần III

=========================================================== 17

HỘI CÁC TRƯỜNG THTP CHUYÊN KHU VỰC DUYÊN HẢI VÀ ðỒNG BẰNG BẮC BỘ

Hội thảo khoa học môn Toán học lần thứ III - 2010

2 22 2 2 2 2 1 0; 3 2 0

2 1 3 2 2 1 2( 2) 3 2 2( 2)2 0

x x xx x x x x x x x

x

− ≥ − + ≥− + − + = − + + + − + + + ⇔

+ =

Từ ñó ta ñược phương trình có nghiệm là 2x = − .

Ví dụ 13. Giải phương trình 16 1

10 ( 1996 2008)1996 2008

x yx y

+ = − − + −− −

.

Nhận xét: Với bài toán này, ta thấy ñây là một phương trình gồm hai ẩn. Do ñó ta

nghĩ ñến biến ñổi phương trình thành phương trình mới có Vt là tổng các bình phương, còn Vp bằng 0.

HD: Biến ñổi phương trình thành

22

4 44 4

4 11996 2008 0

1996 2008x y

x y

− − + − − = − −

.

Từ ñó ta ñược phương trình có nghiệm là ( ; ) (2012;2009)x y = .

Ví dụ 14. Giải phương trình 3

1 2 12

x y y x xy− + − = .

HD: ðk 1; 1x y≥ ≥ .

Ta có 1 3

1 2 1 ( 2 1) ( 2 1)2 2

x y y x y x x x y y xy− + − = − − − − − − +

2 21 3( 1 1) ( 1 1)

2 2y x x y xy= − − − − − − + .

Khi ñó phương trình ñã cho tương ñương với 2 2

1; 1

1( 1 1) ( 1 1) 0

2

x y

y x x y

≥ ≥

− − + − − =

.

Từ ñó ta ñược phương trình có nghiệm là ( ; ) (2;2)x y = . 3.3 Một số bài tập tương tự: (Chuyên ñề còn tiếp tục hoàn thiện) 4. PHƯƠNG PHÁP LƯỢNG GIÁC 4.1 Một số lưu ý

Khi giải phương trình vô tỷ bằng phương pháp lượng giác ta có thể ñặt

( ) sinf x α= nếu [ ]( ) 1;1f x ∈ − với ñiều kiện ;2 2

π πα

∈ −

hoặc ( ) cosf x α= với ñiều

kiện

[ ]0;α π∈ . Cũng có khi ñặt ( ) tan ; ( ) cotf x f xα α= = … ñể ñưa phương trình ñã cho

về phương trình lượng giác. Giải phương trình lượng giác rồi từ ñó tìm nghiệm của phương trình ñã cho.

4.2 Một số ví dụ

Ví dụ 1. Giải phương trình 24 1 4 1 1x x− + − = .

Page 16: Kỷ yếu hội thảo các trường chuyên dh db bắc bộ lần III

=========================================================== 18

HỘI CÁC TRƯỜNG THTP CHUYÊN KHU VỰC DUYÊN HẢI VÀ ðỒNG BẰNG BẮC BỘ

Hội thảo khoa học môn Toán học lần thứ III - 2010

Nhận xét: Bài toán này (ñã xét ở trên) cũng có thể giải bằng phương pháp lượng giác, tuy nhiên với bài này cách giải bằng lượng giác chỉ mang tính chất tham khảo.

HD: ðặt 4

24

4 1 cos; 0;

24 1 sin

x yy

x y

π − = ∈ − =

. Khi ñó ta ñược phương trình

8 4 2

2 6 4 2

cos 2cos 8cos 7 0

( 1)(...) 0

(cos 1)(cos cos cos 7) 0

cos 1

y y y

cosy

y y y y

y

− + − =

⇔ − =

⇔ − + − + =

⇔ =

Do vậy phương trình có một nghiệm là 1

2x = .

Ví dụ 2. Giải phương trình 2

1 12 2

1x x+ =

−.

HD: ðặt cos , (0; ),2

x y y yπ

π= ∈ ≠ . Phương trình ñã cho trở thành

1 12 2 sin cos 2.sin 2

cos siny y y

y y+ = ⇔ + = . ðặt sin cos , 2 2y y z z+ = − ≤ ≤ .

suy ra 2sin 2 2sin cos 1y y y z= = − , ta ñược 2z = và 2

2z = − .

Với 2z = thì 4

= , do ñó 2

2x = .

Với 2

2z = − thì

11

12y

π= , do ñó

1 3

2 2x

+= − .

Vậy phương trình có nghiệm là 2

2x = và

1 3

2 2x

+= − .

Ví dụ 3. Giải phương trình 3 2 3 2(1 ) 2(1 )x x x x+ − = − .

HD: ðk 1 1x− ≤ ≤ .

ðặt sin , ;2 2

x y yπ π

= ∈ − suy ra cos 0y ≥ .

Khi ñó phương trình trở thành 3 3sin cos 2 sin cosy y y y+ = .

ðặt sin cos , 2; 2y y z z + = ∈ − (chính xác là 1; 2z ∈ − ), biến ñổi phương trình

ta ñược 3 22. 3 2 0z z z+ − − = ( 2)( 2 1)( 2 1) 0z z z⇔ − + − + + =

2 1 2z z⇔ = ∨ = − .

Nếu 2z = thì thì 4

= , do ñó 2

2x = .

Nếu 1 2z = − thì sin cos 1 2y y+ = − 21 1 2x x⇔ + − = −

Page 17: Kỷ yếu hội thảo các trường chuyên dh db bắc bộ lần III

=========================================================== 19

HỘI CÁC TRƯỜNG THTP CHUYÊN KHU VỰC DUYÊN HẢI VÀ ðỒNG BẰNG BẮC BỘ

Hội thảo khoa học môn Toán học lần thứ III - 2010

21 1 2 0

1 2 2 2 1

2

x x

x

⇔ − = − − ≥

− − −⇔ =

Vậy phương trình có 2 nghiệm trên.

4.3 Một số bài tập tương tự

Bài 1. Giải phương trình 3 24 3 1x x x− = − . (HD: ðặt cosx y= , phương trình có tập nghiệm là

5 3 2cos ;cos ;cos

8 8 4 2S

π π π = = −

).

Bài 2. Giải phương trình ( )2 6 2 35 3 1 8 (1 )x x x+ − = + − .

Bài 3. Giải phương trình 2

2 21

xx

x+ =

−.

Bài 4. Giải phương trình 2 2( 3 2 ) 1 3 2x x x x− − = − .

Bài 5. Giải phương trình 2

22

(1 )3 1

1

x xx

x

+= −

−.

Bài 6. Giải phương trình 2 3

25 3

(1 )1

6 20 6

xx

x x x

+= +

− +.

Bài 7. Giải phương trình 2 22 1 2 1 1x x x x+ − + − = . 5. MỘT SỐ PHƯƠNG PHÁP KHÁC 5.1 Một số lưu ý

Ngoài những phương pháp thường gặp ở trên, ñôi khi ta cũng có những lời giải khác lạ ñối với một số phương trình vô tỷ. Cũng có thể ta sử dụng kết hợp các phương pháp ở trên ñể giải một phương trình.

5.2 Một số ví dụ

Ví dụ 1. Giải phương trình 2 23 2. 9 4 2. 16 5x x x x− + + − + = . HD: Nếu 0x ≤ thì Vt 3 4 7 5≥ + = > = Vp (phương trình không có nghiệm).

Nếu 0x > thì ta xét tam giác vuông ABC với 090A = , AB = 4; AC = 3. Gọi AD là phân giác của góc A, lấy M thuộc tia AD.

ðặt AM = x, xét 2 2 9 3 2.ACM CM x x∆ ⇒ = + − và xét 2 2 16 4 2.ABM BM x x∆ ⇒ = + − .

Từ ñó suy ra Vt = 5CM BM BC+ ≥ = . Dấu ñẳng thức xảy ra khi M D≡ ,hay

Page 18: Kỷ yếu hội thảo các trường chuyên dh db bắc bộ lần III

=========================================================== 20

HỘI CÁC TRƯỜNG THTP CHUYÊN KHU VỰC DUYÊN HẢI VÀ ðỒNG BẰNG BẮC BỘ

Hội thảo khoa học môn Toán học lần thứ III - 2010

2 2

2 2

3

4

1 6 9

1 6 1 6 .9 4 8 2 . 9 1 6 .9 3 6 2 .

7 1 2 2 . 0

1 2 2

7

C M

B M

C M B M

x x x x

x x

x

=

⇔ =

⇔ + − = + −

⇔ − =

⇔ =

Vậy phương trình có nghiệm là 12 2

7x = .

Ví dụ 2. Giải phương trình 2 2 2 444 4 1 2 3 5 16x x x y y y x− + + + + − − = − + − .

Nhận xét: Bài toán này không khó, chỉ kiểm tra tính cẩn thận của học sinh mà thôi vì

sau khi ñặt ñiều kiện ñã tìm ñược giá trị của x. Tuy nhiên nếu học sinh học hời hợt sẽ ngồi nhìn mà không làm ñược bài.

HD: ðặt ñk cho phương trình xác ñịnh ta sẽ ñược 2x = . Khi ñó phương trình trở

thành 1 2y y− = − , suy ra 3

2y = . Vậy phương trình có một nghiệm là

3( ; ) 2;

2x y

=

.

Ví dụ 3. Giải phương trình 3 2 3 23 7 1 8 8 1 2x x x x x+ − − − + − − = .

HD: ðặt 3 2 3 23 7 1; 8; 8 1y x z x x t x x= + − = − − = − − ,

suy ra 2y z t+ + = và 3 3 3 8y z t+ + = (1).

Mặt khác ( )3

8y z t+ + = (2).

Từ (1) và (2) ta ñược 3 3 3 3( ) ( ) 3( )( )( ) 0y z t y z t y z z t t y+ + − + + = + + + =

0 (3)

0 (4)

0 (5)

y z y z

z t z t

t y t y

+ = = − ⇔ + = ⇔ = − + = = −

.

Xét (3) ta ñược 1 9x x= − ∨ = , xét (4) ñược 1x = và (5) ñược 0 1x x= ∨ = . Vậy tập nghiệm của phương trình là { }1;0;1;9S = − .

Ví dụ 4. Giải phương trình 2 24 20 4 29 97x x x x− + + + + = .

HD: Trong mặt phẳng tọa ñộ xét hai véc tơ ( 2;4)a x= −r

và ( 2;5)b x= − −r

.

Khi ñó ta ñược ( 4;5)a b+ = −r r

, suy ra 97a b+ =r r

và ta cũng có 2 4 20a x x= − +r

,

2 4 29b x x= + +r

. Phương trình trở thành a b a b+ = +r r r r

, ñẳng thức ñó xảy ra khi ar

và br

cùng chiều 2 2

4 5

x x− − −⇔ = . Từ ñó ta ñược phương trình có một nghiệm là

2

9x = .

Ví dụ 5. Giải phương trình 2 2 4 21 2 1 2 2( 1) (2 4 1)x x x x x x x+ − + − − = − − + .

HD: ðặt 2 22 1 ( 1)y x x x= − = − − , suy ra 2 2

0 1

( 1) 1

y

x y

≤ ≤

− = −.

Page 19: Kỷ yếu hội thảo các trường chuyên dh db bắc bộ lần III

=========================================================== 21

HỘI CÁC TRƯỜNG THTP CHUYÊN KHU VỰC DUYÊN HẢI VÀ ðỒNG BẰNG BẮC BỘ

Hội thảo khoa học môn Toán học lần thứ III - 2010

Ta ñược 2 2 21 1 2(1 ) (1 2 )(1)y y y y+ + − = − − .

Mặt khác 2 21 1 1 1 2 (2)y y y y+ + − ≥ + − ≥ − .

Từ (1) và (2), suy ra 2 2 2 22(1 ) (1 2 ) 2y y y− − ≥ −

ðặt 2y z= , ta ñược 0 1z≤ ≤ và 2 22(1 ) (1 2 ) 2 (4 10 7) 0z z z z z z− − ≥ − ⇔ − + ≤

0z⇔ ≤ (do 24 10 7 0z z− + > ).

Do ñó 0z = , suy ra 0y = hay 22 0x x− =0

2

x

x

=⇔ =

.

Vậy phương trình có nghiệm là 0x = và 2x = . §2. MỘT SỐ BÀI TOÁN THI LẬP ðỘI TUYỂN HỌC SINH GIỎI TỈNH

BẮC GIANG Chọn ñội tuyển của tỉnh Bắc Giang thi học sinh giỏi quốc gia cũng có những bài toán

giải phương trình vô tỷ. Sau ñây là một số bài. Bài 1 (Lập ñội tuyển HSG quốc gia tỉnh Bắc Giang năm học 2004 – 2005)

Giải phương trình 3 23 32 11 4 4 14 5 13 2x x x x x x− − − + − + = + − . Bài 2 (Kiểm tra ñội tuyển HSG quốc gia tỉnh Bắc Giang năm học 2004 – 2005)

Giải phương trình 3 2 3 3 3 22 2 3 1 2 3 1x x x x x x+ − − + = − − − . Bài 3 (Lập tiền ñội tuyển HSG quốc gia tỉnh Bắc Giang năm học 2006 – 2007)

Giải phương trình 4 8 4 2 3 3x x x x+ + + = + + . Bài 4 (Dự tuyển toán QG gửi Bộ GD-ðT của Bắc Giang năm học 2006 – 2007)

Giải phương trình 2 2 22 3 2 1 3 3x x x x x x− + = − + + − .

Bài 5. (Kiểm tra ñội tuyển HSG quốc gia tỉnh Bắc Giang năm học 2007 – 2008)

Giải phương trình 2

2

2007 2008 2009

2007

x x x

x x

− +=

+.

Bài 6. (Giáo sư dạy ñội tuyển toán tỉnh Bắc Giang năm học 2004 – 2005)

Giải các phương trình sau:

1) 21 3 2 1x x x x+ + − = + . 4) 2 1 58

2x

x+ = .

2) 3 47 80x x x+ + = + . 5) 4 32

8x x= + .

3) 33 1 2(2 1)x x+ = − . 6) 2 32 4 3 4x x x x+ + = + .

Page 20: Kỷ yếu hội thảo các trường chuyên dh db bắc bộ lần III

=========================================================== 22

HỘI CÁC TRƯỜNG THTP CHUYÊN KHU VỰC DUYÊN HẢI VÀ ðỒNG BẰNG BẮC BỘ

Hội thảo khoa học môn Toán học lần thứ III - 2010

§3. MỘT SỐ BÀI TOÁN THI HỌC SINH GIỎI CỦA MỘT SỐ QUỐC GIA Thực tế bài toán giải phương trình vô tỷ trong kỳ thi học sinh giỏi quốc gia là không

khó. Tuy nhiên ñể làm ñược việc lớn thì trước hết phải làm tốt việc nhỏ, do ñó học sinh muốn ñoạt giải từ khuyến khích trở lên phải làm tốt bài toán này. Dù biết vậy nhưng không phải học sinh xuất sắc nào cũng vượt qua ñược.

Bài 1 (1995 - Bảng A. VMO)

Giải phương trình 3 2 43 8 40 8 4 4 0x x x x− − + − + = . HD: ðk 1x ≥ − .

Khi ñó xét 3 2( ) 3 8 40f x x x x= − − + và 4( ) 8 4 4g x x= + trên ñoạn [ )1;− +∞ .

Ta ñược ( ) ( )f x g x= . Áp dụng BðT Cô-si cho bốn số không âm, ta ñược

4 4 4 4 4 44 1( ) 2 .2 .2 (4 4) (2 2 2 (4 4)) 13(1)

4g x x x x= + ≤ + + + + = + . ðẳng thức xảy ra khi và

chỉ khi 44 4 2 3x x+ = ⇔ = . Mặt khác 3 2 23 8 40 13 ( 3)( 9) 0x x x x x x− − + ≥ + ⇔ − − ≥

2( 3) ( 3) 0(2)x x⇔ − + ≥ . ðẳng thức xảy ra khi và chỉ khi 3x = . Từ (1) và (2), ta ñược ( ) 13 ( )g x x f x≤ + ≤ . Cả hai ñẳng thức ñều xảy ra khi 3x = , thỏa

mãn ñiều kiện. Vậy phương trình có nghiệm duy nhất là 3x = .

Nhận xét: Ta có thể sử dụng ñạo hàm ñể xét sự biến thiên của các hàm số ( )f x và

( )g x trên ñoạn [ )1;− +∞ , ta ñược [ )1:min ( ) (3) 13f x f− +∞

= = và [ )1:max ( ) (3) 13g x g− +∞

= = .

Hoặc ta có thể ñặt 4 4 4x y+ = , với 0y ≥ sau ñó dùng ñạo hàm ñể khảo sát sự biến

thiên của hàm số 12 8 4( ) 24 16 512 2816f y y y y y= − + − + ( '( ) 2( 2). ( )f y y h y= − với ( ) 0h y > ).

Bài 2 (1995 - Bảng B. VMO)

Giải phương trình 2 32 11 21 3 4 4 0x x x− + − − = .

HD: ðặt 3 4 4x y− = .

Khi ñó 3 4

4

yx

+= và suy ra

6 32 8 16

6

y yx

+ += . Từ ñó ta có phương trình

6 3 3 6 31 11( 8 16) ( 4) 3 21 0 14 24 96 0(1)

8 4y y y y y y y+ + − + − + = ⇔ − − + =

2 4 3 2( 2) ( 4 12 18 14) 0(2)y y y y y⇔ − + + + + = .

Do 0y ≤ thì Vt(1) dương, do ñó ta xét 0y > , khi ñó 4 3 24 12 18 14 0y y y y+ + + + > .

Nên từ (2) ta thấy 2y = hay 3 4 4 2x − = , ta ñược 3x = .Thử lại ñúng. Vậy phương trình có nghiệm duy nhất là 3x = . Bài 3 (2002 - Bảng A. VMO)

Giải phương trình 4 3 10 3 2x x− − = − . HD: Cách 1 (ðáp án)

Page 21: Kỷ yếu hội thảo các trường chuyên dh db bắc bộ lần III

=========================================================== 23

HỘI CÁC TRƯỜNG THTP CHUYÊN KHU VỰC DUYÊN HẢI VÀ ðỒNG BẰNG BẮC BỘ

Hội thảo khoa học môn Toán học lần thứ III - 2010

ðk 74 10

27 3x≤ ≤ . Với ñiều kiện ñó phương trình ñã cho tương ñương với phương trình

2 2 24 3 10 3 4 4 9(10 3 ) (4 )x x x x x x− − = − + ⇔ − = −

4 3 2

2

8 16 27 29 0

( 3)( 2)( 7 15) 0

x x x x

x x x x

⇔ − + + − =

⇔ − + − + =

3x⇔ = (do ñk và 2 7 15 0x x− + > với mọi x thỏa mãn ñk) Vậy phương trình có nghiệm duy nhất là 3x = .

Cách 2: ðặt 10 3x y− = , suy ra 4

03

y≤ ≤ (1) và 2 210 4

2 03 3

y yx x

− −= ⇒ − = >

với mọi y thỏa mãn (1).

Khi ñó ta ñược 2 4 24 8 16

4 3 4 33 9

y y yy y

− − +− = ⇔ − =

4 3

2

8 27 20 0

( 1)( 4)( 3 5) 0

y y y

y y y x

⇔ − + − =

⇔ − + − + =

1y⇔ = .

Hay ta ñược 10 3 1x− = 3x⇔ = . Vậy phương trình có nghiệm duy nhất là 3x = . Bài 4 (1998-CMO)

Giải phương trình 1 1

1x xx x

= − + − .

Nhận xét: ðây là bài toán thi học sinh giỏi của Canada, có thể nói là ñơn giản, nhẹ nhàng với học sinh tinh ý nhưng cũng ñầy cạm bẫy với mọi học sinh.

Thật vậy, từ ñk xác ñịnh của phương trình ta phải dẫn ñến ñược 1x > .

Với ñk ñó, phương trình tương ñương với 1 1

1x xx x

− − = −

2 2

1 11x x

x x

⇔ − − = −

(do hai vế không âm với mọi

1x > )

2 2( 1) 2 ( 1) 0x x x x⇔ − − − + =

2 2( 1 ) 0x x⇔ − − =

2 1 0x x⇔ − − = . Từ ñó suy ra 1 5

2x

+= .

Cũng có thể từ 2 2( 1) 2 ( 1) 0x x x x− − − + = , chuyển 22 ( 1)x x − sang vế phải rồi bình

phương hai vế, sau ñó ñặt 1

2x y− = ta ñược phương trình trùng phương ẩn

1

2y > , giải

phương trình này tìm ñược 5

2y = . Từ ñó suy ra

1 5

2x

+= nhưng cách này hơi dài.

Vậy phương trình có nghiệm duy nhất là 1 5

2x

+= .

Page 22: Kỷ yếu hội thảo các trường chuyên dh db bắc bộ lần III

=========================================================== 24

HỘI CÁC TRƯỜNG THTP CHUYÊN KHU VỰC DUYÊN HẢI VÀ ðỒNG BẰNG BẮC BỘ

Hội thảo khoa học môn Toán học lần thứ III - 2010

§4. MỘT SỐ BÀI TẬP TỰ LÀM Sau ñây là một số bài tập tự làm mà chúng ta có thể sử dụng các phương pháp ở trên. Bài 1. Giải các phương trình sau:

1) 2 2 21 1 2x x x x x x+ − + − + = − + .

2) 2 21 1 (1 2 1 )x x x+ − = + − .

3) 2

2

1 2

1

x x x

x x

− +=

+.

4) 22 4 2 5 1x x x x− + − = − − .

5) 3 2 3 2 3 33 2001 3 7 2002 6 2003 2002x x x x x− + − − + − − = . Bài 2. Giải các phương trình sau:

1) 2 2 22 3 2 1 3 3x x x x x x− + = − + + − .

2) 42 60

65 7x x

+ =− −

.

3) ( 2) 1 2 2 0x x x− − − + = .

4) 3 3 3 33 1 5 2 9 4 3 0x x x x+ + − + − − − = .

5) 2 24 4 10 8 6 10x x x x− − = − − . Bài 3. Giải các phương trình sau:

1) 2(2004 )(1 1 )x x x= + − − .

2) 3 3x x x− = + .

3) 5 5x x x x− − − − = .

4) 4 3 316 5 6 4x x x+ = + .

5) 3 2 33 2 ( 2) 6 0x x x x− + + − = .

Bài 4. Giải các phương trình sau: 1) 235 1 9 2 3 1x x x x− + − = + − .

2) 2428 27

2. 27 24 1 63 2

x x x+ + = + + .

3) 13 1 9 1 16x x x− + + = .

4) 3 386 5 1x x+ − − = .

5) 3 2 32 ( 4) 7 3 28 0x x x x x− − − − − + = . Bài 5. Giải các phương trình sau:

1) 2 2

22 2 2 2

x x

x x

+ −+ =

+ + − −.

2) 22 2 4 4 2 9 16x x x+ + − = + .

Page 23: Kỷ yếu hội thảo các trường chuyên dh db bắc bộ lần III

=========================================================== 25

HỘI CÁC TRƯỜNG THTP CHUYÊN KHU VỰC DUYÊN HẢI VÀ ðỒNG BẰNG BẮC BỘ

Hội thảo khoa học môn Toán học lần thứ III - 2010

3) 2 32 5 2 4 2( 21 20)x x x x− + = − − .

4) 3 3 2x x x− = + .

5) 2

4 3 2 3 12 2 2 1 ( )

xx x x x x x

x

−+ + − + = + .

Bài 6. Giải các phương trình sau:

1) 3 3 36 6 6x x− + + = .

2) 4 1 5

2x x xx x x

+ − = + − .

3) 2 4 3 22 4 7 4 3 2 7x x x x x x+ + = + + − − .

4) 2 24 61 1 1 1x x x x− + + − + − = .

5) 2

2 21

3x x

− = −

.

Bài 7. Giải các phương trình sau:

1) ( ) ( )2 23 2 1 1 1 3 8 2 1x x x x+ − = + + + .

2) 2 32( 2) 5 1x x+ = + .

3) 6 4 2 264 112 56 7 2 1x x x x− + − = − .

4) ( )2 3 3 21 1 (1 ) (1 ) 2 1x x x x+ − + − − = + − .

5) ( )2

2 3 3 2 11 1 (1 ) (1 )

33

xx x x

−+ − + − − = + .

Bài 8. Giải các phương trình sau: 1) 3 36 6 4 4 0x x− + − = .

2) 2 32( 3 2) 3 8x x x− + = + .

3) 6 23 31 1 1x x x+ − − = − .

4) 2 2315 3 8 2x x x+ = + + − .

5) 2 3 3 244 4 4(1 ) (1 ) 1 (1 )x x x x x x x x+ − + − = − + + − .

Bài 9. Giải các phương trình sau: 1) 3 31 3 3 1x x+ = − .

2) 2

35

121

xx

x+ =

−.

3) 2 32 11 21 3 4 4 0x x x− + − − = .

4) 4 3 2 24 6 4 2 10 2x x x x x x+ + + + + + = .

5) 2 2 22 2 2

321 1 4 4

(2 3)x x x x x

x x+ + − − + − + =

+.

Bài 10. Giải các phương trình sau:

Page 24: Kỷ yếu hội thảo các trường chuyên dh db bắc bộ lần III

=========================================================== 26

HỘI CÁC TRƯỜNG THTP CHUYÊN KHU VỰC DUYÊN HẢI VÀ ðỒNG BẰNG BẮC BỘ

Hội thảo khoa học môn Toán học lần thứ III - 2010

1) 2

31

1

xx

x+ =

+.

2) ( 1) 1 5 1 4 4 0x x x x− − + − + − = .

3) 4 2 2 210 14 19 (5 38) 2x x x x− + = − − .

4) 2 2( 1) 2 3 1x x x x+ − + = + .

5) 2 211 1 2

2x x x− − = − .

Bài 11. Giải các phương trình sau:

1) 1 3

1 04 2

x

x x

+− =

+ +.

2) 3 3 2 0x x x− − + = .

3) 3 38 4 6 1 1 0x x x− − + − = .

4) ( )2 2 23 2 2 2 1 0x x x x+ − + − + − = .

5) 2 23 5 12 5 0x x x+ + − + − = . Bài 12. Giải các phương trình sau:

1) 2 32( 8) 5 8x x+ = + .

2) 24 3 4 3 10 3x x x− = − − .

3) ( 3) (4 )(12 ) 28x x x x+ − + = − .

4) 2 2 2 32 1 6 9 6 ( 1)(9 ) 38 10 2x x x x x x x+ + − + + − = + − − .

5) 2 2 27 22 28 7 8 13 31 14 4 3 3( 2)x x x x x x x− + + + + + + + = + . Bài 13. Giải các phương trình sau:

1) 4 2 2 2 2 2 22

14 16 9 2 2x y x y x y y x

x

− + + − − = +

.

2) 2 2 2 2 3 21 1 1 12 ... 2 3 3 1

4 4 4 4x x x x x x x x− + − + + − + + + = + + + .

Trong ñó biểu thức vế trái có tất cả 2008 dấu căn thức bậc hai.

Page 25: Kỷ yếu hội thảo các trường chuyên dh db bắc bộ lần III

=========================================================== 27

HỘI CÁC TRƯỜNG THTP CHUYÊN KHU VỰC DUYÊN HẢI VÀ ðỒNG BẰNG BẮC BỘ

Hội thảo khoa học môn Toán học lần thứ III - 2010

LÀM NGƯỢC BẤT ðẲNG THỨC Nguyễn ðức Vang (THPT chuyên Bắc Ninh)

Trong báo toán số 377(tháng 11 năm 2008) có bài toán sau:

“Tìm số thực k nhỏ nhất sao cho với mọi bộ số thực không âm x, y, z ta luôn có:

{ }xzzyyxMaxkxyzzyx

−−−+≤++

,,.3

3 ”.

Bắt chước cách làm ấy, tôi khai thác một số bất ñẳng thức quen biết, bằng cách thêm vào

vế bé một lượng ñồng bậc tối thiểu ñể làm thay ñổi sự chênh lệch.

Bài 1. Tìm số thực k nhỏ nhất sao cho bất ñẳng thức sau ñúng với mọi x, y không

âm:

2222 .2 yxkxyyx −+≤+ .

Bài 2. Tìm số thực k nhỏ nhất sao cho bất ñẳng thức sau ñúng với mọi x, y không

âm:

yxkyxyx −++≤+ .)(2 22 .

Bài 3. Tìm số thực k nhỏ nhất sao cho bất ñẳng thức sau ñúng với mọi x, y không

âm:

{ }xzzyyxMaxkzyxzyx −−−+++≤++ ,,.)(3 222 .

Bài 4. Tìm số thực k nhỏ nhất sao cho bất ñẳng thức sau ñúng với mọi x, y:

44444 yx.k)2

yx(2yx −+

+≤+

Bài 5. Tìm số thực k nhỏ nhất sao cho bất ñẳng thức sau ñúng với mọi x, y không

âm:

nnnnnyxk

yxyx −+

+≤+ .)

2(2 (với n là số nguyên

dương)

Bài 6. Tìm số thực k nhỏ nhất sao cho bất ñẳng thức sau ñúng với mọi x, y, z:

{ }2222222222 ,,.max)()(3 xzzyyxkzyxzyx −−−+++≤++

Bài 7. Tìm số thực k nhỏ nhất sao cho bất ñẳng thức sau ñúng với mọi x, y, z:

{ }22221

222

21 .max)...()...( jinn xxkxxxxxxn −++++≤+++

Bài 8. Tìm số thực k nhỏ nhất sao cho bất ñẳng thức sau ñúng với mọi x, y không

âm:

qkn

n

n

k

k xxMaxkxxnx −+≤∑=

....11

.

Bài 9. Tìm số thực k nhỏ nhất sao cho bất ñẳng thức sau ñúng với mọi x, y

2;0π

Page 26: Kỷ yếu hội thảo các trường chuyên dh db bắc bộ lần III

=========================================================== 28

HỘI CÁC TRƯỜNG THTP CHUYÊN KHU VỰC DUYÊN HẢI VÀ ðỒNG BẰNG BẮC BỘ

Hội thảo khoa học môn Toán học lần thứ III - 2010

)(sin.coscos2

cos 2yxkyx

yx−++≤

+

Bài 10. Tìm số thực k nhỏ nhất sao cho bất ñẳng thức sau ñúng với mọi a, b không

âm:

22.)2

(2)()( bakba

fbfaf −++

≤+

trong ñó f(x) = x2 + 2x +3.

HƯỚNG DẪN GIẢI

Bài 1.

+) Giả sử bất ñẳng thức 2222 .2 yxkxyyx −+≤+ (*)

ñúng với mọi x, y không âm.

Cho x = 0, y = 1 suy ra 1k ≥ .

+) Ta chứng minh 2 2 2 22 . , , : 0x y x y x y x y x y+ ≤ + − ∀ ≥ ≥ .

Thật vậy, BðT trên tương ñương với y.xy 2 ≤

BðT này ñúng vì 0yx ≥≥ .

Vậy số thực k nhỏ nhất cần tìm là .1k 0 =

Bài 2.

+) Giả sử bất ñẳng thức yxkyxyx −++≤+ .)(2 22 (*)

ñúng với mọi x, y không âm.

Cho x = 0, y = 1 suy ra 12k −≥ .

+) Ta chứng minh 2 22( ) . ( 2 1)( ), , : 0x y x y x y x y x y+ ≤ + + − − ∀ ≥ ≥ .

Thật vậy, BðT trên tương ñương với

xyyy).22(x2)yx(2 222 ≤⇔−+≤+

BðT này ñúng vì 0yx ≥≥ .

Vậy số thực k nhỏ nhất cần tìm là .12k 0 −=

Bài 3.

+) Giả sử bất ñẳng thức { }xzzyyxMaxkzyxzyx −−−+++≤++ ,,.)(3 222

(*)

ñúng với mọi x, y không âm.

Cho x = 1, y = z = 0 suy ra 13k −≥ .

+) Ta chứng minh 2 2 23( ) ( 3 1)( ); , , : 0x y z x y z x z x y z x y z+ + ≤ + + + − − ∀ ≥ ≥ ≥ .

Thật vậy, BðT trên tương ñương với

zx)332(yz)32(y.x3z)13(2y 22 −+−+≤−+

Page 27: Kỷ yếu hội thảo các trường chuyên dh db bắc bộ lần III

=========================================================== 29

HỘI CÁC TRƯỜNG THTP CHUYÊN KHU VỰC DUYÊN HẢI VÀ ðỒNG BẰNG BẮC BỘ

Hội thảo khoa học môn Toán học lần thứ III - 2010

BðT này ñúng vì

−≥−

−≥−

−+≥

2

2

22

z)332(zx)332(

z)32(yz)32(

z)13(yxy3

.

Vậy số thực k nhỏ nhất cần tìm là .13k 0 −=

Cách 2:

ðặt f(x;y;z) = 0zyx );zx)(13(zyx)zyx(3 222 ≥≥≥−−−−−−++ .

Dùng ñạo hàm, chỉ ra ñược .0)z;z;z(f);z;y;y(f)z;y;x(f =≤≤

Bài 4.

+) Giả sử bất ñẳng thức qknn xxkMaxxxxxn −+++≤++ ....)...( 122

1 (*)

ñúng với khi .0x...xx n21 ≥≥≥≥

Cho 1nk0x...x,1x n21 −≥⇒=== .

+) BðT )xx)(1n(x...x)x...x(n n1n12

n2

1 −−+++≤++ , với

.0x...xx n21 ≥≥≥≥

chứng minh ñược bằng cách dồn biến như cách 2 của bài 3.

Vậy số thực k nhỏ nhất cần tìm là .1nk 0 −=

Bài5.

+) Giả sử bất ñẳng thức 44444 yx.k)2

yx(2yx −+

+≤+

ñúng với mọi x, y không âm.

Cho x = 0, y = 1 suy ra 8

7k ≥ .

+) Dùng ñạo hàm, ta chứng minh ñược: )yx.(8

7)

2

yx(2yx 44444 −+

+≤+ , với

0yx ≥≥

Vậy số thực k nhỏ nhất cần tìm là .8

7k 0 =

Bài6.

+) Giả sử bất ñẳng thức

)xz,zy,yx(Max.k)zyx()zyx(3 2222222222 −−−+++≤++

ñúng với x, y, z không âm.

Cho x = 1, y = z = 0 suy ra 2≥k .

+) Dùng ñạo hàm, ta chứng minh ñược )zx(2)zyx()zyx(3 222222 −+++≤++

với 0≥≥≥ zyx

Vậy số thực k nhỏ nhất cần tìm là .2k 0 =

Bài 7

Page 28: Kỷ yếu hội thảo các trường chuyên dh db bắc bộ lần III

=========================================================== 30

HỘI CÁC TRƯỜNG THTP CHUYÊN KHU VỰC DUYÊN HẢI VÀ ðỒNG BẰNG BẮC BỘ

Hội thảo khoa học môn Toán học lần thứ III - 2010

+) Cho x1 = 1, x2 = …= xn = 0 1k −≥⇒ n

+) ðặt f(x1;x2;…;xn) = n(x12 + …+x2

n) – (x1+…+xn)2 – (n - 1)(x2

1 – xn2),

với 0.... 21 ≥≥≥≥ nxxx . Khi

ñó 0)x;...;f(x)x;...;x;x;f(x)x;...;x;f(x nnn322n21 ≤≤≤ .

Vậy số thực k nhỏ nhất cần tìm là .10 −= nk

Bài 8

+) Giả sử bất ñẳng thức qkn

n

n

k

k xxMaxkxxnx −+≤∑=

....11

ñúng khi x1, …, xn không âm.

Cho x1 = x2 = …= xn-1 =1, xn = 0 suy ra 1nk −≥ .

+) Ta chứng minh )xx)(1n(x...xnx n1n

n1

n

1kk −−+≤∑

=

với 0x...xx n21 ≥≥≥≥

Thật vậy, BðT trên tương ñương với

1n

n1n

1n

2kk x)2n(x...xnx.nx −+≤+∑

=

BðT ñúng vì

−≤++ −

nn21n

11n2

x...x.xnnx

x)2n(x...x

Vậy số thực k nhỏ nhất cần tìm là .1nk 0 −=

Bài 9

Với 02

≥≥≥ yxπ

, ta có:

2

22 2 2 2

2

2cos (cos cos ) 4cos .sin cos2 2 4 2sin ( ) 4sin .cos 4cos .cos

2 2 4 2

1

4cos .c8

x y x y x y x yx y

x y x y x y x yx y

π

+ + − +− +

= = ≤− − − −−

≤2 2

3 2 2os4π

=+

1nk −≥

Bài 10

+)Với a > b > 0 ta có 2

1

)(2

)2

(2)()(

22→

+

−=

+−+

ba

ba

ba

bafbfaf

+) Dễ dàng chứng minh bất ñẳng thức 22.2

1)

2(2)()( ba

bafbfaf −+

+≤+ với a, b

không âm.

Vậy giá trị nhỏ nhất của k là: 2

1=k

Page 29: Kỷ yếu hội thảo các trường chuyên dh db bắc bộ lần III

=========================================================== 31

HỘI CÁC TRƯỜNG THTP CHUYÊN KHU VỰC DUYÊN HẢI VÀ ðỒNG BẰNG BẮC BỘ

Hội thảo khoa học môn Toán học lần thứ III - 2010

CHỨNG MINH BẤT ðẲNG THỨC BẰNG CÁCH SỬ DỤNG BẤT ðẲNG THỨC SẮP XẾP LẠI VÀ BẤT ðẲNG THỨC CHEBYSHEV

ðào Quốc Huy - Tổ Toán – Tin

Trường THPT Chuyên Biên Hòa – Hà Nam

Bất ñẳng thức là một chuyên ñề quan trọng trong chương trình bồi dưỡng học sinh giỏi Quốc gia. Trong các phương pháp chứng minh bất ñẳng thức thì phương pháp áp dụng bất ñẳng thức cổ ñiển thường xuyên ñược sử dụng, ñã có rất nhiều bài toán chứng minh bất ñẳng thức mà lời giải ñề cập ñến việc sử dụng bất ñẳng thức liên hệ giữa Trung bình cộng - Trung bình nhân (AM-GM), bất ñẳng thức Cauchy – Schwarz, bất ñẳng thức Holder, bất ñẳng thức Schur …. Trong khuôn khổ bài viết, tôi xin ñề cập ñến bất ñẳng thức Sắp xếp lại và một số bài tập sử dụng bất ñẳng thức này. Bên cạnh ñó, bài viết cũng ñề cập ñến một phương pháp sử dụng bất ñẳng thức Chebyshev (coi như hệ quả của bất ñẳng thức Sắp xếp lại) ñể ñánh giá một số bất ñẳng thức 3 biến dạng phân thức.

I. Bất ñẳng thức Sắp xếp lại:

Giả sử 1 2 ... na a a≤ ≤ ≤ và 1 2 ... nb b b≤ ≤ ≤ ( )*n∈� là hai dãy các số thực. Ta ñặt

1 1 2 2

1 2 1 1

...

...n n

n n n

A a b a b a b

B a b a b a b−

= + + +

= + + +

Gọi 1 2( , ,..., )nx x x là một hoán vị của 1 2( , ,..., )nb b b , ñặt 1 1 2 2 ... n nX a x a x a x= + + +

Khi ñó ta có bất ñẳng thức sau A X B≥ ≥

Dấu ñẳng thức xảy ra khi và chỉ khi các ia tất cả bằng nhau hoặc các ib tất cả bằng

nhau.

Chứng minh:

Trước hết ta chứng minh A X≥ bằng phương pháp qui nạp:

- Với 1n = , kết quả là hiển nhiên.

- Giả sử bất ñẳng thức ñúng cho n k= , với 1n k= + ta ñặt 1k ib x+ = và 1k jx b+ = .

Từ bất ñẳng thức ( )( )1 1 0k i k j

a a b b+ +− − ≥ ta thu ñược 1 1 1 1i j k k k j i ka b a b a b a b+ + + ++ ≥ + ,

như vậy trong X ta có thể thay ñổi ix và 1kx + ñể thu ñược tổng lớn hơn. Sau khi ñổi ta áp

dụng giả thiết qui nạp cho k thành phần ñầu tiên của tổng X và suy ra A X≥ .

Bất ñẳng thức X B≥ ñược suy ra từ A X≥ bằng cách xét dãy 1 1...n nb b b−− ≤ − ≤ ≤ −

thay cho dãy 1 2 ... nb b b≤ ≤ ≤ .

Với kí hiệu như trên, một cách ngắn gọn ta coi A là tổng các chỉ số “cùng chiều”, B là tổng các chỉ số “ñảo chiều”, còn X là tổng các chỉ số “tùy ý”. Bất ñẳng thức Sắp xếp lại cho ta: tổng cùng chiều ≥ tổng tùy ý ≥ tổng ñảo chiều.

Page 30: Kỷ yếu hội thảo các trường chuyên dh db bắc bộ lần III

=========================================================== 32

HỘI CÁC TRƯỜNG THTP CHUYÊN KHU VỰC DUYÊN HẢI VÀ ðỒNG BẰNG BẮC BỘ

Hội thảo khoa học môn Toán học lần thứ III - 2010

Việc áp dụng bất ñẳng thức Sắp xếp lại quan trọng nhất ở chỗ biến ñổi bất ñẳng thức cần chứng minh về dạng có các vế là tổng của tích các phần tử tương ứng của 2 dãy mà thứ tự của chúng liên quan với nhau (cùng thứ tự hoặc ngược thứ tự). Chẳng hạn hai dãy

{ }, ,a b c và { }3 3 3, ,a b c có cùng thứ tự, còn với , , 0x y z > thì hai dãy { }, ,x y z và

1 1 1, ,

x y z x y z

+ + + ngược thứ tự.

II. Sử dụng bất ñẳng thức Sắp xếp lại:

Mặc dù bất ñẳng thức Sắp xếp lại ñược phát biểu cho các số thực nhưng trong các bài tập dưới ñây giả thiết thường cho ñiều kiện các số dương hoặc không âm, ñiều này nhằm mục ñích sắp xếp 2 dãy cùng chiều của giả thiết ñược thỏa ñáng. Bên cạnh ñó, nếu không có gì ñặc biệt tác giả xin không trình bày trường hợp xảy ra dấu ñẳng thức ( bởi vì nó hoàn toàn như phát biểu ở trên, ñẳng thức xảy ra khi 1 trong 2 dãy là dãy dừng ), ñồng thời tác giả xin

ñược sử dụng kí hiệu ∑ thay thế cho cyc

∑ trong các bài toán chứng minh bất ñẳng thức

quay vòng của 3 biến. Ngoài cách áp dụng bất ñẳng thức Sắp xếp lại, có nhiều bài toán trong số những bài dưới ñây hoàn toàn có thể giải bằng những phương pháp khác, và bên cạnh sử dụng bất ñẳng thức Sắp xếp lại ta còn áp dụng một số bất ñẳng thức cổ ñiển khác.

Bài toán 1:

Cho n số thực ( , 2)n n∈ ≥� : 1 2, ,..., na a a thỏa mãn 1 2

1 2

... 0

... 1n

n

a a a

a a a

+ + + =

+ + + =

Chứng minh rằng 1 2

12 ...

2n

na a na

−+ + + ≤

Bài giải:

Kí hiệu 1 2, ,..., tj j j và 1 2, ,..., ks s s là các phần tử { }1, 2,..., n∈ sao cho

1 2 1 2... 0 ...

t kj j j s s sa a a a a a≥ ≥ ≥ ≥ ≥ ≥ ≥ ≥

Từ giả thiết ta suy ra 1 2

1...

2tj j ja a a+ + + = và

1 2

1...

2ks s sa a a+ + + = − .

Không giảm tính tổng quát, ta có thể giả sử 1 22 ... 0na a na+ + + ≥ ( vì nếu trái lại ta

dùng phép ñặt 'i ia a= − ). Theo bất ñẳng thức Sắp xếp lại ta có:

1 1 1

1 1 1

1 21 2 ... 1 2 ... ( 1) ...

11 1 ... 1 ...

2

k k t

k k t

n s s s j j

s s s j j

a a na a a ka k a na

na a a na na

+ + + ≤ + + + + + + +

−≤ + + + + + + ≤

Bất ñẳng thức ñã cho ñược chứng minh.

Page 31: Kỷ yếu hội thảo các trường chuyên dh db bắc bộ lần III

=========================================================== 33

HỘI CÁC TRƯỜNG THTP CHUYÊN KHU VỰC DUYÊN HẢI VÀ ðỒNG BẰNG BẮC BỘ

Hội thảo khoa học môn Toán học lần thứ III - 2010

Bài toán 2:

Cho các số thực , , 0a b c ≥ thỏa mãn 1a b c+ + = .

Chứng minh rằng 2

2a b

b c

+≥

+∑

Bài giải :

Ta có 2 2 ( ) ( )

1a b a b a b c a a b

b c b c b c

+ + + + += = +

+ + +∑ ∑ ∑

bất ñẳng thức cần chứng minh trở thành ( )a a b

a b cb c

+≥ + +

+∑ (*)

Chú ý là 2 dãy { }, ,a b c và 1 1 1

, ,b c c a a b

+ + + có cùng thứ tự, theo bất ñẳng thức Sắp

xếp lại ta có 2

( ) ( )a ab ca ab

a b cb c b c b c b c

+ ≥ + = + ++ + + +

∑ ∑ . Vậy (*) ñược chứng minh.

Bài toán 3:

Cho , , 0.x y z >

Chứng minh rằng ( ) ( ) ( )2 2 2 1y x z y x z

x y z y z x z x y− − −

+ + + ≤

Bài giải:

ðặt ( ) ( ) ( )2 2 2, ,a x y z b y z x c z x y= + = + = + .

Lấy lôgarit tự nhiên 2 vế, ñưa bất ñẳng thức cần chứng minh về dạng:

ln lnx c x a≤∑ ∑ , bất ñẳng thức này ñúng theo bất ñẳng thức Sắp xếp lại với nhận xét

rằng 2 dãy { }, ,x y z và { }, ,a b c cùng thứ tự, ñồng thời hàm ln t ñồng biến trên ( )0;+∞ nên

dãy { }ln , ln , lna b c cũng có thứ tự như 2 dãy trên, ta suy ra ñiều phải chứng minh.

Bài toán 4:

Cho tam giác nhọn ABC .

Chứng minh rằng 3

1 sin sin2

A B− ≥∑

Page 32: Kỷ yếu hội thảo các trường chuyên dh db bắc bộ lần III

=========================================================== 34

HỘI CÁC TRƯỜNG THTP CHUYÊN KHU VỰC DUYÊN HẢI VÀ ðỒNG BẰNG BẮC BỘ

Hội thảo khoa học môn Toán học lần thứ III - 2010

Bài giải:

Sử dụng ñịnh lý sin, bất ñẳng thức cần chứng minh tương ñương với:

( )2

2 2

2

33 4 4 0 0

4

R abR R ab R ab R

R ab R

−≤ − ⇔ − − ≥ ⇔ ≥

− +∑ ∑ ∑ (*)

( ở ñây a,b,c,R theo thứ tự là ñộ dài các cạnh BC,CA,AB và bán kính ñường tròn ngoại tiếp tam giác ABC ).

Từ bất ñẳng thức quen thuộc 2 2 2 29a b c R+ + ≤ mà 2 23 2ab ab ab a b− = ≤ +

ta thu ñược: ( )2 23 3R ab c ab− ≥ − (1).

Ta sẽ chứng minh 2

2 24 4

c ab

R ab R R ab R≥

− + − +∑ ∑ (2).

Vì 2 dãy { }, ,a b c và 2 2 2

1 1 1, ,

4 4 4R bc R R ca R R ab R

− + − + − + có cùng thứ tự

nên theo bất ñẳng thức Sắp xếp lại ta có

2 2 2

2 2 2 2

2 2

4 4 4 4

c a b ab

R ab R R ab R R ab R R ab R≥ + ≥

− + − + − + − +∑ ∑ ∑ ∑

Vậy (2) ñược chứng minh, kết hợp với (1) ta suy ra (*) ñược chứng minh.

Bài toán 5:

Cho , , 0.a b c >

Chứng minh rằng 2 2 2

2 2 2

1 1 1

1 1 1

a b c a b c

b c a b c a

+ + ++ + ≥ + +

+ + +

Bài giải:

Ta có

( ) ( )( ) ( )( ) ( ) ( ) ( )

( )( )

2 2 2 2 2 2 2 2 2 2 2 2

2 2 2 2 2 2

2 22 2

1 1 1 2

2 1 1

1 11 1

a b a b a b ab a b ab a b ab a b

a b a b a b a b

b a ab b aa b

+ + + ≥ + + = + + + ≥ + +

+ + + + +⇒ + = ≥ = +

+ ++ +∑ ∑ ∑ ∑ ∑ ∑

Theo bất ñẳng thức Sắp xếp lại

( ) ( )

2 2 2 2 2 2

2 2 2 22 2 2 2

1

1 1 11 1

a a a a b a

b b b bb b b b

+= + ≥ + =

+ + ++ +∑ ∑ ∑ ∑ ∑ ∑ .

Từ ñó ta có:

Page 33: Kỷ yếu hội thảo các trường chuyên dh db bắc bộ lần III

=========================================================== 35

HỘI CÁC TRƯỜNG THTP CHUYÊN KHU VỰC DUYÊN HẢI VÀ ðỒNG BẰNG BẮC BỘ

Hội thảo khoa học môn Toán học lần thứ III - 2010

2 2

2

22 2 2 2

2 2 2 2

1 1 2

1 1 1 11 2 1

1 1 1 1

a a b a

b b a b

a b a a

b a b b

+ = + + +

+ + + +≥ + + + = +

+ + + +

∑ ∑ ∑ ∑

∑ ∑ ∑ ∑

hay 2

2

1

1

a a

b b

+≥

+∑ ∑ (ñpcm).

Bài toán 6:

Cho , , 0a b c ≥ thỏa mãn 1a b c+ + = .

Chứng minh rằng 2 2 2 43

9a b c abc+ + + ≥

Bài giải:

Ta cần chứng minh 2 2 2 34( )( ) 3 ( )

9a b c a b c abc a b c+ + + + + ≥ + +

Khai triển rồi ñưa bất ñẳng thức về dạng ( )35 3 3a abc ab a b+ ≥ +∑ ∑

- Theo bất ñẳng thức Schur: có ( )3 3a abc ab a b+ ≥ +∑ ∑ (1).

- Theo bất ñẳng thức Sắp xếp lại: có 3 2 2 2a a b b c c a≥ + +∑ và 3 2 2 2a ab bc ca≥ + +∑ ,

cộng từng vế ta thu ñược ( ) ( )3 32 4 2a ab a b a ab a b≥ + ⇔ ≥ +∑ ∑ ∑ ∑ (2).

Cuối cùng, cộng từng vế của (1) và (2) ta có ñpcm.

Bài toán 7:

Cho , , , 0a b c d ≥ thỏa mãn 4a b c d+ + + = .

Chứng minh rằng 2 2 2 2 4a bc b cd c da d ab+ + + ≤

Bài giải:

Giả sử ( ), , ,p q r s là hoán vị của ( ), , ,a b c d sao cho p q r s≥ ≥ ≥ .

Page 34: Kỷ yếu hội thảo các trường chuyên dh db bắc bộ lần III

=========================================================== 36

HỘI CÁC TRƯỜNG THTP CHUYÊN KHU VỰC DUYÊN HẢI VÀ ðỒNG BẰNG BẮC BỘ

Hội thảo khoa học môn Toán học lần thứ III - 2010

Khi ñó

( ) ( ) ( ) ( )

( ) ( ) ( ) ( ) ( )( )

( )( )

2 2 2 2

22 221 1

2 4 4 2

a bc b cd c da d ab a abc b bcd c cda d dab

p pqr q pqs r prs s qrs pq rs pr qs

pq rs pr qs p q r sp s q r

+ + + = + + +

≤ + + + = + +

+ + + + + + ≤ = + + ≤

(bất ñẳng thức ñầu tiên là bất ñẳng thức Sắp xếp lại, hai bất ñẳng thức sau là bất ñẳng thức AM-GM)

ðể ý là 4p q r s a b c d+ + + = + + + = , ta có ñpcm.

III.Bất ñẳng thức Chebyshev dạng mẫu số

Bất ñẳng thức Chebyshev cổ ñiển có thể coi như là hệ quả của bất ñẳng thức Sắp xếp lại (xem bài tập áp dụng 1 phần V), từ dạng cổ ñiển này người ta mở rộng bất ñẳng thức Chebyshev theo một vài hướng, sau ñây là một dạng mở rộng có nhiều ứng dụng ñể chứng minh bất ñẳng thức:

Bất ñẳng thức Chebyshev dạng mẫu số (còn gọi là dạng Engel) ñược phát biểu như sau:

a) Nếu ta có 1 2

1 2

1 2

...

...

n

n

n

aa a

x x x

x x x

≥ ≥ ≥

≥ ≥ ≥

hoặc 1 2

1 2

1 2

...

...

n

n

n

aa a

x x x

x x x

≤ ≤ ≤

≤ ≤ ≤

thì ta có

( )1 21 2

1 2 1 2

......

...nn

n n

n a a aaa a

x x x x x x

+ + ++ + + ≤

+ + +

b) Nếu ta có 1 2

1 2

1 2

...

...

n

n

n

aa a

x x x

x x x

≤ ≤ ≤

≥ ≥ ≥

hoặc 1 2

1 2

1 2

...

...

n

n

n

aa a

x x x

x x x

≥ ≥ ≥

≤ ≤ ≤

thì ta có

( )1 21 2

1 2 1 2

......

...nn

n n

n a a aaa a

x x x x x x

+ + ++ + + ≥

+ + +

(Chứng minh 2 kết quả này bằng cách sử dụng trực tiếp bất ñẳng thức Chebyshev)

Hai kết quả trên, kết hợp với việc thêm các biểu thức phù hợp, trở nên hiệu quả trong việc ñánh giá các bất ñẳng thức ñối 3 biến có chứa phân thức, mặc dù chúng chỉ là mở rộng ñơn giản từ bất ñẳng thức Chebyshev. ðể làm rõ thêm, chúng ta xét một vài ví dụ sau:

Bài toán 1: Cho , , 0a b c > thỏa mãn

1 1 11

1 1 1a b b c c a+ + ≥

+ + + + + +

Page 35: Kỷ yếu hội thảo các trường chuyên dh db bắc bộ lần III

=========================================================== 37

HỘI CÁC TRƯỜNG THTP CHUYÊN KHU VỰC DUYÊN HẢI VÀ ðỒNG BẰNG BẮC BỘ

Hội thảo khoa học môn Toán học lần thứ III - 2010

Chứng minh rằng a b c ab bc ca+ + ≥ + +

Lời giải: Ta có

1 1 1

1 1 1

a b c

a b b c c a ab ac a bc ba b ca cb c+ + = + +

+ + + + + + + + + + + +

Không giảm tính tổng quát, giả sử c b a≤ ≤ , thế thì c ca ca b bc ba a ab ac+ + ≤ + + ≤ + +

Lại có a b c

ab ac a bc ba b ca cb c≥ ≥

+ + + + + + luôn ñúng (vì bất ñẳng thức này

a b c⇔ ≥ ≥ )

Do ñó theo a) thì

( )

( )

1 1 1

1 1 13

2

a b c

a b b c c a ab ac a bc ba b ca cb c

a b c

a b c ab bc ca

+ + = + ++ + + + + + + + + + + +

+ +≤

+ + + + +

Kết hợp với giả thiết ta suy ra

( )( )

31

2

a b ca b c ab bc ca

a b c ab bc ca

+ +≥ ⇔ + + ≥ + +

+ + + + +

Ta có ñiều phải chứng minh, ñẳng thức xảy ra khi 1a b c= = = .

Bài toán 2: Cho , , 0a b c > thỏa mãn 3a b c+ + = .

Chứng minh rằng

4 9 4 9 4 913

ab bc ca

ab a b bc b c ca c a

+ + ++ + ≥

+ + + + + +

Lời giải:

Không mất tính tổng quát, giả sử a b c≥ ≥ , khi ñó từ bất ñẳng thức luôn ñúng

( )( )1 0a b c+ − ≥ ta suy ra ab a b ac a c+ + ≥ + + . Tương tự ta thu ñược

ab a b ac a c bc b c+ + ≥ + + ≥ + +

Cũng từ bất ñẳng thức luôn ñúng ( )( )( )3 3 0b c a a− − + ≥ ta thu ñược

4 9 4 9ab ca

ab a b ca c a

+ +≤

+ + + +, tương tự ta có

4 9 4 9 4 9ab ca bc

ab a b ca c a bc b c

+ + +≤ ≤

+ + + + + +

Page 36: Kỷ yếu hội thảo các trường chuyên dh db bắc bộ lần III

=========================================================== 38

HỘI CÁC TRƯỜNG THTP CHUYÊN KHU VỰC DUYÊN HẢI VÀ ðỒNG BẰNG BẮC BỘ

Hội thảo khoa học môn Toán học lần thứ III - 2010

Do ñó theo b) VT của bất ñẳng thức cần chứng minh

( )( )

( )12 81 12 81

2 6

ab bc ca ab bc ca

ab bc ca a b c ab bc ca

+ + + + + +≥ =

+ + + + + + + +

Ta chỉ cần chứng minh ( )3 4 27

13 36

abab

ab

+≥ ⇔ ≤

+

∑∑

bất ñẳng thức cuối cùng này ñúng do ( )

2

33

a b cab bc ca

+ ++ + ≤ = .

Bài toán 3: Cho tam giác nhọn ABC .

Chứng minh rằng

1 1 1 3

1 tan tan 1 tan tan 1 tan tan 1 2 3A B B C C A+ + ≤

+ + + + + + +

Lời giải: Tương tự Bài toán 1 ta có bất ñẳng thức

( )( )

1 1 1

1 tan tan 1 tan tan 1 tan tan3 tan tan tan

tan tan tan 2 tan tan tan tan tan tan

A B B C C A

A B C

A B C A B B C C A

+ ++ + + + + +

+ +≤

+ + + + +

Từ bất ñẳng thức ( )3ab bc ca abc a b c+ + ≥ + + và ñẳng thức tan tanA A=∑ ∏

( )( )

( ) ( )

3 tan tan tan

tan tan tan 2 tan tan tan tan tan tan

3 32 tan tan tan tan tan tan 3 tan tan1 1 2tan tan tan tan

3 3

1 2 33 tan1 2

tan

A B C

A B C A B B C C A

A B B C C A A A

A B CA

A

A

+ +

+ + + + +

= ≤+ +

+++ +

= =+

+

∑∏∑

∏∑

Ta có ñiều phải chứng minh.

Bài toán 4: Cho , , 0a b c > thỏa mãn 2 2 2 1a b c+ + = .

Chứng minh rằng

( )9

1 1 1 2

a b b c c a

ab bc ca a b c

+ + ++ + ≤

+ + + + +

Page 37: Kỷ yếu hội thảo các trường chuyên dh db bắc bộ lần III

=========================================================== 39

HỘI CÁC TRƯỜNG THTP CHUYÊN KHU VỰC DUYÊN HẢI VÀ ðỒNG BẰNG BẮC BỘ

Hội thảo khoa học môn Toán học lần thứ III - 2010

Lời giải: Bất ñẳng thức cần chứng minh tương ñương với

( )( ) 2 2 2

2 2 2

2 3 2 23 0 0

1

a b a b c a b c ab ac bc

a b c ab ab

+ + + + + − − −− ≥ ⇔ ≥

+ + + + ∑ ∑

Không giảm tính tổng quát, giả sử 1 1 1a b c ab ac bc≥ ≥ ⇒ + ≥ + ≥ +

Ta cần tiếp tục kiểm tra bất ñẳng thức

( ) ( ) ( ) ( ) ( ) ( )( )( )

( ) ( ) ( ) ( )( )( ) ( )( )

2 2 2 2 2 2

2 2 3 3 3 2 2 2

3 2 2 2 2

2 2 3 2 2 2 2 2

2 2

3 2 2 3 2 2

1 1

2 3 2 0

2 2 3 3 3 2 2 0

2 1 2 1 2 3 3 0

2 2

a b c ab ac bc a c b ac ab bc

ab ac

b c a b c a b c abc b c a b c a b c

b c b c a a abc ab abc ac a b a c

b c b a c a abc a a a b c ab ac

b c b c b c

+ + − − − + + − − −≤

+ +

⇔ − − − + − − − + − − − ≥

⇔ − + − − − + + + − − ≥

⇔ − − + − + + − + + + + ≥

⇔ − + + +( )2 22 2 0abc ab ac+ + ≥

bất ñẳng thức trên luôn ñúng, tương tự ta thu ñược

2 2 2 2 2 2 2 2 23 2 2 3 2 2 3 2 2

1 1 1

a b c ab ac bc a c b ac ab bc b c a bc ab ca

ab ac bc

+ + − − − + + − − − + + − − −≤ ≤

+ + +

Do ñó theo b) ta có

( ) ( )2 2 2 22 2 2 3 3 2 2 123 2 20

1 3 3

a b c ab ac bc a aba b c ab ac bc

ab ab ab

+ + − − − −+ + − − −≥ = ≥

+ + +

∑ ∑ ∑∑

∑ ∑

Ta có ñiều phải chứng minh.

Bài toán 5: Cho các số thực , ,x y z sao cho 1x y z+ + = .

Chứng minh rằng

2 2 2

9

1 1 1 10

x y z

x y z+ + ≤

+ + +

Lời giải: Ta có

2 2 2 2 2 21 1 1 1 1 1

x y zx y z

x y z x y z+ + ≤ + +

+ + + + + +

Do ñó ta chỉ cần chứng minh bất ñẳng thức trong trường hợp , , 0x y z > .

Không mất tính tổng quát, giả sử x y z≥ ≥ , khi ñó 2 2 21 1 1x y z+ ≥ + ≥ +

Page 38: Kỷ yếu hội thảo các trường chuyên dh db bắc bộ lần III

=========================================================== 40

HỘI CÁC TRƯỜNG THTP CHUYÊN KHU VỰC DUYÊN HẢI VÀ ðỒNG BẰNG BẮC BỘ

Hội thảo khoa học môn Toán học lần thứ III - 2010

Tiếp tục, ta kiểm tra ( )( )2 21 0

1 1

x yx y xy

x y≥ ⇔ − − ≥

+ +

Vì 1 2 1x y z x y xy xy+ + = ≥ + ≥ ⇒ < nên bất ñẳng thức trên ñúng.

Tương tự ta có 2 2 21 1 1

x y z

x y z≥ ≥

+ + +

Do ñó theo a) ta thu ñược

( )

( )2 2 2 2 2 2 2

3 3 911 1 1 3 1033

x y zx y z

x y z x y zx y z

+ ++ + ≤ ≤ =

+ + + + + + + + +

Ta có ñiều phải chứng minh.

Bài toán 6: Cho , , 0a b c > thỏa mãn

1 1 11

1 2 1 2 1 2ab bc ca+ + ≥

+ + +

Chứng minh rằng 3a b c abc+ + ≥

Lời giải:

Ta có

1 1 1

1 2 1 2 1 2 2 2 2

a b c

ab bc ca a abc b abc c abc+ + = + +

+ + + + + +

Không giảm tính tổng quát, giả sử a b c≥ ≥ , thế thì 2 2 2a abc b abc c abc+ ≥ + ≥ +

lại có 2 2 2

a b cab ac bc

a abc b abc c abc≥ ≥ ⇔ ≥ ≥

+ + + ( luôn ñúng )

do ñó theo a) ta có

( )31 1 1

1 2 1 2 1 2 2 2 2 6

a b ca b c

ab bc ca a abc b abc c abc a b c abc

+ ++ + = + + ≤

+ + + + + + + + +

kết hợp với giả thiết ta suy ra

( )31 3

6

a b ca b c abc

a b c abc

+ +≥ ⇔ + + ≥

+ + +

Ta có ñiều phải chứng minh, ñẳng thức xảy ra khi 1a b c= = =

Bài toán 7: Cho , , 0a b c > thỏa mãn 3ab bc ca+ + = .

Chứng minh rằng

Page 39: Kỷ yếu hội thảo các trường chuyên dh db bắc bộ lần III

=========================================================== 41

HỘI CÁC TRƯỜNG THTP CHUYÊN KHU VỰC DUYÊN HẢI VÀ ðỒNG BẰNG BẮC BỘ

Hội thảo khoa học môn Toán học lần thứ III - 2010

( ) ( ) ( )2 2 2

1 1 1 3

1 1 1 1 2a b c b c a c a b abc+ + ≤

+ + + + + + +

Lời giải: Bất ñẳng thức cần chứng minh tương ñương với

( ) ( ) ( )

( )( )

( )( )

( )( )

( ) ( ) ( )

2 2 2

1 1 1 1 1 10

1 2 1 1 2 1 1 2 1

1 1 10

1 3 1 3 1 3

1 1 10

1 3 1 3 1 3

abc a b c abc b c a abc c a b

a bc b ac c ba

a bc b ac c ba

bc ac ba

abc bc abc ac abc ba

− + − + − ≥+ + + + + + + + +

− − −⇔ + + ≥

+ − + − + −

− − −⇔ + + ≥

+ − + − + −

Không mất tính tổng quát, giả sử a b c≥ ≥ , khi ñó

( ) ( ) ( )1 3 1 3 1 3abc ba abc ac abc bc+ − ≤ + − ≤ + −

Và ta cũng có ( ) ( ) ( )

1 1 1

1 3 1 3 1 3

bc ac ba

abc bc abc ac abc ba

− − −≥ ≥

+ − + − + −

(vì 3 2 2 23 3 1ab bc ca a b c abc= + + ≥ ⇒ ≤ )

Do ñó theo b) ta có

( ) ( ) ( )( )

( )3 31 1 1

01 3 1 3 1 3 3 9

ab bc cabc ac ba

abc bc abc ac abc ba abc ab bc ca

− − −− − −+ + ≥ =

+ − + − + − + − − −

Ta có ñiều phải chứng minh.

IV. Một số bài tập áp dụng

Bài tập 1: (bất ñẳng thức Chebyshev)

Kí hiệu ,A B giống như trong bất ñẳng thức Sắp xếp lại.

CMR: ( )( )1 2 1 2... ...n na a a b b b

A Bn

+ + + + + +≥ ≥

Bài tập 2:

Cho n số thực dương 1 2, ,..., nc c c (với *n ∈� ). Ta kí hiệu

( ) ( )

( ) ( ) ( ) ( )

1 22 2 21 2 1 2

1

1 2 1 2

... , ... ,

... , 1 1 ... 1

n n

n

n n

RMS c c c n AM c c c n

GM c c c HM n c c c

= + + + = + + +

= = + + +

CMR: RHM AM GM HM≥ ≥ ≥

Bài tập 3:

Cho , , 0.a b c > CMR: 1

2

k ka a

a b

≥+

∑ ∑ ( 2)k∀ ≥

Bài tập 4:

Page 40: Kỷ yếu hội thảo các trường chuyên dh db bắc bộ lần III

=========================================================== 42

HỘI CÁC TRƯỜNG THTP CHUYÊN KHU VỰC DUYÊN HẢI VÀ ðỒNG BẰNG BẮC BỘ

Hội thảo khoa học môn Toán học lần thứ III - 2010

Cho ( ), , 1;a b c ∈ +∞ . CMR: 2 2 2log log log log log logab bc ca a bc ab c abcc a b bc ca ab+ + ≥ + +

Bài tập 5:

Kí hiệu , ,a b c lần lượt là ñộ dài các cạnh , ,BC CA AB của tam giác nhọn ABC .

CMR: ( )4cos cos cos

a b b c c aa b c

C A B

+ + ++ + ≥ + +

Bài tập 6:

Cho , , 0a b c > . Chứng minh các bất ñẳng thức sau:

a) ( ) ( ) ( )2 2 2

0a b b c b c c a c a a b

a b b c c a

− − −+ + ≥

+ + +

b) 3 3 3 3 3 3

2 2 2 2 2 2

a b b c c aa b c

b c c a a b

+ + ++ + ≥ + +

+ + +

c) 5 5 5 2 2 2

3 3 3 3 3 3 2

a b c a b c

a b b c c a

+ ++ + ≥

+ + +

Bài tập 7:

Cho , , 0a b c > thỏa mãn 3a b c+ + = .

CMR: 1 1 1 3

9 9 9 8ab bc ca+ + ≤

− − −

Bài tập 8:

Cho , , , 0a b c d ≥ thỏa mãn 4a b c d+ + + = .

CMR: 1 1 1 1

15 5 5 5abc bcd cda dab

+ + + ≤− − − −

Bài tập 9:

Cho , , 0a b c ≥ . CMR: ( ) 2 2 23 8 8 8a b c a bc b ca c ab+ + ≥ + + + + +

Bài tập 10:

Chứng minh rằng với , , 0a b c ≥ và 2 0k≥ ≥ ta có bất ñẳng thức

2 2 2

2 2 2 2 2 2 2 2 20

a bc b ca c ab

b c ka a c kb b a kc

− − −+ + ≥

+ + + + + +

V. Tài liệu tham khảo:

1. G.H. Hardy, J.E. Littlewood, G. Polya, Bất ñẳng thức.

2. Phạm Kim Hùng, Sáng tạo bất ñẳng thức ( tập 1).

3. VIMF (Nhiều tác giả), Discovery Inequalities (Third version).

-------------------------------------------------------------------------------

Page 41: Kỷ yếu hội thảo các trường chuyên dh db bắc bộ lần III

=========================================================== 43

HỘI CÁC TRƯỜNG THTP CHUYÊN KHU VỰC DUYÊN HẢI VÀ ðỒNG BẰNG BẮC BỘ

Hội thảo khoa học môn Toán học lần thứ III - 2010

TÍNH TUẦN HOÀN TRONG DÃY SỐ NGUYÊN Tác giả: Ngô Thị Hải

Giáo viên trường THPT chuyên Nguyễn Trãi, Hải Dương.

Dãy số là một lĩnh vực khó và rất rộng. ðể giải ñược các bài toán loại này không chỉ ñòi hỏi người làm Toán phải sử dụng nhiều kiến thức khác nhau của Toán học mà còn phải

có khả năng sáng tạo rất cao. Trong các bài toán về dãy số một vấn ñề ñược quan tâm nhiều

là tính chất số học của dãy số như: tính chia hết, tính chất nguyên hay tính chính phương…

Chúng rất ña dạng và phong phú. Trong nhiều trường hợp, dãy số chỉ là vỏ bề ngoài còn

bản chất bài toán là một bài số học. Chính vì lẽ ñó, các bài toán về số học nói chung, các

bài toán về tính chất số học của dãy số nói riêng thường xuất hiện trong các kì thi học sinh

giỏi quốc gia và quốc tế, vì nó bao gồm nhiều bài toán hay và khó. Trong khuôn khổ của bài

viết này tôi chỉ ñề cập ñến một khía cạnh rất nhỏcủa dãy số nguyênñólà tính tuần hoàn, hi

vọng rằng ñây là một tài liệu tham khảo tốt cho các em học sinh khá và giỏi. Trước hết ta

hãy xem ñịnh lý sâu ñây:

ðịnh lý: Cho dãy số nguyên truy hồi cấp k ( k là số nguyên dương) nghĩa là

Nếu dãy bị chặn thì nó là dãy tuần hoàn kể từ lúc nào ñó. Chứng minh: Giả sử dãy bị chặn bởi số nguyên dương M, nghĩa là .

Xét các bộ k số Có tối ña

bộ khác nhau nên trong bộ ñầu tiên phỉa có 2 bộ trùng nhau. Chẳng hạn

Nghĩa là

Mà nên

ðặt thì ta có

Vậy dãy tuần hoàn với chu kì kể từ

Hệ quả : Cho dãy số nguyên thoả mãn

trong ñó là các số nguyên và m là số

nguyên dương lớn hơn 1. Gọi là số dư trong phép chia cho m. Khi ñó dãy tuần hoàn.

Chứng minh: Theo giả thiết ta có . Theo tính chất của ñồng dư thức ta có

Theo các xác ñịnh ta có tức là dãy bị chặn và truy hồi tuyến tính

cấp k nên theo ñịnh lý trêndãy tuần hoàn kể từ lúc nào ñó, nghĩa là sao cho

Chọn ta ñược

Vậy . Tương tự ta có = ,…,

Do ñó dãy tuần hoàn với chu kì T.

Page 42: Kỷ yếu hội thảo các trường chuyên dh db bắc bộ lần III

=========================================================== 44

HỘI CÁC TRƯỜNG THTP CHUYÊN KHU VỰC DUYÊN HẢI VÀ ðỒNG BẰNG BẮC BỘ

Hội thảo khoa học môn Toán học lần thứ III - 2010

Sau ñây tôi sẽ ñưa ra một số ví dụ ñiển hình về việc áp dụng ñịnh lý trên. Các bài toán

nêu ra ở ñây ñều sử dụng ñến tính tuần hoàn của dãy số dư. Giả sử là số dư trong phép

chia cho một số nguyên dương m nào ñó. Khi ñó dãy bị chặn và cũng có cùng công

thức truy hồi với dãy nên theo hệ quả trên nó là dãy tuần hoàn.

Bài 1: Cho Với là số dư của phép chia cho

100. Tìm số dư trong phép chia cho 8.

Bài giải: Gọi là số dư trong phép chia cho 4. Theo giả thiết

nên

Mặt khác tức là dãy bị chặn do ñó dãy này tuần hoàn.

Ta tính ñược

Dễ kiểm tra tuần hoàn chu kì 6, nghĩa là

Lại có . Do nên cùng tính chẵn lẻ suy ra

hay

Vậy Mà

. Do ñó

chia hết cho 8.

Bài 2: Cho dãy , n=0,1,2,… xác ñịnh bởi và

Chứng minh rằng: chia hết cho 20

Bài giải: Từ công thức truy hồi của dãy ta thấy ( .Gọi là số dư trong

phép chia cho 4. Khi ñó Hơn nữa ( nên tương tự

bài 1 dãy tuần hoàn chu kì 6.

Ta có ( . Vì vậy

tức là chia hết cho 4.

Mặt khác với ta có

Suy ra

Vậy . Do ñó

chia hết cho 20. Bài 3:

Cho dãy , n=,1,2,3… xác ñịnh bởi

Chứng minh rằng tồn tại vô số số hạng của dãy chia hết cho 2005. Bài giải:

Page 43: Kỷ yếu hội thảo các trường chuyên dh db bắc bộ lần III

=========================================================== 45

HỘI CÁC TRƯỜNG THTP CHUYÊN KHU VỰC DUYÊN HẢI VÀ ðỒNG BẰNG BẮC BỘ

Hội thảo khoa học môn Toán học lần thứ III - 2010

Ta có chia hết cho 2005. Gọi là số dư trong phép chia cho 2005. Từ

công thức truy hồi của dãy ta có

ðồng thời dãy tuần hoàn kể từ lúc nào ñó, nghĩa là sao cho

.

Chọn ta ñược

Vậy . Tương tự ta cũng có

Do ñó hay chia hết cho 2005 (ñpcm)

Bài 4:

Cho dãy , n=,1,2,3… xác ñịnh bởi

Chứng minh rằng với mọi số nguyên dương tồn tai vô số số tự nhiên sao cho

cùng chia hết cho .

Bài giải:

Xét dãy , n=,1,2,3… xác ñịnh như sau

Ta tính ñược Do ñó

Gọi là số dư trong phép chia cho m. Khi ñó dãy tuần hoàn nghĩa là tồn tại số

tự nhiên T>1 sao cho

Vậy chia hết cho m với hay

chia hết cho m với .

Bài 5: Gọi là nghiệm dương lớn nhất của phương trình Xét dãy xác

ñịnh theo công thức sau: Tìm số dư trong phép chia cho 17.

Bài giải:

ðặt Ta có >0,

.

Do là hàm lien tục trên R nên phương trình có 3 nghiệm phân biệt:

ðặt Khi ñó là nghiệm của phương trình sai phân tuyến

tính thuần nhất có pt ñặc trưng là Do ñó ta có

Hay trong ñó

( sử dụng ñịnh lý Vi-et) Vì vậy Do >0

Suy ra .

Page 44: Kỷ yếu hội thảo các trường chuyên dh db bắc bộ lần III

=========================================================== 46

HỘI CÁC TRƯỜNG THTP CHUYÊN KHU VỰC DUYÊN HẢI VÀ ðỒNG BẰNG BẮC BỘ

Hội thảo khoa học môn Toán học lần thứ III - 2010

Lại có nên

(do .

Vậy =( . Cho nên

=

Vậy

Gọi là số dư trong phép chia cho 17. Khi ñó dãy tuần hoàn và bằng tính toán trực tiếp ta có

Dễ kiểm tra tuần hoàn chu kì 16, nghĩa là là

Vậy nên

hay chia 17dư 6. Cuối cung tôi xin nêu thêm 2 bài tập khác có thể giải theo phương pháp này ñể bạn ñọc

tham khảo

Bài 1:

Cho dãy , n=,1,2,3… xác ñịnh bởi

Chứng minh rằng: a) Mọi số hạng của dãy ñều là số nguyên dương. b) Có vô số nguyên dương n sao cho có 4 chữ số tận cùng là 2003.

c) Không tồn tại số nguyên dương n sao cho có 4 chữ số tận cùng là 2004.

Hướng dẫn: Biến ñổi ñể dẫn ñến

Bài 2:

Dãy số nguyên , n=,1,2,3… xác ñịnh bởi

Chứng minh có vô số số hạng của dãy chia hết cho 1986.

Page 45: Kỷ yếu hội thảo các trường chuyên dh db bắc bộ lần III

=========================================================== 47

HỘI CÁC TRƯỜNG THTP CHUYÊN KHU VỰC DUYÊN HẢI VÀ ðỒNG BẰNG BẮC BỘ

Hội thảo khoa học môn Toán học lần thứ III - 2010

ðỊNH LÝ PASCAL VÀ ỨNG DỤNG Lê ðức Thịnh

GV THPT Chuyên Trần Phú – Hải Phòng

Trong bài viết chuyên ñề này tôi muốn ñề cập ñến một ñịnh lý có rất nhiều ứng dụng ña dạng, ñó là ñịnh lý Pascal về lục giác nội tiếp ñường tròn. Trong thực tế áp dụng, khi thay

ñổi thứ tự các ñiểm, hay là khi xét các trường hợp ñặc biệt ta sẽ thu ñược rất nhiều kết quả

khác nhau.

Trước hết ta phát biểu nội dung ñịnh lý:

ðịnh lý Pascal: Cho các ñiểm A, B,C, D,E,F cùng thuộc một ñường tròn (có thể hoán ñổi thứ tự). Gọi

P AB DE,Q BC EF,R CD FA= Ç = Ç = Ç .

Khi ñó các ñiểm P,Q,R thẳng hàng.

Chứng minh: Gọi

X EF AB, Y AB CD, Z CD EF.= Ç = Ç = Ç

Áp dụng ñịnh lý Menelaus cho tam giác

XYZ ñối với các ñường thẳng BCQ,DEP,FAR ,

ta có:

( )

( )

( )

CY BX QZ1 1

CZ BY QX

FZ AX RY1 2

FX AY RZEZ PX DY

1 3EX PY DZ

× × =

× × =

× × =

Mặt khác, theo tính chất phương tích của một ñiểm ñối với ñường tròn ta có:

( )YC.YD YB.YA, ZF.ZE ZD.ZC,XB.XA XF.XE 4= = =

Nhân (1),(2) và (3) theo vế, ta ñược:

( )

QZ RY PX CY.BX.FZ.AX.EZ.DY1

QX RZ PY CZ.BY.FX.AY.EX.DZ

QZ RY PX YC.YD ZF.ZE XB.XA1 5

QX RZ PY YB.YA ZD.ZC XF.ZE

× × × =

Û × × × × × =

Thế (4) vào (5), ta ñược QZ RY PX

1.QX RZ PY

× × =

Vậy P,Q,R thẳng hàng (theo ñịnh lý Menelaus).

ðường thẳng PQR ở trên ñược gọi là ñường thẳng Pascal ứng với bộ ñiểm

A,B,C,D, E, F .

Bằng cách hoán vị các ñiểm A,B,C,D, E, F ta thu ñược rất nhiều các ñường thẳng

Pascal khác nhau, cụ thể ta có tới 60 ñường thẳng Pascal.

Z

YX

R

Q

P AB

C

DE

F

Page 46: Kỷ yếu hội thảo các trường chuyên dh db bắc bộ lần III

=========================================================== 48

HỘI CÁC TRƯỜNG THTP CHUYÊN KHU VỰC DUYÊN HẢI VÀ ðỒNG BẰNG BẮC BỘ

Hội thảo khoa học môn Toán học lần thứ III - 2010

Chẳng hạn hình vẽ bên minh họa trường hợp các

ñiểm ACEBFD.

Ngoài ra khi cho các ñiểm có thể trùng nhau (khi ñó lục giác suy biến thành tam giác, tứ giác, ngũ giác), ví dụ

E Fº thì cạnh EF trở thành tiếp tuyến của ñường tròn tại

E , ta còn thu thêm ñược rất nhiều các ñường thẳng Pascal

khác nữa. Hình vẽ dưới ñây minh họa trường hợp các ñiểm

ABCDEE, ABCCDD, AABBCC:

Tiếp theo ta ñưa ra các bài toán ứng dụng ñịnh lý Pascal:

Bài toán 1: (ðịnh lý Newton)

Một ñường tròn nội tiếp tứ giác ABCD lần lượt tiếp xúc với các cạnh AB, BC,CD,DA

tại E,F,G,H .

Khi ñó các ñường thẳng AC,EG,BD, FH ñồng quy.

R

Q

YP AB

C

D

E

R

Q

P

AD

BC

Q

R

PB

C

A

P

QR

A

B

C D

E

F

Page 47: Kỷ yếu hội thảo các trường chuyên dh db bắc bộ lần III

=========================================================== 49

HỘI CÁC TRƯỜNG THTP CHUYÊN KHU VỰC DUYÊN HẢI VÀ ðỒNG BẰNG BẮC BỘ

Hội thảo khoa học môn Toán học lần thứ III - 2010

Lời giải: Gọi O EG FH,X EH FG= Ç = Ç .

Vì D là giao ñiểm của các tiếp tuyến với ñường tròn tại G,H, áp dụng

ñịnh lý Pascal cho các ñiểm E,G,G, F,H,H , ta có:

EG FH O,

GG HH D,

GF HE X.

Ç =

Ç =

Ç =

Suy ra O,D,X thẳng hàng.

Áp dụng ñịnh lý Pascal cho các ñiểm

E,E,H,F,F,G, ta có:

EE FF B,

EH FG X,

HF GE O.

Ç =

Ç =

Ç =

Suy ra B,X,O thẳng hàng.

Từ ñó ta ñược B,O,D thẳng hàng.

Vậy EG,FH,BD ñồng quy tại O .

Chứng minh tương tự ñối với ñường thẳng AC ta ñược ñiều phải chứng minh.

Bài toán 2: Cho tam giác ABC nội tiếp trong một ñường tròn. Gọi D, E lần lượt là các ñiểm chính

giữa của các cung AB,AC ; P là ñiểm tuỳ ý trên cung BC ; DP AB Q,PE AC RÇ = Ç = .

Chứng minh rằng ñường thẳng QR chứa tâm I của ñường tròn nội tiếp tam giác ABC .

Lời giải: Vì D,E lần lượt là ñiểm chính giữa của các

cung AB, AC nên CD, BE theo thứ tự là các ñường

phân giác của góc · ·ACB,ABC .

Suy ra I CD EB.= Ç Áp dụng ñịnh lý Pascal cho sáu ñiểm

C, D,P,E,B,A, ta có:

CD EB IÇ = ;

DP BA Q;Ç =

PE AC R.Ç =

Vậy Q, I,R thẳng hàng.

Bài toán 3: (Australia 2001)

X

O

C

D

A

B

G

E

H

F

I R

QE

D A

B C

P

Page 48: Kỷ yếu hội thảo các trường chuyên dh db bắc bộ lần III

=========================================================== 50

HỘI CÁC TRƯỜNG THTP CHUYÊN KHU VỰC DUYÊN HẢI VÀ ðỒNG BẰNG BẮC BỘ

Hội thảo khoa học môn Toán học lần thứ III - 2010

Cho tam giác ABC nội tiếp ñường tròn (O), ñường cao ñỉnh A, B, C lần lượt cắt (O) tại

A’, B’, C’. D nằm trên (O), DA ' BC A",DB' CA B",DC' AB C"Ç = Ç = Ç = .

Chứng minh rằng: A”, B”, C”, trực tâm H thẳng hàng.

Lời giải: Áp dụng ñịnh lý Pascal cho sáu ñiểm

A,A ',D,C ',C,B, ta có:

AA ' C 'C H,

A 'D CB A",

DC' BA C".

Ç =

Ç =

Ç =

Vậy H, A",C" thẳng hàng.

Tương tự suy ra A”, B”, C”, H thẳng hàng.

Bài toán 4: (IMO Shortlist 1991)

P thay ñổi trong tam giác ABC cố ñịnh. Gọi P’, P” là hình chiếu vuông góc của P trên

AC, BC, Q’, Q” là hình chiếu vuông góc của C trên AP, BP, gọi X P 'Q" P"Q '= Ç .

Chứng minh rằng: X di chuyển trên một ñường cố ñịnh.

Lời giải: Ta có:

· · · · 0CP 'P CP"P CQ 'P CQ"P 90= = = =

Nên các ñiểm C,P ',Q",P,Q ',P" cùng thuộc

một ñường tròn.

Áp dụng ñịnh lý Pascal cho sáu ñiểm

C, P ',Q",P,Q ',P" ta có:

CP ' PQ ' A,

P 'Q" Q 'P" X,

Q"P P"C B.

Ç =

Ç =

Ç =

Vậy A, X,B thẳng hàng.

Vậy X di chuyển trên ñường thẳng AB cố ñịnh.

Bài toán 5: (Poland 1997) Ngũ giác ABCDE lồi thỏa mãn:

· · 0CD DE,BCD DEA 90= = = . ðiểm F trong ñoạn

AB sao cho AF AE

BF BC= ×

Chứng minh rằng: · · · ·FCE ADE,FEC BDC= = .

Lời giải:

HC"

B"

A"

C'

B'

A'

B C

A

D

X

Q"

Q'

P"

P'

A

B C

P

R

Q

P

F

A E

D

C

B

Page 49: Kỷ yếu hội thảo các trường chuyên dh db bắc bộ lần III

=========================================================== 51

HỘI CÁC TRƯỜNG THTP CHUYÊN KHU VỰC DUYÊN HẢI VÀ ðỒNG BẰNG BẮC BỘ

Hội thảo khoa học môn Toán học lần thứ III - 2010

Gọi P AE BC= Ç , Q, R lần lượt là giao ñiểm của AD và BD với ñường tròn ñường

kính PD, G QC RE= Ç .

Áp dụng ñịnh lý Pascal cho sáu ñiểm P,C,Q, D, R,E, ta có:

PC DR B,

CQ RE G,

QD EP A.

Ç =

Ç =

Ç =

Vậy A,G,B thẳng hàng.

Lại có:

·

·

·

·

·

·

·

·

·

·

DAG

DBG

DAE

DBC

sin GQDDA GQSAG DG.DA.sin GDQ DA.GQ DA.sin QREDG

BG S DB.GRDG.DB.sin GDR sin GRD DB.sin RQCDB GR

DG

SDA.sin ADE DA.DE.sin ADE AE

S BCDB.sin BDC DB.DC.sin BDC

AG AFF G

BG BF

× ×= = = = =

× ×

= = = =

Þ = Þ º

Từ ñó dễ dàng có · · · ·FCE ADE,FEC BDC= = .

Bài toán 6: Cho tam giác ABC nội tiếp ñường tròn (O), A’, B’, C’ là trung ñiểm BC, CA, AB. Chứng minh rằng tâm ñường tròn ngoại tiếp các tam giác AOA’, BOB’, COC’ thẳng

hàng.

Lời giải: Gọi A”, B”, C” là trung ñiểm của OA,

OB, OC. I, J, K là tâm các ñường tròn ngoại tiếp các tam giác AOA’, BOB’, COC’. Khi ñó I là giao ñiểm của các trung trực của OA và OA’, hay chính là giao ñiểm của B”C” và tiếp tuyến của ñường tròn (O;OA”) tại A”. Tương tự với J, K.

Áp dụng ñịnh lý Pascal cho sáu ñiểm

A", A",B",B",C",C" ta có:

A"A" B"C" I,

A"B" C"C" K,

B"B" C"A" J.

Ç =

Ç =

Ç =

Vậy I, J,K thẳng hàng.

Bài toán 7: (China 2005) Một ñường tròn cắt các cạnh của tam giác ABC theo thứ tự tại các ñiểm

1 2 1 2 1 2D ,D ,E ,E ,F ,F . 1 1 2 2 1 1 2 2 1 1 2 2D E D F L,E F E D M,F D F E N∩ = ∩ = ∩ = .

Chứng minh rằng AL, BM, CN ñồng quy.

K

J

IB"

A"

C"

C' B'

A'

O

B C

A

Page 50: Kỷ yếu hội thảo các trường chuyên dh db bắc bộ lần III

=========================================================== 52

HỘI CÁC TRƯỜNG THTP CHUYÊN KHU VỰC DUYÊN HẢI VÀ ðỒNG BẰNG BẮC BỘ

Hội thảo khoa học môn Toán học lần thứ III - 2010

Lời giải:

Gọi 1 1 2 2 1 1 2 2 1 1 2 2D F D E P,E D E F Q,F E F D R∩ = ∩ = ∩ = .

Áp dụng ñịnh lý Pascal cho sáu ñiểm 2 1 1 1 2 2E ,E ,D ,F ,F ,D ta có:

2 1 1 2

1 1 2 2

1 1 2 2

E E FF A,

E D F D L,

D F D E P.

Ç =

Ç =

Ç =

Suy ra A, L,P thẳng hàng.

Tương tự B, M, Q thẳng hàng, C, N, R thẳng hàng.

2 1 1 2 1 2 2 1 1 2 1 2 2 1 1 2 1 2E E D F CA D F X,F F E D AB E D Y,D D F E BC FE Z∩ = ∩ = ∩ = ∩ = ∩ = ∩ =

Áp dụng ñịnh lý Pascal cho sáu ñiểm 1 1 1 2 2 2F ,E ,D ,D , F ,E ta có:

1 1 2 2

1 1 2 2

1 2 2 1

F E D F R,

E D F E Q,

D D E F Z.

Ç =

Ç =

Ç =

Suy ra Q,R, Z thẳng hàng.

Tương tự P, Q, Y thẳng hàng, Z, P, X thẳng hàng.

Xét các tam giác ABC, PQR có: X CA RP,Y AB PQ, Z BC QR= ∩ = ∩ = ∩ .

Áp dụng ñịnh lý Desargues suy ra các ñường thẳng AP AL,BQ BM,CR CN≡ ≡ ≡ ñồng

quy. Bài toán 8: (ðịnh lý Brianchon) Lục giác ABCDEF ngoại tiếp một

ñường tròn. Khi ñó AD, BE, CF ñồng quy.

Lời giải:

Z

NM

R

Q

P

L

F2

F1

E2

E1

D2D1

A

B

C

NPM

A

B

C D

E

F

H

G

L

K

J

I

Page 51: Kỷ yếu hội thảo các trường chuyên dh db bắc bộ lần III

=========================================================== 53

HỘI CÁC TRƯỜNG THTP CHUYÊN KHU VỰC DUYÊN HẢI VÀ ðỒNG BẰNG BẮC BỘ

Hội thảo khoa học môn Toán học lần thứ III - 2010

Ta sẽ chứng minh ñịnh lý này bằng cực và ñối cực ñể thấy rằng Pascal và Brianchon là hai kết quả liên hợp của nhau.

Gọi các tiếp ñiểm trên các cạnh lần lượt là G, H, I, J, K, L. Khi ñó GH, HI, IJ, JK, KL, LG lần lượt là ñối cực của B, C, D, E, F, A.

Gọi GH JK N, HI KL P, IJ LG=M∩ = ∩ = ∩

Theo Pascal cho lục giác GHIJKL ta có M, N, P thẳng hàng. Mà M, N, P lần lượt là ñối cực của AD, BE, CF nên suy ra AD, BE, CF ñồng quy tại

cực của ñường thẳng MNP.

Bài toán 9: Cho tam giác ABC, các phân giác và ñường cao tại ñỉnh B, C là BD, CE, BB’, CC’.

ðường tròn nội tiếp (I) tiếp xúc với AB, AC tại N, M. Chứng minh rằng MN, DE, B’C’ ñồng quy.

Lời giải: Gọi hình chiếu của C trên BD là P,

hình chiếu của B trên CE là Q. Dễ chứng minh:

��

� � � 0ANMI ICP NMI PMI 180

2= = ⇒ + =

Nên M, N, P thẳng hàng. Tương tự suy ra M, N, P, Q thẳng

hàng. Áp dụng ñịnh lý Pascal cho sáu

ñiểm B',C ',B,P,Q,C, ta có:

B 'C ' PQ S,

C 'B QC E,

BP CB' D.

Ç =

Ç =

Ç =

Vậy S, E,D thẳng hàng, hay là MN, DE, B’C’ ñồng quy tại S.

Bài toán 10: Cho tam giác ABC nội tiếp ñường tròn (O). Tiếp tuyến của (O) tại A, B cắt nhau tại S.

Một cát tuyến quay quanh S cắt CA, CB tại M, N, cắt (O) tại P, Q. Chứng minh rằng M, N, P, Q là hàng

ñiểm ñiều hòa.

Lời giải: Vẽ tiếp tuyến ME, MD của (O) cắt SA,

SB tại K, L. Áp dụng ñịnh lý Newton cho tứ giác

ngoại tiếp SKML ta có BE, AD, SM, KL ñồng quy.

P

Q

S

C'

B'

N

M

IE

D

A

B C

I

L

K

D

E

MN QS

A

B

CP

Page 52: Kỷ yếu hội thảo các trường chuyên dh db bắc bộ lần III

=========================================================== 54

HỘI CÁC TRƯỜNG THTP CHUYÊN KHU VỰC DUYÊN HẢI VÀ ðỒNG BẰNG BẮC BỘ

Hội thảo khoa học môn Toán học lần thứ III - 2010

Áp dụng ñịnh lý Pascal cho sáu ñiểm A, D, E, E,B,C, ta có:

AD EB I,

DE BC N ',

EE CA M.

Ç =

Ç =

Ç =

Vậy I, N ',M thẳng hàng, hay N N '≡ , tức là N DE∈ .

Do DE là ñối cực của M ñối với (O) nên M, N, P, Q là hàng ñiểm ñiều hòa.

Bài toán 11: (ðịnh lý Steiner) ðường thẳng Pascal của các lục giác ABCDEF, ADEBCF, ADCFEB ñồng quy.

Lời giải:

Gọi 1 1 2AB DE P ,BC EF Q , AD BC P ,∩ = ∩ = ∩ =

2 3 3DE CF Q ,AD FE P ,CF AB Q .∩ = ∩ = ∩ =

Áp dụng ñịnh lý Pascal cho sáu ñiểm

A,B,C,F,E,D, ta có:

1 3 1 3

1 2 1 2

2 3 2 3

P Q Q P AB FE P,

P Q Q P BC ED Q,

Q Q P P CF DA R.

Ç = Ç =

Ç = Ç =

Ç = Ç =

Vậy P,Q,R thẳng hàng.

Áp dụng ñịnh lý Desargues suy ra các ñường thẳng

1 1 2 2 3 3P Q ,P Q ,P Q ñồng quy.

Hay ñường thẳng Pascal của các lục giác ABCDEF, ADEBCF, ADCFEB ñồng quy.

Bài toán 12: (ðịnh lý Kirkman) ðường thẳng Pascal của các lục giác ABFDCE, AEFBDC, ABDFEC ñồng quy.

Ta ñã biết ở trên là có 60 ñường thẳng Pascal. Cứ 3 ñường một ñồng quy tạo ra 20 ñiểm Steiner. Trong 20 ñiểm Steiner cứ 4 ñiểm một lại nằm trên một ñường thẳng tạo ra 15 ñường thẳng Plucker. Ngoài ra 60 ñường thẳng Pascal ñó lại cứ 3 ñường một ñồng quy tạo ra 60 ñiểm Kirkman. Mỗi ñiểm Steiner lại thẳng hàng với 3 ñiểm Kirkman trên 20 ñường thẳng Cayley. Trong 20 ñường thẳng Cayley, cứ 4 ñường một lại ñồng quy tạo ra 15 ñiểm Salmon …

ðể kết thúc xin ñưa ra một số bài toán khác áp dụng ñịnh lý Pascal: Bài toán 13: (MOSP 2005) Cho tứ giác nội tiếp ABCD, phân giác góc A cắt phân giác góc B tại E. ðiểm P, Q lần

lượt nằm trên AD, BC sao cho PQ ñi qua E và PQ song song với CD.

Chứng minh rằng AP BQ PQ+ = .

R

Q

P

Q3P3

Q2

P2

Q1P1

A F

B

C D

E

Page 53: Kỷ yếu hội thảo các trường chuyên dh db bắc bộ lần III

=========================================================== 55

HỘI CÁC TRƯỜNG THTP CHUYÊN KHU VỰC DUYÊN HẢI VÀ ðỒNG BẰNG BẮC BỘ

Hội thảo khoa học môn Toán học lần thứ III - 2010

Bài toán 14:

Các ñiểm P, Q trong tam giác ABC sao cho � � 0BP CP,BQ CQ,ABP ACQ 180= = + = .

Chứng minh rằng � �BAP CAQ= .

Bài toán 15: (IMO Shortlist 2007)

Cho tam giác ABC cố ñịnh, các trung ñiểm 1 1 1A ,B ,C của BC, CA, AB tương ứng.

ðiểm P thay ñổi trên ñường tròn ngoại tiếp tam giác. Các ñường thẳng 1 1 1PA ,PB ,PC cắt lại

ñường tròn tại A’, B’, C’ tương ứng. Giả sử các ñiểm A, B, C, A’, B’, C’ ñôi một phân biệt và các ñường thẳng AA’, BB’, CC’ tạo ra một tam giác.

Chứng minh rằng diện tích của tam giác ñó không phụ thuộc vào vị trí của P.

Bài toán 16: Hai tam giác ABC, A’B’C’ có cùng ñường tròn ngoại tiếp. Các cạnh của hai tam giác

cắt nhau tại 6 ñiểm tạo ra một hình lục giác. Chứng minh rằng các ñường chéo của hình lục giác ñó ñồng quy. Bài toán 17: (IMO 2010)

ðiểm P nằm trong tam giác ABC với CA CB≠ . Các ñường AP, BP, CP cắt lại ñường tròn ngoại tiếp tại K, L, M. Tiếp tuyến của ñường tròn ngoại tiếp tại C cắt AB ở S. Giả sử

SC SP= . Chứng minh rằng MK ML= . Bài toán 18: (MEMO 2010) ðường tròn nội tiếp tam giác ABC tiếp xúc các cạnh BC, CA, AB tại D, E, F tương ứng.

K là ñối xứng của D qua tâm ñường tròn nội tiếp. DE cắt FK tại S. Chứng minh rằng AS song song BC.

Page 54: Kỷ yếu hội thảo các trường chuyên dh db bắc bộ lần III

=========================================================== 56

HỘI CÁC TRƯỜNG THTP CHUYÊN KHU VỰC DUYÊN HẢI VÀ ðỒNG BẰNG BẮC BỘ

Hội thảo khoa học môn Toán học lần thứ III - 2010

HÀM SỐ HỌC VÀ MỘT SỐ BÀI TOÁN VỀ HÀM SỐ HỌC

Trường THPT Chuyên Hưng Yên Hàm số học là hàm số có miền xác ñịnh là tập con của tập số tự nhiên . Kí hiệu A là tập hợp tất cả các số học. I.Một số tính chất chung của các hàm số học 1.Tích chập Dirichlet (gọi là tích chập) ðịnh nghĩa 1. Cho f và g là hai hàm số học. Tích chập Dirichlet gọi là tích chập của

f và ,g ký hiệu là f * g , xác ñịnh bởi

( ) ( ) ( ) *,*|

Nnd

ngdfngf

nd

∈∀

=∑

trong ñó tổng lấy trên tất cả các số nguyên dương d mà chia hết n . Ví dụ 1. Ta xét hai hàm số học sau nếu

• ( )

=0

1nδ

• ( ) 1=ne với mọi .*Nn ∈ Khi ñó với mọi Af ∈ ta có

( ) ( ) ( ) ( )nfd

ndfnfa

nd

=

=∑

|

*) δδ với mọi *Nn ∈ ⇒ .* ff =δ

( ) ( ) ( ) ( )∑∑ =

=

ndnd

dfd

nedfnefb

||

*) với mọi *Nn ∈ .

ðịnh lý 1. Với Ahgf ∈,, ta có

( ) ( )( ) .***)

.****)

.**)

hfgfhgfiii

hgfhgfii

fggfi

+=+

=

=

2. Toán tử ( )AfRafTa ∈∈ , và ( )AfLf ∈

ðịnh nghĩa 2. Cho ,Ra ∈ với mỗi Af ∈ ta xác ñịnh toán tử fTa như sau

( ) ( ) .*NnnnfnfTa

a ∈∀=

ðịnh nghĩa 3. Với mỗi Af ∈ ,ta xác ñịnh toán tử Lf như sau

( ) ( ) .*ln NnnnfnLf ∈∀= Ví dụ 2. Với hai hàm số học δ và e , ta có • ( ) ( ) ( ) δδδδδ =⇒∈∀== a

a

a TNnnnnnT * .

• ( ) ( ) .*NnnnneneTaa

a ∈∀==

• ( ) ( ) .*0ln NnnnnL ∈∀== δδ

• ( ) ( ) .*lnln NnnnnenLe ∈∀== ðịnh lý 2. Cho Agf ∈, ta có

( )( ) ( )( ) .)

.,**)

.***)

LgLfgfLiii

RagTfTgfTii

LgfLfggfLi

aaa

+=+

∈=

+=

iv) Ký hiệu kf * là tích chập của f với chính nó k lần, nghĩa là

n�u n =1

n�u *Nn ∈ , n>1

Page 55: Kỷ yếu hội thảo các trường chuyên dh db bắc bộ lần III

=========================================================== 57

HỘI CÁC TRƯỜNG THTP CHUYÊN KHU VỰC DUYÊN HẢI VÀ ðỒNG BẰNG BẮC BỘ

Hội thảo khoa học môn Toán học lần thứ III - 2010

.*...***;*;* 10fffffff

k === δ k lần Khi ñó với +∈ Zk thì ( ) ( ) .*** 1

AfLfkffLkk ∈∀= −

3. Hàm số Mobius µ ðịnh nghĩa 4. Cho ,Af ∈ hàm Ag ∈ ñược gọi là nghịch ñảo tích chập của f nếu

δ=gf * , ký hiệu .* 1−= fg ðịnh lý 3. Cho Agf ∈, , ta có

i) 1*−f tồn tại khi và chỉ khi ( ) .01 ≠f

ii) Nếu tồn tại 1*−f thì 1*−

f ñược xác ñịnh duy nhất theo quy nạp như sau

( )( )

( )( )

( )∑>

−−

∈≥∀

−=

=

ndd

Nnnd

nfdf

fnf

ff

|1

11

1

.,2*1

1*

,1

11*

ðặc biệt nếu p là số nguyên tố thì ( ) ( )( )

.1

*2

1

f

pfpf −=−

iii) Nếu tồn tại 1*−f thì ( ) ( ).*** 1

LfffLf −=−

iv) Nếu tồn tại 11 *,* −−gf thì cũng tồn tại ( ) 1** −

gf và ñược xác ñịnh như sau

( ) =−1** gf11 *** −−

gf

Nhận xét: Từ ví dụ 1 và ñịnh lý 3 suy ra luôn luôn tồn tại 1*−δ và 1*−e và xác ñịnh

( )( )

( )

( )( )

( ) ( )∑>

−−

>∀==

−=

===

ndd

nnd

ndn

|1

11

1

.10*1

1*

,111

11*

δδδδ

δ

δδ

δ

⇒ δδ =−1* . ðối với hàm số học e việc tìm 1*−

e sẽ khó hơn , ta sẽ xét dưới ñây. ðịnh nghĩa 5. Hàm Mobius µ ñược ñịnh nghĩa là nghịch ñảo tích chập của hàm e,

nghĩa là 1*−= eµ .

Nhận xét: Từ ví dụ 1 và ñịnh nghĩa hàm Mobius ta suy ra kết quả quan trọng sau

( ) ( )( ) ( )∑

>

====

nd nnêu

nnêunned

| .10

,11* δµµ

ðịnh lý 4. Ta có i) Nếu *Nn ∈ thì

( ) ( )

−=

0

1

1r

ii) Nếu AFf ∈, thì

( ) ( ) ( ) ( ) .*||

Nnd

nFdnfdfnF

ndnd

∈∀

=⇔= ∑∑ µ

n�u n = 1

n�u n có phân tích tiêu chu�n là n = p1p2…pr, r>1

n�u t�n t�i s� nguyên t� p sao cho p2|n.

Page 56: Kỷ yếu hội thảo các trường chuyên dh db bắc bộ lần III

=========================================================== 58

HỘI CÁC TRƯỜNG THTP CHUYÊN KHU VỰC DUYÊN HẢI VÀ ðỒNG BẰNG BẮC BỘ

Hội thảo khoa học môn Toán học lần thứ III - 2010

(Công thức ñảo ngược ðêñêkin-Liuvin) Từ ñịnh lý trên ta có

( ) ( ) ( ) ( ) ( ) ( ) ( ) ( ) .115.315;028;112;11 231=−====−=−==

−µµµµµµ

4. Hàm nhân tính ðịnh nghĩa 6. Cho Af ∈ và f không ñồng nhất bằng không,

Hàm f ñược gọi là hàm nhân tính nếu ( ) ( ) ( )nfmfmnf = với mọi *, Nnm ∈ thỏa mãn

(m,n) =1. Hàm f ñược gọi là hàm hoàn toàn nhân tính nếu ( ) ( ) ( )nfmfmnf = với mọi *, Nnm ∈ .

Ký hiệu M là tập hợp tất cả các hàm nhân tính. Nhận xét: Từ ñịnh nghĩa trên, giả sử k

kpppnααα ...21

21= là sự phân tích tiêu chuẩn của n, ta có kết

quả sau

• Nếu f là hàm nhân tính thì ( ) ( ).1

∏=

=k

i

iipfnf

α

• Nếu f là hàm hoàn toàn nhân tính thì ( ) ( )( )ααpfpf = với mọi *N∈α và p

nguyên tố. • ðể chứng minh hai hàm nhân tính bằng nhau chỉ cần chứng minh chúng bằng nhau

trên mọi lũy thừa của các số nguyên tố. • ðể chứng minh hai hàm hoàn toàn nhân tính bằng nhau chỉ cần chứng minh chúng

bằng nhau trên tập các số nguyên tố. • Với f là hàm nhân tính, nếu m = n = 1 thì (m,n) = 1 suy ra ( ) ( ) ( ),111 fff = do vậy

( ) 01 =f hoặc ( ) 11 =f

Nếu ( ) 01 =f thì với mọi n cũng có (n,1) = 1 ⇒ ( ) ( ) ( )1fnfnf = =0, nên f là hàm ñồng

nhất bằng không, vậy nếu f là hàm nhân tính thì ( ) 11 =f .

• Nếu 21 , ff là các hàm nhân tính, hàm tích f của chúng ñược ñịnh nghĩa bởi:

( ) ( ) ( ) *21 Nnnfnfnf ∈∀= , ký hiệu 21 fff = thì f cũng là hàm nhân tính.

• Ta dễ dàng kiểm tra ñược - Với mỗi *Nn ∈ hàm ( ) n

aaf = với mọi *Na ∈ là một hàm nhân tính. - Hàm δ là hàm nhân tính. - Hàm e là hàm hoàn toàn nhân tính.

ðịnh lý 5. ( tính chất cơ bản của hàm nhân tính) Nếu *,1 Nnn ∈> và k

kpppnααα ...21

21= là sự phân tích tiêu chuẩn của n thì với

mọi hàm nhân tính f ta có

( ) ( ) ( )( ) ( ) ( )( ) ( )1,...1......1|

111 k

kk

nd

pfpfpfpfdfαα ++++++=∑

trong ñó tổng ở vế trái lấy trên tất cả các ước số dương d của n. Chứng minh

Nếu ta khai triển vế phải (1), thì ta sẽ ñược một tổng gồm các số hạng có dạng ( ) ( ) ( )k

kpfpfpfβββ ...21

21 , trong ñó ii αβ ≤≤0 với mọi i = 1,2,3,..,k.

Theo giả thiết f là hàm nhân tính nên ( ) ( ) ( ) ( )kk

kk pppfpfpfpfββββββ ...... 2121

2121 =

nhưng ( )kippp iikk ,...,3,2,1,0...21

21 =≤≤ αββββ chính là một ước d của n và mọi ước d của n

ñều có dạng ñó. Vì vậy vế phải của (1) là tổng của những số hạng có dạng ( )df , trong ñó d chạy khắp chỉ một lần tất các ước dương của n, ñó chính là vế trái của (1) ( ñpcm).

Page 57: Kỷ yếu hội thảo các trường chuyên dh db bắc bộ lần III

=========================================================== 59

HỘI CÁC TRƯỜNG THTP CHUYÊN KHU VỰC DUYÊN HẢI VÀ ðỒNG BẰNG BẮC BỘ

Hội thảo khoa học môn Toán học lần thứ III - 2010

Nhận xét: Vận dụng ñịnh lý trên ñối với hàm nhân tính ( ) naaf = với mỗi *Na ∈ và

k

kpppaααα ...21

21= là sự phân tích tiêu chuẩn của a thì ta có

( ) ( )....1......1 2

|1

211

1 kn

k

n

k

n

k

nd

nnnnppppppd

αα ++++++++=∑

• Nếu n = 0 và ký hiệu d(a) là các số ước dương của a thì ( ) ( )( ) ( ).1...111 21

||

0 +++=== ∑∑ k

adad

dad ααα

• Nếu n = 1 và ký hiệu ( )aσ là tổng các ước dương của a thì ( ) .|∑=

ad

daσ

Do ñó ( ) ( ) ( ) .1

1...

1

1...1......1

1

1

112

1211

1

1

−=++++++++=

++

k

kkkk

p

p

p

pppppppa

k

k

αααασ

ðịnh lý 6. Cho Ra ∈ và Mf ∈ ta có

i) 1*−f tồn tại

ii) MfTa ∈ và nếu f là hàm hoàn toàn nhân tính thì fTa cũng là hàm hoàn toàn nhân

tính.

iii) ( ) .1*1* −−= fTfT aa

Chứng minh.

i) Ta có ( ) 11 =f suy ra 1*−f tồn tại (theo ñịnh lý 3 )

ii) Nếu Ra ∈ và *, Nnm ∈ thỏa mãn (m,n) = 1 thì

( ) ( )( ) ( ) ( ) ( ) ( ).nfTmfTnnfmmfmnmnfmnfT aa

aaa

a ===

vì vậy ta ñược MfTa ∈ .

Nếu f là hàm hoàn toàn nhân tính thì ( ) ( ) ( ) *, Nnmnfmfmnf ∈∀= suy ra

( ) ( )( ) ( ) ( ) ( ) ( ).nfTmfTnnfmmfmnmnfmnfT aa

aaa

a ===

hay fTa là hàm hoàn toàn nhân tính.

iii) Do Mf ∈ theo i) tồn tại 1*−f , vì vậy ta có

( ) ⇒=== −− δδaaaa TffTfTfT1*1* ** ( ) .1*1* −−

= fTfT aa Dựa vào các kết quả trên ta hoàn toàn có thể chứng minh ñịnh lý sau ðịnh lý 7. Cho ., Agf ∈ i) Nếu MgfthìMgf ∈∈ *, ii) Nếu .* MgthìMgfvàf ∈∈

iii) Nếu Mf ∈ thì .1*Mf ∈−

II.Một số hàm số học thường gặp 1. Tổng các ước ðịnh nghĩa 7. Cho n là số nguyên dương, với mỗi số thực α ta gọi hàm ασ là tổng lũy

thừa α của các ước dương của n, nghĩa là ( ) .*

|0

Nndn

ndd

∈∀=∑>

αασ

Khi 1=α ta viết ( ) ∑>

∈∀==

ndd

Nndnn

|0

1 *,)(σσ ( )nσ còn ñược gọi là tổng các ước

dương của n. Khi 0=α ta viết ( ) ∑

>

∈∀==

ndd

Nnndn

|0

0 *,1)(σ trong ñó d(n) là số các ước dương

của n.

Page 58: Kỷ yếu hội thảo các trường chuyên dh db bắc bộ lần III

=========================================================== 60

HỘI CÁC TRƯỜNG THTP CHUYÊN KHU VỰC DUYÊN HẢI VÀ ðỒNG BẰNG BẮC BỘ

Hội thảo khoa học môn Toán học lần thứ III - 2010

ðịnh lý 8. Ta có i) .* ReTe ∈∀= ασ αα

ii) ασ là hàm nhân tính.

iii)Với 1, >∈ nNn và n có phân tích tiêu chuẩn là r

rpppnααα ...21

21= ta có các kết quả sau

• Nếu 0≠α thì ( )( )

∏=

+

−=

r

i i

i

p

pn

1

1

αα

ασ

• Nếu 0=α thì ( ) ( )∏=

+=r

i

in1

1ασ α

Chứng minh

i) Nếu *Nn ∈ thì

( )( ) ( ) ( ) ( ) ( ),*||||

ndddedeTdeTd

neneTe

ndndndnd

ααα

ααα σ====

= ∑∑∑∑

suy ra .* eTe αασ =

ii) Do eTe α, là hàm nhân tính suy ra eTe α* là hàm nhân tính theo ñịnh lý 7, vì

vậy eTe αασ *= là hàm nhân tính.

iii) Vì ασ là hàm nhân tính nên ta chỉ cần tính ( )1αασ ip với ,1 ri ≤≤

Nếu 0=α thì ( ) ( ) .1|

01 ∑==

i

ip

i

k

ii pdpα

αασ Mà i

ipα chỉ có các ước là i

iii pppα,...,,,1 2 do

ñó ( ) ( ) ( )11

0 +== iiiipdp ασ αα

Nếu 0≠α thì ( ) ( )1

1...1

)1(2

0

1

−=++++==

+

=

∑ α

αααααα

ααασ

i

i

iii

t

t

iip

pppppp

i

i

i

Vì vậy ta ñược ( )( )

=+

≠−

=

+

01

01

1)1(

1

αα

ασ α

αα

αα

nêu

nêup

p

p

i

i

i

i

i

Từ ñó ta có kết quả cần chứng minh Ví dụ3: Tính ( ) ( ) ( ) ( ) ( ).10;10;18;18;18 2020 σσσσσ

Giải

( ) ( )

( ) ( )

( )

( )

( ) .13015

15

12

12)5.2(10

4)11)(11()10()5.2(10

45513

13

12

12)3.2(18

.4212

12

13

133.218

6)12)(11(3.2)18(18

2

4

2

4

22

00

2

6

2

42

22

222

200

=−

−==

=++===

=−

−==

=−

−==

=++===

σσ

σσ

σσ

σσ

σσ

d

d

2. Số các ước ðịnh nghĩa 8. Cho k, n là các số nguyên dương, ta gọi hàm dk(n) là cách viết n thành

tích của k nhân tử, trong ñó thứ tự của các nhân tử cũng ñược tính. Nhận xét

Page 59: Kỷ yếu hội thảo các trường chuyên dh db bắc bộ lần III

=========================================================== 61

HỘI CÁC TRƯỜNG THTP CHUYÊN KHU VỰC DUYÊN HẢI VÀ ðỒNG BẰNG BẮC BỘ

Hội thảo khoa học môn Toán học lần thứ III - 2010

• d1(n)= 1; d2(n)= d(n) là số các ước dương của n.

• Theo ñịnh nghĩa thì dk+1(n) là số cách viết n như là tích của (k+1) nhân tử hay là số

cách viết n có dạng =n a1a2…ak+1. Nếu cố ñịnh ak+1|n thì 1+ka

ncó thể viết như là

tích của k nhân tử và có

+1k

ka

nd cách. Vì vậy ta ñược ∑ ≥∀=+

ni

kk kidnd|

1 1)()(

ðịnh lý 9. Cho *Nk ∈ , ta có i) k

k ed*=

ii) dk là hàm nhân tính

iii)Nếu ∏=

=r

i

iipn

1

α là sự phân tích tiêu chuẩn của n thì với mọi 1≥k ta có

( ) ∏=

−−+=

r

i

k

kk iCnd

1

11α

Chứng minh

i) Chứng minh quy nạp theo k. Nếu k = 1 thì d1(n) = 1 = e(n) suy ra d1=e

*1.

Nếu k = 2 thì ∑==nd

ndnd|

2 1)()(

Ta lại có ( ) ,1)()(*)(||

2* ∑∑ =

==

ndnd d

nedeneene .*)()( 2*

2 Nnnendrasuy ∈∀=

Vì vậy ta ñược d2= e*2

. Giả sử với mọi 11 −≤≤ km ta có dm= e

*m.

Khi ñó ta có ( )

( ) ( )( ) )()(*)(

)()()(

*1*

|

1*

|

1*

|1

neneeiei

ne

ieidnd

kk

ni

k

ni

k

ni

kk

==

=

==

−−

−−

∑∑

hay dk = e*k *Nk ∈∀ (ñpcm).

ii)Vì e là hàm nhân tính suy ra dk = e*k là hàm nhân tính (theo ñịnh lý 7).

iii) Sử dụng kết quả: Nếu m,n là các số nguyên không âm thì )1(11

0

+++

=+∑ = m

nm

m

i

n

ni CC

Vì dk là hàm nhân tính nên ta chỉ cần chứng minh ( ) ,11

−−+= k

kk Cpd αα (2) với p nguyên tố,

*N∈α . Thật vậy ta có Nếu k = 1 thì ( ) ,1 1

10

1−

−+=== k

kCCpd ααα suy ra (2) ñúng.

Nếu k = 2 thì ( ) ( ) ,1 11

112

−−++ ==+== k

kCCpdpd αααα α suy ra (2) ñúng.

Giả sử với mọi km ≤≤1 mà ( ) ,11

−−+= m

mm Cpd αα khi ñó ta có

( ) ( ) k

k

i i

k

ki

i

kk CCpdpd += =

−−++ ∑ ∑ === α

α αα

0 0

111 ( theo (1)) nên (2) ñúng với k+1.

Vậy (2) ñúng với mọi *Nk ∈

Vì vậy nếu ∏=

=r

i

iipn

1

α là sự phân tích tiêu chuẩn của n thì ( ) ∏=

−−+=

r

i

k

kk iCnd

1

11α

3. Hàm Ơle ( )nϕ

Page 60: Kỷ yếu hội thảo các trường chuyên dh db bắc bộ lần III

=========================================================== 62

HỘI CÁC TRƯỜNG THTP CHUYÊN KHU VỰC DUYÊN HẢI VÀ ðỒNG BẰNG BẮC BỘ

Hội thảo khoa học môn Toán học lần thứ III - 2010

ðịnh nghĩa 9. Cho *Nn ∈ , ta gọi hàm ( )nϕ là số các số nguyên dương nguyên tố với n

và nhỏ hơn n, tức là ( ) .1

1),(1∑

=≤≤

=

ndnd

ðịnh lý 10. Ta có

i) ( ) .)(|∑=

nk k

nkn µϕ

ii) .* 1eTµϕ = iii) ϕ là hàm nhân tính

iv) Nếu n là số nguyên dương thì ( ) ∏

−=

np pnn

|

11ϕ .

Chứng minh

i) Từ nhận xét của ñịnh nghĩa 5 ta có với mọi n>1 thì

( ) ⇒=∑ 0|nd

dµ ( ) ( )∑∑∑∑∑==

=≤≤

===n

dnkdkndk

n

dnd

nd

kkn1

||),|(1

1),(1

.)(11 µµϕ

Cố ñịnh k là là ước của n, ta phải lấy tổng với những giá trị của d thỏa mãn nd ≤≤1 mà

d chia hết cho k. Nếu d=qk thì do nd ≤≤1 suy ra k

nq ≤≤1 và

( ) ( ) ( )∑ ∑∑∑∑==

===kn

q nknknk

kn

q k

nkkkn

/

1 |||

/

1

.1)( µµµϕ (ñpcm)

ii). Theo i) ta có

( ) ( ) ( ) ( ) ( )⇒=

=

== ∑∑∑ eT

k

neTk

k

n

k

nek

k

nkn

nknknk

11|||

*µµµµϕ eT1*µϕ = .

iii). Theo ii) ta có eT1*µϕ = . Mà eT1,µ là hàm nhân tính nên ϕ là hàm nhân tính.

iv) Vì ϕ là hàm nhân tính ñể xác ñịnh ( )nϕ ta chỉ cần tính giá trị của ( ),αϕ p với *N∈α và p nguyên tố. Ta có

( ) ( ) ( ) ( ) ( ) ∏∑∑ −=⇒

−=−=+=== −−

=

npi

ii

pk pnn

pppppppppp

k

pkp

i \

110

0|

).1

1()(1

1 ϕµµµµϕ αααααα

αα

α

III. Một số bài tập về hàm số học Bài 1. Cho *Nn ∈ , ta có a) ∑ =

nd

nd|

.)(ϕ ( Hệ thức Gauss)

b) ( )∑ =

nd

ni

ndi

|

.)( σϕ

Giải

a) Do eT1*µϕ = và e(n)= 1 với mọi *Nn ∈ nên ta có

( ) ( )

.))(()(

)(**)(*)()(

1*1

|

1||

−===

==

=

∑∑

evìnneTd

neTened

nedd

nd

ndnd

µϕ

µϕϕϕ

b) Ta có ( )( ) ( )( ) ( )( ) ( )∑ ====

nd

nneeTneeeTndi

ndi

|11 .*****)( σµϕϕ (theo ñịnh lý

8)

Page 61: Kỷ yếu hội thảo các trường chuyên dh db bắc bộ lần III

=========================================================== 63

HỘI CÁC TRƯỜNG THTP CHUYÊN KHU VỰC DUYÊN HẢI VÀ ðỒNG BẰNG BẮC BỘ

Hội thảo khoa học môn Toán học lần thứ III - 2010

Nếu n là số nguyên tố

nếu tồn tại số nguyên tố p sao cho p2|n,

nếu n có phân tích tiêu chuẩn là n = p1p2…pr,

r ≥ 2

nếu n là số nguyên tố

nếu tồn tại số nguyên tố p sao cho p2|n,

nếu n có phân tích tiêu chuẩn là n = p1p2…pr,

r ≥ 2

Bài 2. ðặt ( ) ( )∏ ∈∀=nd

Nndn|

*µφ . Chứng minh rằng

( )

=

1

0

1

Giải

Sử dụng ñịnh lý 4 về công thức tính ( )nµ , ta xét các trường hợp sau

• Nếu n là số nguyên tố thì ( ) ( ) ( ) 1)(1|

−=== ∏ ndnnd

µµµφ

• Nếu tồn tại số nguyên tố p sao cho p2|n, thì mpn

α=

với ( ) 1,,2 =∈≥ pmvàNαα suy ra

( ) ( ) ( ) 0)(||

=== ∏∏≠ α

µµµφ α

pd

ndnd

ppdn

• Nếu n có phân tích tiêu chuẩn là n = p1p2…pr,( r ≥ 2) thì

( ) ( ) ( ) ∏∏∏∏≤<<≤≤<≤= −

==riii

riii

rii

ii

r

i

i

nd r

rpppppppppdn

121

21

21

21...1

2111|

)...()...()...()()(1 µµµµµµµφ

( ) ( ) ( ) ( ) ( ) ( )

( ) ( ) )...2(

)1(32

21

1321

1

11...111rrrr

rr

rrrrr

rCCC

rCCrCCC

n

n

+++

−=

−−−−−=−

φ

φ

Mà ( ) ( ) 1212112...2

−−−==+++

rrrr

rrrnrasuyrrCCC φ

Vì 2≥r nên ( ) 11 ≥−r . Ta có 22 1 ≥−r hay ( ) 1=nφ Do ñó trong mọi trường hợp ta ñều có

( )

=

1

0

1

Bài 3 Cho *, NnMf ∈∈

a) Nếu n lẻ, chứng minh rằng ( ) ( ) ( ).1||

/ ∑∑ =−ndnd

dndfdf

b) Nếu n chẵn, n = 2sm với 1≥s , m lẻ, chứng minh

rằng ( ) ( ) ( ) ( ) ( ).221|||

/ ∑∑∑ −=−mk

s

ndnd

dnkffdfdf

c)Tính giá trị của tổng S = ( )( ) ( )∑ −−nd

dndf

|

/ .11

Giải

a)Nếu n lẻ, d|n suy ra d lẻ và n|d cũng lẻ. Khi ñó ta có ( ) 11 |−=−

dn và

( ) ( ) ( )∑ ∑∑ −=−=−nd ndnd

dndfdfdf

| ||

| )()(1 (ñpcm).

b)Nếu n chẵn, n = 2sm với 1≥s và m lẻ thì có

Với d|n suy ra d có dạng 1,...,2,1,2 −== simdi hoặc ,2 kd

i= trong ñó i = 0,1,…,s và k|m.

Ta có

Page 62: Kỷ yếu hội thảo các trường chuyên dh db bắc bộ lần III

=========================================================== 64

HỘI CÁC TRƯỜNG THTP CHUYÊN KHU VỰC DUYÊN HẢI VÀ ðỒNG BẰNG BẮC BỘ

Hội thảo khoa học môn Toán học lần thứ III - 2010

( ) ( ) ( ) ( ) ( ) ( )

( ) ( ) ( )( )

( )

( ) ( ) ( ) ( )

( ) ( ) ( )( )adungápkfkfmf

kfkfmf

kfmf

kfmfdf

mk

ss

i mk

is

i

i

mk

skms

i mk

is

i

i

s

i mk

is

i

i

s

i mk

ikns

i

imn

nd

dn

k

mis

is

ii

∑∑∑∑

∑∑∑∑

∑∑∑

∑∑∑∑

−+=

−++=

−+−=

−+−=−

=

=

=

=

=

=

=

=

−−

|

1

0 |

1

1

|

/1

0 |

1

1

0 |

21

1

2

0 |

2|1

1

2/

|

/

222

2122

2121

21211

( ) ( ) ( )

( ) ( ) ( )∑∑

∑ ∑∑∑

=−=

−+==

=

mk

ss

nd

s

i mk

s

mk

is

i

i

kvìkffdf

kfkfmf

||

0 ||

1

1

,1,2)(22

2222

suy ra ( ) ( ) ( ) ( )∑∑∑ −=−mk

s

ndnd

dnkffdfdf

|||

/ .22)(1 (ñpcm)

c) Nếu n lẻ theo câu a) ta có ( )( ) ( ) ( ).211| |

/∑ ∑=−−=nd nd

dndfdfS

Nếu n chẵn, n = 2sm, s ≥ 1,m lẻ thì theo câu b) ta ñược

( ) ( )∑∑∑ =−−=mk

s

nd

dn

nd

kffdfdfS||

/

|

)(22)(1)(

Do ñó ta có

=∑

mk

s

nd

kff

df

S

|

|

)()2(2

)(2

Nhận xét: Từ bài toán trên ta có các kết quả sau 1. Nếu ϕ=f , sử dụng hệ thức Gauss (bài 1) ta có

( ) ( )−

=−∑ 01

|

/ nd

nd

dnϕ

2. Nếu ( ) *Nnnnf ∈∀= thì

( )

=−∑ ∑

∑∑ +

nd mk

s

nd

nd

dn

kd

d

d

| |

1

|

|

/

21

Bài 4. Chứng minh các ñẳng thức sau ( ) ( )

( )( )

( ) .*)()

.*)(*)()

.*)(*)()

*/)(/)()

2

||

3

23

2

22

|

2

|

2

Nnkdkdd

Nnndndc

Nnndndb

Nnkndkknkda

nknk

nknk

∈∀

=

∈∀=

∈∀=

∈∀=

∑∑

∑∑

µ

µ

µµ

Giải

a)Theo ñịnh nghĩa tích chập, ñẳng thức cần chứng minh tương ñương với việc chứng minh

n�u n l�

n�u n ch�n, n = 2sm, s ≥1,m l�

nếu n lẻ

nếu n chẵn

nếu n lẻ

nếu n chẵn, n = 2sm, s ≥ 1,m lẻ

Page 63: Kỷ yếu hội thảo các trường chuyên dh db bắc bộ lần III

=========================================================== 65

HỘI CÁC TRƯỜNG THTP CHUYÊN KHU VỰC DUYÊN HẢI VÀ ðỒNG BẰNG BẮC BỘ

Hội thảo khoa học môn Toán học lần thứ III - 2010

( )( ) ( )( )ndnd ** 22 µµ = (1)

Do µ,d là các hàm nhân tính nên dvàdd **,, 2222 µµµ cũng là hàm nhân tính. Vì

vậy ñể chứng minh (1) ta chỉ cần chứng minh ( )( ) ( )( ),** 22 αα µµ pdpd = với p nguyên tố và 1≥α

Thật vậy ta có

( )( ) ( ) ( )

( ) ( ) ( ) ( )

( ) ( ) 12111

1

*

22

122

0

22

+=+−−+=

+=

=

=

−∑

ααα

µµ

µµ

αα

ααα

ppdpd

ppdpdi

ii

( )( ) ( ) ( )

( ) ( ) ( ) ( )( ) ( ) 12111

1

*

122

0

22

+=+−−+=

+=

=

=

−∑

ααα

µµ

µµ

αα

ααα

pdppd

pdppdi

ii

á

suy ra ( )( ) ( )( ).** 22 αα µµ pdpd = (ñpcm)

b) Giả sử r

rpppnααα ...21

21= là sự phân tích tiêu chuẩn của n.

Khi ñó r

rpppnααα 22

221

2 ...21= và ( ) ( )∏=

+=r

i

ind1

2 12α

Mặt khác theo câu a) ta có ( )( ) ( )( ) ∏∏==

+==r

i

i

r

i

ipdnd11

22 )12(** 1 αµµ α ⇒

( ) )(*)( 22ndnd µ= (ñpcm)

c) Do 23

2 *, µdd là hàm nhân tính. ðể chứng minh ñẳng thức ñã cho ta chỉ cần chứng

minh ( )( )αα µ pdpd2

32 *)( = với mọi p nguyên tố và 1≥α .

( )

( ) ( ) ( ) ( ) ( ) ( ) ( )

( ) ,112

1)*(

.1)(

2221

22

23

23

0

23

23

22

+=++=+=

+==

+=

++

=

−∑

ααα

µµµµ

α

αα

ααα

αα

α

CC

ppdpdppdpd

pd

i

i

ii

suy ra ( )( )αα µ pdpd2

32 *)( = ⇒ ( ) .*)(*)( 2

32

Nnndnd ∈∀= µ

d) Ta có

( ) ( ) )(*/)()( 3

|

3

|

3nedknekdkd

nknk

==∑∑

( )( )( ) ( ) ( )nednedknekdkdnknk

22

2

|

2

|

**)/()()( ==

=

∑∑

Do ñó ñẳng thức cần chứng minh tương ñương với phải chứng minh

( )( ) ( ) ( ) *** 23Nnnedned ∈∀=

hay ( )( ) ( ) ( ),** 23 ααpedped = với mọi p nguyên tố và 1≥α .

Thật vậy ta có

Page 64: Kỷ yếu hội thảo các trường chuyên dh db bắc bộ lần III

=========================================================== 66

HỘI CÁC TRƯỜNG THTP CHUYÊN KHU VỰC DUYÊN HẢI VÀ ðỒNG BẰNG BẮC BỘ

Hội thảo khoa học môn Toán học lần thứ III - 2010

( )( ) ( ) ( ) ( ) ( )

( ) ( ) ( )

( )( ) ( ) ( ) ( )

( )( ) ( )( ) ( ) ( )4

21

4

211...21

1*

4

211...21

1*

2222

2

0

2

0

22333

0

3

0

3

0

33

++=

++=++++=

+=

=

++=++++=

+===

∑∑

∑∑∑

==

===

ααααα

ααα

αααα

ααααα

ii

ii

ii

i

i

ii

ipepdped

ipdpepdped

⇒ ( )( ) ( ) ( ),** 23 ααpedped = vậy ta có ñpcm ( )

2

||

3 )(

= ∑∑

nknk

kdkd

Bài tập tự luyện Bài 1. Cho Mf ∈ , chứng minh rằng

a) f là hàm hoàn toàn nhân tính khi và chỉ khi .* 1*fff µ=−

b) ( ) ( ) ( )( ) ppfdfdnpnd

,1||

∏∑ −=µ nguyên tố.

c) ( ) ( ) ( )( ) ppfdfdnpnd

,1||

2 ∏∑ −=µ nguyên tố.

d).Nếu n là số nguyên dương, ký hiệu ( )nw là số các ước nguyên tố phân biệt của n thì

( ) ( )nw

nd

d 2|

2 =∑µ

Bài 2. Gọi )(0nσ là kí hiệu tổng các ước lẻ của số nguyên dương n . Chứng minh rằng

a) .)1()(/

||

0dn

dn

nd

∑ −−=σ

b) ),2/(2)()(0nnn σσσ −= với n là số chẵn.

c) )(0nσ là hàm nhân tính.

Bài 3. Chứng minh rằng

a) .*,2)( Nnnnd ∈∀≤

b) .2,)()(2 ≥∈∀+≤ nNnnnn ϕσ

c) .*,2)()( 2/)1(NnNknndnnn

kk

k

k ∈∈∀≤≤≤ +σ

d) .*),()( Nnndnn ∈∀≤ ϕ

Page 65: Kỷ yếu hội thảo các trường chuyên dh db bắc bộ lần III

=========================================================== 67

HỘI CÁC TRƯỜNG THTP CHUYÊN KHU VỰC DUYÊN HẢI VÀ ðỒNG BẰNG BẮC BỘ

Hội thảo khoa học môn Toán học lần thứ III - 2010

MỘT SỐ BÀI TOÁN SỐ HỌC TRONG CÁC KÌ THI OLYMPIC TOÁN

Trần Xuân ðáng

(THPT Chuyên Lê Hồng Phong – Nam ðịnh)

Trong kỳ thi Olympic toán Quốc tế lần thứ 49 ñược tổ chức tại Tây Ban Nha có bài

toán sau (bài toán 1) mà tác giả của nó là Kestutis Cesnavicius (Lithuania) (Litva).

Bài toán 1: Chứng minh rằng tồn tại vô số số nguyên dương n sao cho 2 1n + có ước

nguyên tố lớn hơn 2 2n n+

Bài toán này là bài toán khó nhất của ngày thi thứ nhất. Lời giải của bài toán 1 ñược

phát triển từ lời giải của các bài toán ñơn giản hơn sau ñây:

Bài toán 2: Chứng minh rằng tồn tại vô số số nguyên dương n sao cho n2 + 1 không là

ước của n!.

(ðề thi chọn ñội tuyển của Inñônêxia dự thi Toán Quốc tế năm 2009) .

Lời giải của bài toán 2:

Bổ ñề: Tồn tại vô số số nguyên tố dạng 4k + 1 (k ∈ N*)

Chứng minh: Gọi A là tập hợp gồm tất các số nguyên tố dạng 4k+1 (k ∈N*) ,

Khi ñó A ≠ rỗng vì 5 ∈ A. Giả sử A là tập hữu hạn. Gọi p0 là phân tử lớn nhất của A ⇒p0

≥ 5 .

Giả sử p1, p2 … pn là tất cả các số nguyên tố nhỏ hơn p0.

ñặt 2 2 2

0 14 ... 1n

a p p p= + khi ñ? a ∈ N*, a > 1. Giả sử q là ước nguyên tố của a

⇒q ≠ pi , ∀ i ∈ {0,1,2 …, n}. Mặt khác (2p0p1… pn) 2 + 1 ≡ 0 (modq)

⇒ - 1 là số chính phương (modq) và q lẻ.

Suy ra qq

q

q

⇒−

⇒=−⇒=

−−

2:2

11)1(1

1 2

1

⇒≡ )4(mod1 q có dạng 4k + 1 (k ∈ N*).

Mặt khác q> p0. ðiều này mâu thuẫn với cách chọn p0. Vậy tồn tại vô số số nguyên tố dạng

4k + 1 (k∈ N*).

Chúng ta chuyển sang việc giải bài toán 2. Giả sử p là số nguyên tố dạng 4k + 1 (k

∈ N*) ⇒ 11)1(1 2

1

−⇒=−=

−−p

p là số chính phương (modp)

⇒ ∃ np ∈ { 0,1,2 …. ,p - 1} sao cho 2pn ⇒−≡ )(mod1 p

2pn +1: p và np! không chia hết

cho p → np ! không chia hết cho 2pn + 1. Ta có: 2

pn + 1 ≥ p ⇒np ≥ 1−p . Vì tồn tại vô

Page 66: Kỷ yếu hội thảo các trường chuyên dh db bắc bộ lần III

=========================================================== 68

HỘI CÁC TRƯỜNG THTP CHUYÊN KHU VỰC DUYÊN HẢI VÀ ðỒNG BẰNG BẮC BỘ

Hội thảo khoa học môn Toán học lần thứ III - 2010

số số nguyên tố p dạng 4k + 1 (k∈ N*) nên tồn tại vô số số nguyên dương n sao cho n2 + 1

không là ước của n!

Bài toán 3: Chứng minh rằng tồn tại vô số số nguyên dương n sao cho ước nguyên tố

lớn nhất của n2 + 1 lớn hơn 2n

(Tạp chí Animath của Pháp năm 2006)

Lời giải của bài toán 3: Giả sử p là số nguyên tố dạng 4k + 1 (k ∈ N*)

Suy ra 11)1(1 2

1

−⇒=−=

−−p

p là số chính phương (modp)

⇒ ∃ x ∈ {0,1,2, … ,p - 1} sao cho x2 ≡ - 1(modp).

Ta có: q2 ≡ (p- q)2 (modp) (q ∈ Z)

⇒ ∃ q ∈ {0,1,2, …, 2

1−p} sao cho q2 ≡ -1 (modp).

Thật vậy giả sử 2

1−p < x < p ⇒ x ≥

2

1+p. ðặt q = p – x, ta có:

q2 = ( p – x)2 ≡ x2 ≡ - 1 (modp) và 0 < q ≤2

1−p. Ta có: q2 +1 M p và

p ≥ 2q +1 > 2q. Suy ra ước nguyên tố lớn nhất của q2 +1 lớn hơn 2q.Vì có vô số số

nguyên tố dạng 4k + 1(k ∈ N*) nên tồn tại vô số số nguyên dương n sao cho n2 +1 có ước

nguyên tố lớn hơn 2n.

Sau ñây là các lời giải của bài toán 1

Lời giải thứ nhất của bài toán 1: Xét số nguyên tố p dạng 4k + 1 (k ∈ N*)

⇒ 11)1(1 2

1

−⇒=−=

−−p

p là số chính phương (modp)

⇒ ∃ x ∈ {0,1,2, … p - 1} sao cho x2 ≡ - 1(modp).

Vì x2 ≡ (p- x)2 (modp) (x ∈ Z) ⇒ ∃ x ∈ {0,1,2 … ,2

1−p} sao cho

x2 ≡ -1 (modp). ⇒ ∃ α ∈ {0,1,2, … ,2

1−p} sao cho

−α

2

1p 2 ≡ -1 (modp)

ðặt m =

−α

2

1p ⇒m ∈ {0,1,2, …,

2

1−p} và m2 ≡ -1 (modp)

Giả sử p > 20. N?u 0 ≤ α ≤ ⇒−+

4

314 p0 < 2α +1 ≤ ⇒

−+

2

114 p

(2α +1)2 <p - 4 Vì m2 ≡ -1 (modp) nên 4m2 ≡ -4 (modp)

Mặt khác 4m2 = (p – 1 -2α )2 ≡ (2α +1)2 (modp) ⇒ (2α +1)2 ≡ -4 (modp)

Page 67: Kỷ yếu hội thảo các trường chuyên dh db bắc bộ lần III

=========================================================== 69

HỘI CÁC TRƯỜNG THTP CHUYÊN KHU VỰC DUYÊN HẢI VÀ ðỒNG BẰNG BẮC BỘ

Hội thảo khoa học môn Toán học lần thứ III - 2010

ðiều ñó là ñiều vô lý vì 0 < (2α +1)2 < p – 4

Vậy α > ⇒−+

4

314 p p > 2m + m2 . Vì m2 +1 M p nên m2 ≥ p -1 ⇒m ≥

1−p . Vì tồn tại vô số số nguyên tố p dạng 4k + 1 (k ∈ N*) nên tồn tại vô số số nguyên

dương n sao cho ước nguyên tố lớn nhất của n2 + 1 lớn hơn 2 2n n+ .

Lời giải thứ 2 của bài toán 1: Giả sử n là số nguyên, n ≥ 24. Giả sử p là ước nguyên

tố của (n!)2 + 1. Hiển nhiên p > n. Giả sử x ∈(0, 2

p) là số dư trong phép chia n ! hoặc – n!

cho p. Khi ñ? 0 < x< p – x < p. Ta c? x2 + 1 chia hết cho p. Thật vậy tồn tại m∈ Z sao cho n!

= mp + x hoặc – n! = mp + x. Trong cả hai trường hợp ta ñều có (n!)2+1 = (mp+x)2 +1 ⇒ x2

+1 = (n!)2 + 1 – m2p2 – 2mpx ⇒x2+1Mp . Từ ñó suy ra p là ước của p2 - 2px + 4x2 + 4 = (p –

2x)2 + 4

⇒p ≤ (p – 2x)2 + 4 ⇒ p ≥ 2x + 4−p

⇒p - 4 ≥ 2x + 4−p - 4 ≥ 2x + 20 – 4> 2x

⇒ p ≥ 2x + 4−p > 2x + x2 Từ ñây suy ra ñiều phải chứng minh

Bài toán sau là bài toán tổng quát của bài toán 1

Bài toán 4: Chứng minh rằng tồn tại vô số số nguyên dương n sao cho n2 + 1 có ước

nguyên tố lớn hơn 2n + 2 n

Bài toán 5: Chứng minh rằng với mỗi số nguyên n ≥ 3, tồn tại cặp số nguyên dương

lẻ (xn, yn) sao cho n

nn yx 27 22 =+

(ðề thi Olympic Toán của Bungari năm 1996)

Lời giải: Với n = 3 , chọn x3 = y3 = 1

Giả sử với n ≥ 3 , tồn tại cặp số nguyên dương lẻ (xn,yn) sao cho n

nn yx 27 22 =+ . Ta

chứng minh rằng mỗi cặp .

(X=2

7,

2nnnn

yxY

yx −=

+) , (X=

2

7,

2nnnn yx

Yyx +

=−

) thoả măn 122 27 +=+ nYX

Thật vậy 22

2

7

27

+

± nnnn yxyx m= 2 ( 227 nn yx + ) = 2. 2n = 2n+1

Vì xn , yn lẻ nên xn = 2k + 1, yn = 2l + 1 (k, l ∈ Z) 12

++=+

⇒ lkyx nn

Page 68: Kỷ yếu hội thảo các trường chuyên dh db bắc bộ lần III

=========================================================== 70

HỘI CÁC TRƯỜNG THTP CHUYÊN KHU VỰC DUYÊN HẢI VÀ ðỒNG BẰNG BẮC BỘ

Hội thảo khoa học môn Toán học lần thứ III - 2010

và lkyx nn −=

2. ðiều ñó chứng tỏ rằng một trong các số

2,

2nnnn

yxyx −+ là lẻ .

Vì vậy với n +1 tồn tại các số tự nhiên lẻ xn+1 và yn+1 thoả măn n

nn yx 27 21

21 =+ ++

+1

Bài toán 6 : Chứng minh rằng với mỗi số nguyên dương n, phương trình x2 + 15y2 =

4n có ít nhất n nghiệm tự nhiên (x,y)

(ðề thi chọn học sinh giỏi Toán Quốc gia năm học 2009 – 2010)

Lời giải: Trước hết ta chứng minh rằng với mỗi số nguyên n ≥ 2 tồn tại cặp số nguyên

dương lẻ (xn , yn) sao cho sao cho n

nn yx 415 22 =+

Thật vậy với n = 2 , chọn x2 = 1 , y2 = 1

Giả sử với n ≥ 2 tồn tại cặp số nguyên dương lẻ (xn , yn) sao cho sao cho

n

nn yx 415 22 =+ . Ta chứng minh rằng mỗi cặp

(X= 2

,2

15nnnn yx

Yxy +

=−

), (X=2

,2

15 nnnnxy

Yxy −

=+

) thoả măn

122 415 +=+ nYX

Thật vậy 22

215

2

15

±+

nnnn xyxy m= 4 ( 22 15 nn yx + ) = 4. 4n = 4n+1

Và xn , yn lẻ nên xn = 2k + 1, yn = 2l + 1 (k, l ∈ Z) 12

++=+

⇒ lkyx nn

và klklxy nn −=

+−+=

2

)12()12(

2. ðiều ñó chứng tỏ rằng một trong các số

2,

2nnnn

xyyx −+ là lẻ . Vì vậy với n +1 tồn tại các số tự nhiên lẻ xn+1 và yn+1 thoả măn

121

21 415 +

++ =+ n

nn yx

Trở lại bài toán 6:

Với n = 1, phương tŕnh nyx 415 22 =+ có 1 nghiệm tự nhiên là (x,y) = (2,0)

Với n = 2, phương tŕnh nyx 415 22 =+ có 2 nghiệm tự nhiên là (x,y)= (4,0); (1,1)

Giả sử với n ≥ 2, phương tŕnh nyx 415 22 =+ có n nghiệm tự nhiên là (x1,y1), (x2,y2), …,

(xn, yn) khi ñó (x,y) = ( 2xk, 2yk) (1 ≤ k ≤ n) là các nghiệm tự nhiên của phương trình

nyx 415 22 =+ +1. Theo chứng minh trên phương trình nyx 415 22 =+ +1 lại có 1 nghiệm tự

nhiên lẻ. Vậy phương tŕnh nyx 415 22 =+ +1 có ít nhất n+1 nghiệm tự nhiên. Bài toán 6 ñă

ñược giải quyết.

Page 69: Kỷ yếu hội thảo các trường chuyên dh db bắc bộ lần III

=========================================================== 71

HỘI CÁC TRƯỜNG THTP CHUYÊN KHU VỰC DUYÊN HẢI VÀ ðỒNG BẰNG BẮC BỘ

Hội thảo khoa học môn Toán học lần thứ III - 2010

Bài toán 7: Tìm tất cả các cặp số nguyên dương (x,y) sao cho yx

yx

+ 22

là số nguyên

và là ước của 1995.

(ðề thi Olympic toán Bungari năm 1995)

Lời giải : Trước hết ta chứng minh

Bổ ñề: Cho số nguyên tố p = 4q + 3 (q ∈ N). Giả sử x, y là các số nguyên sao cho x2 +

y2 chia hết cho p, Khi ñó x và y chia hết cho p. Thật vậy nếu x Mp thì y Mp .

Giả sử x không chia hết p ⇒y không chia hết cho p

Theo ñịnh lý nhỏ Phecma ta có x p-1 ≡ 1 (modp) ⇒x4q+2 ≡ 1 (modp). Tương tự y4q+2

≡ 1 (modp) . Ta có x2 + y2Mp ⇒x2 ≡ -y2 (modp)

⇒ (x2)2q+1 ≡ (-y2)2q+1 (modp) ⇒ x4q+2 ≡ - y4q+2 (modp) ⇒ 1 ≡ - 1 ( modp) ⇒ p = 2

(vô lí). Bổ ñề ñã ñược chứng minh.

Áp dụng bổ ñề vào bài toán 7: Giả sử tồn tại các số nguyên dương x,y sao cho x> y ,

yx

yx

+ 22

là số nguyên và yx

yx

+ 22

là ước của 1995 . ðặt k = yx

yx

+ 22

thì x2 +y2 = k( x –y) và

k là ước của 1995 = 3.5.7.19.N?u k M3 th7 k= 3 k1 (k1 ∈N*) (k1 không chia hết cho 3) ⇒ x2

+ y2 M3 ⇒ x M3 và y M3 ⇒x = 3x1 , y = 3y1 (x1 , y1 ∈N*, x1 > y1) ⇒ )( 11121

21 yxkyx −=+ .

N?u k = 1 thì x2 + y2 = x – y . ðó là ñiều vô lí vì x2 + y2 ≥ x + y > x – y (vì x,y ≥ 1 )

Nếu k = 5 thì x2 + y2 = 5(x – y) ⇒ (2x - 5)2 + (2y +5)2 = 50 ⇒ x = 3 , y = 1 hoặc x = 2

, y = 1

Nếu k = 7 , tương tự như trên, tồn tại k2∈N* sao cho k = 7 k2 (k2 không chia hết cho

7) x = 7x2 , y = 7y2 (x2, y2 ∈N* , x2 > y2) và )( 22222

22 yxkyx −=+

Nếu k M19 thì tồn tại k3 ∈N* sao cho k = 19k3 (k3 không chia hết cho 19 ), x = 19x3 ,

y = 19y3 (x3, y3 ∈N* , x3 > y3 ) và )( 33323

23 yxkyx −=+

Vậy tất cả các cặp số nguyên dương (x,y) cần tìm có dạng (3c, c), (2c, c), (c, 2c), (c,

3c) trong ñó c ∈ {1,3,7,19,21,57,133,399} .

Bài toán 8: Tìm tất cả các cặp số nguyên dương (x,y) sao cho số A =

yx

yx

+ 22

là số nguyên và là ước của 2010.

(ðề thi Olympic Toán khu vực duyên hải ñồng bằng Bắc Bộ năm học 2009 – 2010)

Lời giải: Trên cơ sở lời giải của bài toán 7 ta chỉ cần tìm các nghiệm nguyên dương

của các phương trình : )(22 yxkyx −=+ với k ∈ { 2,5, 10}. Phương tŕnh x2 + y2 = 2 (x- y)

không có nghiệm nguyên dương . Thật vậy giả sử x,y ∈N* , x > y và x2 + y2 = 2 (x- y) ⇒

Page 70: Kỷ yếu hội thảo các trường chuyên dh db bắc bộ lần III

=========================================================== 72

HỘI CÁC TRƯỜNG THTP CHUYÊN KHU VỰC DUYÊN HẢI VÀ ðỒNG BẰNG BẮC BỘ

Hội thảo khoa học môn Toán học lần thứ III - 2010

x2 + y2 ≥ 2x +y2 > 2(x – y). ðó là ñiều vô lý . Phương trình x2 + y2 = 5 (x- y) có các

nghiệm nguyên dương là (x,y) = (3,1), (2,1). Phương trình x2 + y2 = 10 (x- y) ⇔ (x-5)2 +

(y+5)2 = 50 có các nghiệm nguyên dương là (x,y) = (6,2) ; (4,2) .

Vậy tất cả các cặp số nguyên dương (x, y) thoả măn ñề bài là (3c, c), (2c, c), (c, 2c) ,

(c, 3c) , (6c, 2c) , (4c, 2c) , ( 2c, 6c), (2c, 4c) trong ñó

c ∈ {1,3,6,7,201}

Cuối cùng là một số bài toán dành ñể luyện tập

Bài toán 9: Chứng minh rằng với mỗi số nguyên dương n, phương tŕnh

7x2 + y2 = 2n+2 luôn có nghiệm nguyên dương.

Bài toán 10: Chứng minh rằng với mỗi số nguyên dương n, phương trình x2 + 15y2 =

4n có ñúng n nghiệm tự nhiên .

Bài toán 11: Cho số nguyên dương n. Gọi Sn là tổng các bình phương của các hệ số

của ña thức f(x) = (1+x)n.

Chứng minh rằng S2n+1 không chia hết cho 3

(ðề thi chọn ñội tuyển Việt Nam dự thi Olympic Toán Quốc tế năm 2010)

Bài toán 12: Chứng minh rằng tồn tại vô số số nguyên dương n sao cho

2n +2 chia hết cho n .

Bài toán 13: Chứng minh rằng tồn tại vô số số nguyên dương n sao cho tất cả các ước

nguyên tố của n2 + n + 1 không lớn hơn n .

(ðề thi chọn ñội tuyển Ukraina dự thi Olympic toán quốc tế năm 2007)

Bài toán 14: Với mỗi số nguyên dương n > 1, kí hiệu p(n) là ước nguyên tố lớn nhất

của n. Chứng minh rằng tồn tại vô số số nguyên n > 1 sao cho:

p(n) < p(n+1) < p(n+2) .

Bài toán 15: Cho các số nguyên a,b thoả măn a>b > 0 . Chứng minh rằng tồn tại vô

số số nguyên dương n sao cho an + bn chia hết cho n .

Bài toán 16: Chứng minh rằng tồn tại vô số số nguyên tố p có tính chất sau: Tồn tại

vô số nguyên dương n sao cho p – 1 không chia hết cho n và n! +1 chia hết cho p.

(ðề thi chọn ñội tuyển của Mônñôva dự thi Olympic toán Quốc tế năm 2007).

Bài toán 17: Chứng minh rằng tồn tại vô số số nguyên dương n sao cho 5n-2 – 1

chia hết cho n.

(ðề thi Olympic toán của Braxin năm 2008)

Page 71: Kỷ yếu hội thảo các trường chuyên dh db bắc bộ lần III

=========================================================== 73

HỘI CÁC TRƯỜNG THTP CHUYÊN KHU VỰC DUYÊN HẢI VÀ ðỒNG BẰNG BẮC BỘ

Hội thảo khoa học môn Toán học lần thứ III - 2010

ðỊNH LÍ LAGRANGE VÀ ỨNG DỤNG

ðặng ðình Sơn

Chuyên Lương Văn Tụy – Ninh Bình

1. ðỊNH LÍ LAGRANGE

1.1. ðỊNH LÍ ROLLE

ðịnh lí: Nếu ( )f x là hàm liên tục trên ñoạn [ ; ]a b , có ñạo hàm trên khoảng ( ; )a b

và ( ) ( )f a f b= thì tồn tại ( ; )c a b∈ sao cho '( ) 0f c = .

Chứng minh:

Vì ( )f x liên tục trên [a; b] nên theo ñịnh lí Weierstrass ( )f x nhận giá trị lớn nhất

M và giá trị nhỏ nhất m trên [a; b].

- Khi M = m ta có ( )f x là hàm hằng trên [a; b], do ñó với mọi ( ; )c a b∈ luôn có

'( ) 0f c = .

- Khi M > m, vì ( ) ( )f a f b= nên tồn tại c (a; b)∈ sao cho ( )f c m= hoặc

( )f c M= , theo bổ ñề Fermat suy ra '( ) 0f c = .

Hệ quả 1: Nếu hàm số ( )f x có ñạo hàm trên (a; b) và ( )f x có n nghiệm (n là số

nguyên dương lớn hơn 1) trên (a; b) thì '( )f x có ít nhất n - 1 nghiệm trên (a; b).

Hệ quả 2: Nếu hàm số ( )f x có ñạo hàm trên (a; b) và '( )f x vô nghiệm trên (a;

b) thì ( )f x có nhiều nhất 1 nghiệm trên (a; b).

Hệ quả 3: Nếu ( )f x có ñạo hàm trên (a; b) và '( )f x có nhiều nhất n nghiệm (n là

số nguyên dương) trên (a; b) thì ( )f x có nhiều nhất n + 1 nghiệm trên (a; b).

Các hệ quả trên ñược suy ra trực tiếp từ ñịnh lí Rolle và nó vẫn ñúng nếu các nghiệm

là nghiệm bội (khi ( )f x là ña thức).

Các hệ quả trên cho ta ý tưởng về việc chứng minh tồn tại nghiệm cũng như xác

ñịnh số nghiệm của phương trình, và nếu như bằng một cách nào ñó ta tìm ñược tất cả các

nghiệm của phương trình (có thể do mò mẫm) thì nghĩa là khi ñó phương trình ñã ñược giải.

Từ ñịnh lí Rolle cho phép ta chứng minh ñịnh lí Lagrange, tổng quát hơn, chỉ cần ta

ñến ý tới ý nghĩa của ñạo hàm (trung bình giá trị biến thiên của hàm số).

1.2. ðỊNH LÍ LAGRANGE (Lagrange's Mean Value Theorem)

Page 72: Kỷ yếu hội thảo các trường chuyên dh db bắc bộ lần III

=========================================================== 74

HỘI CÁC TRƯỜNG THTP CHUYÊN KHU VỰC DUYÊN HẢI VÀ ðỒNG BẰNG BẮC BỘ

Hội thảo khoa học môn Toán học lần thứ III - 2010

ðịnh lí: Nếu ( )f x là hàm liên tục trên ñoạn

[ ; ]a b , có ñạo hàm trên khoảng ( ; )a b thì tồn tại

( ; )c a b∈ sao cho ( ) ( )

'( )f b f a

f cb a

−=

−.

Chứng minh:

Xét hàm số:

( ) ( )( ) ( )

f b f aF x f x x

b a

−= −

−.

Ta có: F(x) là hàm liên tục trên ñoạn [ ; ]a b , có

ñạo hàm trên khoảng ( ; )a b và ( ) ( )F a F b= .

Theo ñịnh lí Rolle tồn tại ( ; )c a b∈ sao cho '( ) 0F c = .

Mà ( ) ( )

'( ) '( )f b f a

F x f xb a

−= −

−, suy ra

( ) ( )'( )

f b f af c

b a

−=

−.

ðịnh lí Rolle là một hệ quả của ñịnh lí Lagrange (trong trường hợp ( ) ( )f a f b= )

Ý nghĩa hình học:

ðịnh lí Lagrange cho

phép ta ước lượng tỉ số ( ) ( )f b f a

b a

− do ñó nó còn ñược gọi là ñịnh lí Giá trị trung bình

(Mean Value Theorem). Từ ñó cho ta ý tưởng chứng minh các ñịnh lí về sự biến thiên của

hàm số, ñặt nền móng cho những ứng dụng của ñạo hàm.

ðịnh lí: Cho hàm số ( )f x có ñạo hàm trên khoảng ( ; )a b .

- Nếu '( ) 0, ( ; ) f x x a b> ∀ ∈ thì ( )f x ñồng biến trên ( ; )a b .

- Nếu '( ) 0, ( ; ) f x x a b< ∀ ∈ thì ( )f x nghịch biến trên ( ; )a b .

- Nếu '( ) 0, ( ; ) f x x a b= ∀ ∈ thì ( )f x là hàm hằng trên ( ; )a b .

Chứng minh:

Giả sử '( ) 0, ( ; ) f x x a b> ∀ ∈ và 1 2 1 2, ( ; ),x x a b x x∈ < , theo ñịnh lí Lagrange, tồn tại

1 2c (x ; x )∈ sao cho 2 1

2 1

( ) ( )'( )

f x f xf c

x x

−=

−.

Mà 1 2'( ) 0 ( ) ( ) ( )f c f x f x f x> ⇒ < ⇒ ñồng biến trên (a; b).

Cho hàm s� ( )f x th�a mãn các gi� thi�t c�a

��nh lí Lagrange, �� th� (C), A(a;f(a)), B(b;f(b)).

Khi �ó trên (C) t�n t�i �i�m C(c;f(c)),c (a; b)∈ mà

ti�p tuy�n c�a (C) t�i C song song v�i ���ng th�ng AB.

Joseph Louis Lagrange (1736 -

1813)

Page 73: Kỷ yếu hội thảo các trường chuyên dh db bắc bộ lần III

=========================================================== 75

HỘI CÁC TRƯỜNG THTP CHUYÊN KHU VỰC DUYÊN HẢI VÀ ðỒNG BẰNG BẮC BỘ

Hội thảo khoa học môn Toán học lần thứ III - 2010

Nếu trong giả thiết của ñịnh lí Lagrange ta thêm vào giả thiết '( )f x ñồng biến hoặc

nghịch biến trên [a; b] thì ta có thể so sánh ( ) ( )f b f a

b a

− với '( ), '( )f a f b .

Cụ thể: '( )f x ñồng biến trên [a;b] ( ) ( )

'( ) '( )f b f a

f a f bb a

−⇒ < <

'( )f x nghịch biến trên [a;b] ( ) ( )

'( ) '( )f b f a

f a f bb a

−⇒ > >

Từ ñây cho ta ý tưởng ứng dụng ñịnh lí Lagrange chứng minh bất ñẳng thức và ñánh

giá các tổng hữu hạn.

Cũng tương tự nếu trong giả thiết của ñịnh lí Lagrange ta thêm vào giả thiết '( )f x

ñồng biến hoặc nghịch biến trên [a; b] thì ta có thể so sánh ( ) ( )f c f a

c a

− với

( ) ( )f b f c

b c

với [ ; ]c a b∈ cho ta ý tưởng ñể chứng minh rất nhiều bất ñẳng thức, như bất ñẳng thức

Jensen…

Ngoài ra ñịnh lí Lagrange còn ñược phát biểu dưới dạng tích phân như sau:

ðịnh lí: Nếu ( )f x là hàm liên tục trên ñoạn [a; b] thì tồn tại ñiểm ( ; )c a b∈ thỏa

mãn: ( ) ( )( )b

a

f x dx f c b a= −∫

ðịnh lí Lagrange dạng tích phân ñược áp dụng chứng minh một số bài toán liên

quan ñến tích phân và giới hạn hàm số.

2. MỘT SỐ ỨNG DỤNG

2.1. CHỨNG MINH SỰ TỒN TẠI NGHIỆM CỦA PHƯƠNG TRÌNH

Bài toán 1. Chứng minh rằng phương trình acosx + bcos2x + ccos3x luôn có nghiệm

với mọi bộ các số thực a, b, c.

Lời giải:

Xét bsin2x s in3x

( ) asinx+ '( ) osx+bcos2x+ccos3x, x .2 3

cf x f x ac R= + ⇒ = ∀ ∈

Mà 0 0(0) ( ) 0 (0; ), '( ) 0f f x f xπ π= = ⇒∃ ∈ = , suy ra ñiều phải chứng minh.

Nhận xét: Bài toán trên có dạng tổng quát:

Cho hàm số f(x) liên tục trên [a; b], chứng minh rằng phương trình f(x) = 0 có ít nhất

một nghiệm trên (a; b).

Phương pháp giải:

Xét hàm F(x) thỏa mãn F(x) liên tục trên [a; b], F’(x) = f(x).g(x) với mọi x thuộc (a;

b), g(x) vô nghiệm trên (a;b) và F(a) = F(b). Theo ñịnh lí Rolle suy ra ñiều phải chứng minh.

Page 74: Kỷ yếu hội thảo các trường chuyên dh db bắc bộ lần III

=========================================================== 76

HỘI CÁC TRƯỜNG THTP CHUYÊN KHU VỰC DUYÊN HẢI VÀ ðỒNG BẰNG BẮC BỘ

Hội thảo khoa học môn Toán học lần thứ III - 2010

Bài toán 2. Cho số thực dương m và các số thực a, b, c thỏa mãn:

02 1

a b c

m m m+ + =

+ +.

Chứng minh rằng ax2 + bx + c = 0 có nghiệm thuộc (0; 1).

Hướng dẫn: Xét hàm số 2 1. . .

( )2 1

m m ma x b x c x

f xm m m

+ +

= + ++ +

.

Tương tự ta có bài toán tổng quát hơn.

Bài toán 3. Cho số thực dương m, số nguyên dương n và các số thực 0 1, ,..., na a a

thỏa mãn: 1 0... 01

n na a a

m n m n m

−+ + + =+ + −

.

Chứng minh rằng 11 1 0... 0n n

n na x a x a x a−

−+ + + + = có nghiệm thuộc (0; 1).

Hướng dẫn: Xét hàm số 11 0( ) ...1

m n m n mn na a af x x x x

m n m n m

+ + −−= + + ++ + −

Bài toán 4.(ðịnh lí Cauchy)

Nếu các hàm số ( ), ( )f x g x là các hàm số liên tục trên ñoạn [ ; ]a b , có ñạo hàm trên

khoảng ( ; )a b và '( )g x khác không trên khoảng ( ; )a b thì tồn tại ( ; )c a b∈ sao cho

( ) ( )'( )

( ) ( )

f b f af c

g b g a

−=

−.

Lời giải: Theo ñịnh Lagrange luôn tồn tại 0 ( ; )x a b∈ sao cho

0

( ) ( )'( )

g b g ag x

b a

−=

−( ) ( )g a g b⇒ ≠ .

Xét hàm số ( ) ( )

( ) ( ) ( )( ) ( )

f b f aF x f x g x

g b g a

−= −

−, ta có: F(x) là hàm liên tục trên ñoạn

[ ; ]a b , có ñạo hàm trên khoảng ( ; )a b và ( ) ( ) ( ) ( )

( ) ( )( ) ( )

f a g b f b g aF a F b

g b g a

−= =

−.

Theo ñịnh lí Rolle tồn tại ( ; )c a b∈ sao cho '( ) 0F c = .

Mà ( ) ( )

'( ) '( )( ) ( )

f b f aF x f x

g b g a

−= −

−, suy ra

( ) ( )'( )

( ) ( )

f b f af c

g b g a

−=

−.

Nhận xét: ðịnh lí Lagrange là hệ quả của ñịnh lí Cauchy (trong trường

hợp ( )g x x= )

Bài toán 5: Cho a + b – c = 0. Chứng minh rằng: asinx+9bsin3x+25csin5x = 0 có ít

nhất 4 nghiệm thuộc [0; π].

Page 75: Kỷ yếu hội thảo các trường chuyên dh db bắc bộ lần III

=========================================================== 77

HỘI CÁC TRƯỜNG THTP CHUYÊN KHU VỰC DUYÊN HẢI VÀ ðỒNG BẰNG BẮC BỘ

Hội thảo khoa học môn Toán học lần thứ III - 2010

Nhận xét: Bài toán này cũng tương tự các bài toán trên. ðể chứng minh ( )f x có ít

nhất n nghiệm ta chứng minh F(x) có ít nhất n + 1 nghiệm với F(x) là một nguyên hàm của

( )f x trên (a;b) (có thể phải áp dụng nhiều lần)

Lời giải: Xét hàm số: ( ) 3 5f x asinx bsin x csin x= − − − , ta có:

'( ) os 3 os3 5 os5f x ac x bc x cc x= − − − , ''( ) 9 3 25 5f x asinx bsin x csin x= + + .

Ta có 1 2 3

3 3 3(0) ( ) ( ) ( ) 0 (0; ), ( ; ), ( ; )

4 4 4 4 4 4f f f f x x x

π π π π π ππ π= = = = ⇒ ∃ ∈ ∈ ∈ sao

cho 1 2 3 4 1 2 5 2 3 4 5(0) '( ) ' ( ) '( ) 0 ( ; ), ( ; ) | ''( ) ''( ) 0f f x f x f x x x x x x x f x f x= = = = ⇒∃ ∈ ∈ = =

mà ''(0) ''( ) 0f f π= = ⇒ ñiều phải chứng minh.

Bài toán 6. Cho ña thức P(x) và Q(x) = aP(x) + bP’(x) trong ñó a, b là các số thực, a

≠ 0. Chứng minh rằng nếu Q(x) vô nghiệm thì P(x) vô nghiệm.

Lời giải: Ta có degP(x) = degQ(x)

Vì Q(x) vô nghiệm nên degQ(x) chẵn. Giả sử P(x) có nghiệm, vì degP(x) chẵn nên

P(x) có ít nhất 2 nghiệm.

- Khi P(x) có nghiệm kép x = x0 ta có x0 cũng là một nghiệm của P’(x) suy ra Q(x) có

nghiệm.

- Khi P(x) có hai nghiệm phân biệt x1 < x2.

Nếu b = 0 thì hiển nhiên Q(x) có nghiệm.

Nếu b ≠ 0 : Xét ( ) ( )a

xbf x e P x= ta có: ( )f x có hai nghiệm phân biệt x1 < x2

a a a a

b b b b1 1'( ) ( ) '( ) ( ( ) '( )) ( )

x x x xaf x e P x e P x e aP x bP x e Q x

b b b= + = + =

Vì ( )f x có hai nghiệm suy ra '( )f x có ít nhất 1 nghiệm hay Q(x) có nghiệm.

2.2. GIẢI PHƯƠNG TRÌNH

Bài toán 7: Giải phương trình: 3 5 2.4x x x+ = (1)

Lời giải:

Nhận xét: 0; 1x x= = là nghiệm của phương trình (1).

Gọi x0 là nghiệm của phương trình ñã cho. Ta ñược:

0 0 0 0 0 0 03 5 2.4 5 4 4 3 (1a)x x x x x x x+ = ⇔ − = −

Page 76: Kỷ yếu hội thảo các trường chuyên dh db bắc bộ lần III

=========================================================== 78

HỘI CÁC TRƯỜNG THTP CHUYÊN KHU VỰC DUYÊN HẢI VÀ ðỒNG BẰNG BẮC BỘ

Hội thảo khoa học môn Toán học lần thứ III - 2010

Xét hàm số 0 0( ) ( 1)x xf t t t= + − , ta có (1a) (4) (3)f f⇔ =

Vì f(t) liên tục trên [3; 4] và có ñạo hàm trong khoảng (3; 4), do ñó theo ñịnh lí Rolle

tồn tại c (3; 4)∈ sao cho: 0 0 01 10

0

0'( ) 0 [( 1) ]=0

1x x

xf c x c c

x

− −=

= ⇒ + − ⇔ =

Vậy phương trình (1) có hai nghiệm x = 0 và x = 1.

Bài toán 8: Giải phương trình: x5 3 2x (2)x − =

Lời giải:

Nhận xét: 0; 1x x= = là nghiệm của phương trình (2).

Gọi x0 là nghiệm của phương trình ñã cho, ta có: 0 0x0 05 5 3 3x (2a)x

x− = −

Xét hàm số: 00( ) x

f t t tx= − , khi ñó: (2a) (5) (3)f f⇔ =

Vì ( )f t liên tục trên [3; 5] và có ñạo hàm trên (3; 5), do ñó theo ñịnh lí Lagrange

luôn tồn tại c (3; 5)∈ sao cho: 0 010

0

0'( ) 0 ( 1)=0

1x

xf c x c

x

−=

= ⇒ − ⇔ =

Vậy phương trình (1) có hai nghiệm x = 0 và x = 1.

Bài toán 9. Giải phương trình: 3194.23 +=+ xxx (3).

Lời giải:

(5) ⇔ 03194.23 =−−+ xxx .

Xét hàm số: 3194.23)( −−+== xxfyxx ta có: 194ln4.23ln3)(' −+= xx

xf

Rxxfxx ∈∀>+= ,0)4(ln4.2)3(ln3)('' 22 hay ''( )f x vô nghiệm, suy ra '( )f x có

nhiều nhất 1 nghiệm, suy ra ( )f x có nhiều nhất 2 nghiệm.

Mà 0)2()0( == ff do ñó (3) có ñúng hai nghiêm 2,0 == xx .

Bài toán 10. Giải phương trình: cos(1 cos )(2 4 ) 3.4cos (4)xx x+ + =

Lời giải:

ðặt cos , ( [-1;1]) t x t= ∈

Page 77: Kỷ yếu hội thảo các trường chuyên dh db bắc bộ lần III

=========================================================== 79

HỘI CÁC TRƯỜNG THTP CHUYÊN KHU VỰC DUYÊN HẢI VÀ ðỒNG BẰNG BẮC BỘ

Hội thảo khoa học môn Toán học lần thứ III - 2010

(3) (1 )(2 4 ) 3.4 (1 )(2 4 ) 3.4 0 t t t tt t⇔ + + = ⇔ + + − =

Xét hàm số: ( ) (1 )(2 4 ) 3.4 t tf t t= + + −

2'( ) 2 4 ( - 2)4 ln 4, ''( ) 2.4 ln 4 ( - 2)4 ln 4 t t t tf t t f t t⇒ = + + = +

Ta có: 2

''( ) 0 2ln 4

f t t= ⇔ = + ⇒ ''( )f t có một nghiệm duy nhất

'( )f t⇒ có nhiều nhất hai nghiệm ( )f t⇒ có nhiều nhất ba nghiệm.

Mặt khác dễ thấy1

(0) ( ) (1) 02

f f f= = = , do ñó ( )f t có ba nghiệm 1

0, ,12

t = .

Kết luận: Nghiệm của phương trình (4) là:

2 , 2 , 2 , 2 3

x k x k x k k Zπ π

π π π= + = ± + = ∈

2.3. CHỨNG MINH BẤT ðẲNG THỨC

Bài toán 11. Cho hai số thực dương a, b thỏa mãn a < b. Chứng minh rằng:

lnb a b b a

b a a

− −< <

Lời giải:

Xét hàm số 1

( ) ln '( ) , (0; ).f x x f x xx

= ⇒ = ∀ ∈ +∞

Theo ñịnh lí Lagrange luôn tồn tại c (a; b)∈ sao cho ( ) ( )

'( )f b f a

f cb a

−=

− hay

1 ln lnln

b a a b b

c b a c a

− −= ⇔ =

− mà

1 1 10 a b c

b c a< < < ⇒ < < ⇒ ln

b a b b a

b a a

− −< < .

Bài toán 12. Chứng minh rằng: 11 1(1 ) (1 ) , (0; ).

1x x

xx x

++ < + ∀ ∈ +∞+

Lời giải:

Ta có: 11 1(1 ) (1 ) [ln( 1) - ln ] ( 1)[ln( 2) - ln( 1)]

1x x

x x x x x xx x

++ < + ⇔ + < + + ++

Page 78: Kỷ yếu hội thảo các trường chuyên dh db bắc bộ lần III

=========================================================== 80

HỘI CÁC TRƯỜNG THTP CHUYÊN KHU VỰC DUYÊN HẢI VÀ ðỒNG BẰNG BẮC BỘ

Hội thảo khoa học môn Toán học lần thứ III - 2010

ðặt ( ) [ln( 1) - ln ]f x x x x= +

Ta có: 1

'( ) ln( 1) ln 1 ln( 1) ln1 1

xf x x x x x

x x= + − + − = + − −

+ +

Áp dụng ñịnh lí Lagrange ñối với hàm số: y = lnt trên [x; x+1], thì tồn tại

c (x; x+1)∈ sao cho:1

'( ) ln( 1) ln ln( 1) ln .f c x x x xc

= + − ⇒ = + −

Mà 1 1 1

0 11

x c xx c x

< < < + ⇒ > >+

1 1 1ln( 1) ln ln( 1) ln 0

1 1x x x x

x x x⇒ > + − > ⇒ + − − >

+ +

'( ) 0, (0;+ )f x x⇒ > ∀ ∈ ∞ ⇒hàm số ( )f x ñồng biến trên (0;+ ).∞

Từ (1) suy ra: '( ) 0, (0; )f x x> ∀ ∈ +∞ ⇒ ( )f x ñồng biến trên (0; ).+∞

Suy ra: ( 1) ( ), (0; )f x f x x+ > ∀ ∈ +∞ ⇒ ñiều phải chứng minh.

Nhận xét: Trong ví dụ trên thực chất của vấn ñề là ta ñi chứng minh hàm số

1( ) (1 )x

F xx

= + ñồng biến trên (0; )+∞ và ta ñi chứng minh hàm số ( ) ln ( )f x F x= ñồng

biến trên (0; )+∞ , ñến ñây bài toán trở về giống như ví dụ 1. Tương tự ta chứng minh ñược

hàm số 11( ) (1 )x

G xx

+= + nghịch biến trên (0; ).+∞

Ta có thể chứng minh bài toán 12 bằng cách khác.

Xét hàm số: ( ) ln(1 )F x x= +

Với mọi cặp số thực x, y bất kì thỏa mãn 0 < x < y, theo ñịnh lí Lagrange, luôn tồn

tại 0 0(0; ), ( ; )x x y x y∈ ∈ thỏa mãn:

0 0

( ) (0) ( ) ( )'( ) , '( )

0

f x f f y f xf x f y

x y x

− −= =

− −

hay 0 0

1 ln(1 ) 1 ln(1 ) ln(1 );

1 1

x y x

x x y y x

+ + − += =

+ + −.

Mà 0 0

1 1

1 1x y>

+ +

ln(1 ) ln(1 ) ln(1 )ln(1 ) ln(1 ).

x y xy x x y

x y x

+ + − +⇒ > ⇒ + > +

Vậy với mọi cặp số thực x, y bất kì thỏa mãn 0 < x < y, luôn có

ln(1 ) ln(1 ),y x x y+ > + thay x bởi 1

y và y bởi

1

x ta có:

Page 79: Kỷ yếu hội thảo các trường chuyên dh db bắc bộ lần III

=========================================================== 81

HỘI CÁC TRƯỜNG THTP CHUYÊN KHU VỰC DUYÊN HẢI VÀ ðỒNG BẰNG BẮC BỘ

Hội thảo khoa học môn Toán học lần thứ III - 2010

1 1 1 1 1 1ln(1 ) ln(1 ) (1 ) (1 )y x

x y y x y x+ > + ⇒ + > +

Bài toán 13. (Bất ñẳng thức Jensen)

Cho hàm số ( )f x có ñạo hàm cấp hai trên (a; b) và ''( ) 0, ( ; )f x x a b> ∀ ∈ .

Chứng minh rằng: 1 2 1 21 2

( ) ( )( ), , ( ; )

2 2

f x f x x xf x x a b

+ +≤ ∀ ∈

Lời giải:

ðẳng thức xảy ra khi 1 2x x= .

Khi 1 2x x< , theo ñịnh lí Lagrange, tồn tại 1 2 1 21 2( ; ), ( ; )

2 2

x x x xc x d x

+ +∈ ∈ thỏa mãn

1 2 1 21 2

2 1 2 1

( ) ( ) ( ) ( )2 2'( ) , '( )

2 2

x x x xf f x f x f

f c f dx x x x

+ +− −

= =− −

.

Mà ''( ) 0, ( ; ) '( )f x x a b f x> ∀ ∈ ⇒ ñồng biến trên (a; b)

1 2 1 2 1 2 1 21 2

( ) ( )'( ) ( ) ( ) ( ) ( ) ( ) ( ).

2 2 2 2

x x x x f x f x x xf c f d f f x f x f f

+ + + +⇒ < ⇒ − < − ⇒ >

Bài toán 14. (Bất ñẳng thức Bernoulli)

Với mọi số thực x thỏa mãn x > -1, chứng minh rằng (1 ) 1 .nx nx+ ≥ +

Lời giải:

- Khi x > 0: xét ( ) (1 )nf t t= + , theo ñịnh lí Lagrange ta có (0; )a x∈ thỏa mãn

1( ) (0) '( ) (1 ) 1 (1 ) (1 ) 1n n nf x f xf a x nx a nx x nx

−− = ⇒ + − = + > ⇒ + > +

- Khi -1< x < 0: xét ( ) (1 )nf t t= + , theo ñịnh lí Lagrange ta có ( ;0)a x∈ thỏa mãn

1( ) (0) '( ) (1 ) 1 (1 ) (1 ) 1n n nf x f xf a x nx a nx x nx

−− = ⇒ + − = + > ⇒ + > +

Vậy (1 ) 1 , (-1; )nx nx x+ ≥ + ∀ ∈ +∞ . ðẳng thức xảy ra khi và chỉ khi x = 0.

Bài toán 15. Cho hàm số ( )f x có ñạo hàm cấp hai trên R, ''( ) 0,f x x R≥ ∀ ∈

( ''( ) 0f x = có số nghiệm ñếm ñược). Chứng minh rằng:

*

1

( ) (0) '( ) ( 1) (1),n

i

f n f f i f n f n N=

− < < + − ∀ ∈∑ .

Lời giải:

Vì ''( ) 0,f x x R≥ ∀ ∈ ( ''( ) 0f x = có số nghiệm ñếm ñược) '( )f x⇒ ñồng biến trên

R.

Theo ñịnh lí Lagrange, luôn tồn tại ( ; 1)ix i i∈ + sao cho:

Page 80: Kỷ yếu hội thảo các trường chuyên dh db bắc bộ lần III

=========================================================== 82

HỘI CÁC TRƯỜNG THTP CHUYÊN KHU VỰC DUYÊN HẢI VÀ ðỒNG BẰNG BẮC BỘ

Hội thảo khoa học môn Toán học lần thứ III - 2010

'( ) ( 1) ( ),if x f i f i i R= + − ∀ ∈ .

Vì '( )f x ñồng biến trên R '( ) '( ) '( 1)if i f x f i⇒ < < +

'( ) ( 1) ( ) '( 1),f i f i f i f i i R⇒ < + − < + ∀ ∈ .

*

1 1

'( ) [ ( 1) ( )] ( 1) (1),n n

i i

f i f i f i f n f n N= =

⇒ < + − = + − ∀ ∈∑ ∑

và *

1 1

'( ) [ ( ) ( 1)] ( ) (0),n n

i i

f i f i f i f n f n N= =

> − − = − ∀ ∈∑ ∑

Nhận xét: Nếu ''( ) 0,f x x R≤ ∀ ∈ thì bất ñẳng thức cần chứng minh sẽ ñổi chiều.

Bài toán 16. Chứng minh rằng: *

1

11 ln ln( 1),

n

i

n n n Ni=

+ > > + ∀ ∈∑ .

Lời giải:

Xét 1

( ) ln '( )f x x f xx

= ⇒ = và '( )f x nghịch biến trên (0 : )+∞

Tương tự bài toán trên ta có: *

1

( ) (1) '(1) '( ) ( 1) (1),n

i

f n f f f i f n f n N=

− + > > + − ∀ ∈∑

*

1

11 ln ln( 1),

n

i

n n n Ni=

⇒ + > > + ∀ ∈∑

Bài toán 17: Cho số nguyên dương k, tìm 52

5 41

1 1

5

k

i i=

∑ (trong ñó [x] là số nguyên lớn

nhất không vượt quá x).

Lời giải:

Xét hàm số 5( ) 5f x x= , ta có: 5 4

1'( ) '( )f x f x

x= ⇒ nghịch biến trên (0; )+∞ .

Suy ra

525 5 55 5 5

5 41 1

1 1( ) (0) '( ) ( 1) (1) 2 2 1 1 2 1

5

kn

k k k

i i

f n f f i f n fi= =

− > > + − ⇒ > > + − > −∑ ∑

510

5 41

1 12 .

5

k

k

i i=

⇒ =

Nhận xét: Từ ba bài toán trên ta nhận thấy ñể ñánh giá tổng *

1

( ),n

i

f i n N=

∈∑

( ( )f x ñồng biến hoặc nghịch biến trên (0 : )+∞ ), chúng ta phải xét hàm số ( )F x là một

nguyên hàm của ( )f x trên (0 : )+∞ và giải quyết tương tự bài toán trên.

Page 81: Kỷ yếu hội thảo các trường chuyên dh db bắc bộ lần III

=========================================================== 83

HỘI CÁC TRƯỜNG THTP CHUYÊN KHU VỰC DUYÊN HẢI VÀ ðỒNG BẰNG BẮC BỘ

Hội thảo khoa học môn Toán học lần thứ III - 2010

Từ việc ước lượng ñược tổng *

1

( ),n

i

f i n N=

∈∑ ta có thể nghĩ ñến bài toán tìm giới hạn

1

lim ( ) ( ),n

i

g n f i=

∑ ta nghiên cứu ở các bài toán sau.

Bài toán 18. Tính 1

0

1 ilim os

2n

n

i

cn

π−

=

∑ .

Lời giải:

Xét 2

( ) sin '( ) os '( ) 0, [ ; ]2 2

n x xf x f x c f x x n n

n n

π π

π= ⇒ = ⇒ ≥ ∀ ∈ −

'( )f x⇒ ñồng biến trên [ ; ]n n− . Suy ra 1

( ) (0) '( ) ( 1) (1)n

i

f n f f i f n f=

− < < + −∑

1 1

0 0

2 i 2 2 1 i 2 1os os sin os os sin

2n 2n 2n 2n 2n 2n

n n

i i

n nc c c c

n n

π π π π π π

π π π π

− −

= =

⇒ < < − ⇒ < < −∑ ∑

Mà 2 2 1 2

lim( os ) lim( sin )2n 2n

cn

π π

π π π= − =

1

0

1 i 2lim os

2n

n

i

cn

π

π

=

⇒ =∑ (Nguyên lí kẹp).

Bài toán 19. Cho phương trình: 1

1.

n

i

ni nx=

=+

Chứng minh rằng: Với mỗi số nguyên dương n phương trình có duy nhất một

nghiệm dương. Kí hiệu nghiệm ñó là xn, tìm limxn.

Lời giải:

Xét 1

1( )

n

i

f x ni nx=

= −+

Ta có: 3

1

1'( ) 0, (0; ) ( )

2 ( )

n

i

f x x f xi nx=

= − < ∀ ∈ +∞ ⇒+

liên tục, nghịch biến trên

[0; )+∞ .

Mà 1

1(0) 0, lim ( ) 0 ( ) 0

n

xi

nf n n f x n f x

i n →+∞=

= − > − = = − < ⇒ =∑ có 1 nghiệm

dương duy nhất.

Xét hàm số ( ) 2n n

F x x nx= + , ta có 1

'( ) '( )n n

n

F x F xx nx

= ⇒+

nghịch biến trên

(0; )+∞ .

1

( ) (0) '( ) ( 1) (1)n

n n n n n

i

F n F F i F n F=

⇒ − > > + −∑

Page 82: Kỷ yếu hội thảo các trường chuyên dh db bắc bộ lần III

=========================================================== 84

HỘI CÁC TRƯỜNG THTP CHUYÊN KHU VỰC DUYÊN HẢI VÀ ðỒNG BẰNG BẮC BỘ

Hội thảo khoa học môn Toán học lần thứ III - 2010

1

12( ) 2( 1 1 )

n

n n n n

i n

n nx nx n nx nxi nx=

⇒ + − > > + + − ++

2( ) 2( 1 1 )n n n n

n nx nx n n nx nx⇒ + − > > + + − +

1 1 11 1

2n n n nx x x xn n

⇒ + − > > + + − +

1 1 21 2 1 1

n n n n n nx x x x x x

n n n⇒ + + < < + + + + < + + +

22 1 2 lim( 1 ) 2n n n nx x x x

n⇒ − < + + < ⇒ + + =

n

1 3 9lim( 1 ) lim lim x

2 4 16n n nx x x⇒ + − = ⇒ = ⇒ = .

Bài toán 20: Cho các số thực dương a, b, c, d thỏa mãn 2 2 2 2 2

4 4 4 4 4

a b c d e

a b c d e

+ + = +

+ + = +

Chứng minh rằng 3 3 3 3 3.a b c d e+ + < +

Nhận xét: Trong bài toán này từ giả thiết 2 2 2 2 2

4 4 4 4 4

a b c d e

a b c d e

+ + = +

+ + = +, ta nhìn thấy ngay

giả thiết của ñịnh lí Rolle với hàm số ( ) ( (2) (4) 0)x x x x xf x a b c d e f f= + + − − = = , khi ñó

ta phải chứng minh (3) 0f < . Vì ( )f x liên tục và (3) 0f < , suy ra tồn tại khoảng ( ; ) 3m n ∋

sao cho ( ) 0, ( ; )f x x m n< ∀ ∈ , do ñó bài toán trở thành xét dấu của ( )f x , vì thế ta cần kiểm

soát ñược các nghiệm của ( )f x .

Lời giải:

Không mất tính tổng quát ta có thể giả sử 1,a b c d e≤ ≤ = ≤

Nếu 2 2 2 21 1 ( 0)d d x x b a e x≥ ⇒ = + ≥ ⇒ + = +

4 4 4 4 4 4 2 2 2 2 4( ) 1 (1 )a b c d e a e x a x e+ + = + ⇔ + + − + = + +

2 2 2

2 2 2 2 2 2 2 2 2

2

0

( 1) ( ) 0 1 1

x e a b

e a x a a e e a e a

e a e a

= = +

⇔ − − + − = ⇔ ⇔= + = +

= =

( Mâu thuẫn

) 1d⇒ <

Tương tự ta có 1a b d e≤ < ≤ <

Xét hàm số ( ) 1 (2) (4) 0x x x xf x a b d e f f= + + − − ⇒ = =

Page 83: Kỷ yếu hội thảo các trường chuyên dh db bắc bộ lần III

=========================================================== 85

HỘI CÁC TRƯỜNG THTP CHUYÊN KHU VỰC DUYÊN HẢI VÀ ðỒNG BẰNG BẮC BỘ

Hội thảo khoa học môn Toán học lần thứ III - 2010

Giả sử ( )f x có nghiệm 0 2;4.x ≠ Theo ñịnh lí Rolle, tồn tại 1 2x x< thỏa mãn:

1 2'( ) '( ) 0f x f x= = hay 1 1 1 1ln ln ln ln ,x x x xa a b b d d e e+ = +

2 2 2 2ln ln ln lnx x x xa a b b d d e e+ = +

2 2 2 2

1 1 1 1

ln ln ln ln

ln ln ln ln

x x x x

x x x x

a a b b d d e e

a a b b d a e b

+ +⇒ =

+ +

Mà 2 2 1 2 1 21 0 ln ln ln lnx x x x x xa b d e a a b b a b a b b

−≤ < ≤ < ⇒ > + ≥ +

2 2

2 1

1 1

ln ln

ln ln

x xx x

x x

a a b bb

a a b b

− +⇒ ≥

+

và 2 2

2 2 2 2 1 1 2 1

1 1

ln lnln ln ln ln 0

ln ln

x xx x x x x x x x

x x

d d e ed d e e d d d e b d

d d e e

− − ++ ≤ + < ⇒ ≤

+

2 2 2 2

1 1 1 1

ln ln ln ln

ln ln ln ln

x x x x

x x x x

a a b b d d e e

a a b b d a e b

+ +⇒ <

+ + (Mâu thuẫn).

Vậy ( )f x chỉ có hai ngiệm x = 2, x = 4 và '( )f x có 1 nghiệm duy nhất, và nó thuộc

(2; 4). Vì ( )f x liên tục nên ( )f x mang cùng một dấu trên mỗi khoảng

( ;2), (2;4), (4; ).−∞ +∞ Mà (0) 1 0 ( ) 0, ( ;0) ( ) 0, (2;4)f f x x f x x= > ⇒ > ∀ ∈ −∞ ⇒ < ∀ ∈ (vì

nếu ( ) 0, (2;4)f x x> ∀ ∈ thì x = 2 là nghiệm của '( )f x ) (3) 0f⇒ < (ñiều phải chứng

minh).

ðịnh lí Lagrange còn ñược sử dụng ñể giải quyết một số bài toán về bất ñẳng thức

ñối xứng, nhằm mục ñích làm giảm số biến. Nếu cần chứng minh bất ñẳng thức ñối xứng n

biến 1 2, ,..., na a a thì ta xét ña thức 1 2( ) ( )( )...( )nf x x a x a x a= − − − , suy ra ( )f x có n

nghiệm, do ñó '( )f x có n – 1 nghiệm 1 2 1, ,..., nb b b − , và dựa vào ñịnh lí Viète ta ñưa về

chứng minh bất ñẳng thức ñối xứng với n – 1 biến 1 2 1, ,..., nb b b − .

Bài toán 21. Cho a < b < c, chứng minh rằng:

2 2 2 2 2 23 3 3a a b c a b c ab bc ca b a b c a b c ab bc ca c< + + − + + − − − < < + + + + + − − − <

Lời giải:

Xét hàm số: ( ) ( )( )( ) ( ) ( ) ( ) 0f x x a x b x c f a f b f c= − − − ⇒ = = =

Theo ñịnh lí Lagrange tồn tại 1 2a x b x c< < < < sao cho:

1( ) ( ) ( ) '( )f a f b a b f x− = − ,

Page 84: Kỷ yếu hội thảo các trường chuyên dh db bắc bộ lần III

=========================================================== 86

HỘI CÁC TRƯỜNG THTP CHUYÊN KHU VỰC DUYÊN HẢI VÀ ðỒNG BẰNG BẮC BỘ

Hội thảo khoa học môn Toán học lần thứ III - 2010

1 1 2( ) ( ) ( ) '( ) '( ) '( ) 0f c f b c b f x f x f x− = − ⇒ = = 2'( ) 3 2( )f x x a b c x ab bc ca= − + + + + +

2 2 2

1 3

a b c a b c ab bc cax

+ + − + + − − −⇒ =

2 2 2

2 3

a b c a b c ab bc cax

+ + + + + − − −=

Do ñó, từ 1 2a x b x c< < < < . Suy ra:

2 2 23 3a a b c a b c ab bc ca b< + + − + + − − − <2 2 2 3a b c a b c ab bc ca c< + + + + + − − − <

Bài toán 22. Cho các số thực không âm a, b, c, d. Chứng minh rằng:

3

4 6

abc bcd cda dab ab bc cd da ac db+ + + + + + + +≤

Lời giải:

Xét ( ) ( )( )( )( )f x x a x b x c x d= − − − − .

ðặt

, , ,p a b c d q ab bc cd da ac bd r abc bcd cda dab s abcd= + + + = + + + + + = + + + =

4 3 2 3 2( ) '( ) 4 3 2f x x px qx rx s f x x px qx r⇒ = − + − + ⇒ = − + −

Ta có ( ) ( ) ( ) ( ) 0f a f b f c f d= = = = , theo ñịnh lí Rolle suy ra '( ) 0f x = có ba

nghiệm (nếu a = b thì a là nghiệm của f’(x)).

Suy ra tồn tại , , w 0u v ≥ thỏa mãn '( ) 4( )( )( w)f x x u x v x= − − −

3 24 4( ) 4( ) 4x u v w x uv vw wu x uvw= − + + + + + −

Page 85: Kỷ yếu hội thảo các trường chuyên dh db bắc bộ lần III

=========================================================== 87

HỘI CÁC TRƯỜNG THTP CHUYÊN KHU VỰC DUYÊN HẢI VÀ ðỒNG BẰNG BẮC BỘ

Hội thảo khoa học môn Toán học lần thứ III - 2010

3

41

21

4

u v w p

uv vw wu q

uvw r

+ + =

⇒ + + =

=

.Mà

23 3 3( )3 3 6 4

uv vw wu uv vw wu q ruvw uvw

+ + + +≥ ⇒ ≥ ⇒ ≥

3

4 6

abc bcd cda dab ab bc cd da ac db+ + + + + + + +⇒ ≤

ðẳng thức xảy ra wu v a b c d⇔ = = ⇔ = = = .

2.4. TÌM GIỚI HẠN DÃY SỐ

ðịnh lí Lagrange ñược sử dụng ñể giải quyết một số bài toán vế giới hạn dãy số, với

các dãy số xác ñịnh bởi hàm số ( )f x và dãy số xác ñịnh bởi nghiệm của một phương trình

( ) 0nf x = , nói chung ( ),f x ( )nf x là các hàm số có ñạo hàm và ñơn ñiệu trên tập xác ñịnh

của chúng, ñạo hàm của chúng có thể ước lượng ñược bởi một bất ñẳng thức. Do ñó nếu tìm

ñược giới hạn là a, ta có thể so sánh ñược hiệu ( ) ( ),nf x f a− ( ) ( )n n nf x f a− với nx a− và có

thể ước lượng ñược xn.

Bài toán 23. Cho dãy số thực (xn) xác ñịnh bởi:1

*1 2

2007

3 ,1

n

n

n

x

xx n N

x+

= = + ∀ ∈ −

Tìm giới hạn của dãy số khi n tiến dần tới dương vô cùng.

Lời giải: Ta có *3, .n

x n N> ∀ ∈

Xét f(x) = 1

32 −

+x

x, ta có:

2 3

1'( )

( 1)f x

x= −

1'( ) , ( 3; )

2 2f x x⇒ < ∀ ∈ +∞ .

Page 86: Kỷ yếu hội thảo các trường chuyên dh db bắc bộ lần III

=========================================================== 88

HỘI CÁC TRƯỜNG THTP CHUYÊN KHU VỰC DUYÊN HẢI VÀ ðỒNG BẰNG BẮC BỘ

Hội thảo khoa học môn Toán học lần thứ III - 2010

Nếu (xn) có giới hạn thì giới hạn ñó là nghiệm lớn hơn 3 của phương trình

( )f x x= . Ta có: 2

( ) 31

xf x x x

x= ⇔ = +

22

2( 3)

1

xx

x⇔ − =

2 2 2( 3 ) 2( 3 ) 3 0x x x x⇔ − − − − =2

2

3 1

3 3

x x

x x

− = −⇔

− =

3 15.

2x

+⇔ =

ðặt 3 15

2a

+= , theo ñịnh lý Lagrange, luôn tồn tại ( ; )n nc x a∈ hoặc ( ; )na x thỏa

mãn: ( ) ( ) '( ) .n n n

f x f a f c x a− = −

1 1

1 1( ) ( ) '( ) ... ( )

2 2 2 2n

n n n n nx a f x f a f c x a x a x a+⇒ − = − = − < − < < −

Mà 1

1lim( ) 0

2 2n

x a− = , do ñó limxn = a = 2

153 +.

Nhận xét:

Trong bài toán trên việc giải phương trình ( )f x x= không nhất thiết phải trình bày,

ta chỉ cần chọn ñược nghiệm thỏa mãn của nó là ñược.

Bài toán trên có dạng tổng quát:

Cho dãy số thực (xn) xác ñịnh bởi:1

*1 ( ),

n n

x a

x f x n N+

=

= ∀ ∈. Chứng minh rằng:

a) Nếu ( )f x là hàm số có ñạo hàm trên khoảng D chứa a và '( ) 1,f x b x D< < ∀ ∈ thì

(xn) có giới hạn hữu hạn khi n tiến dần ñến dương vô cùng.

b) Nếu ( )f x là hàm số có ñạo hàm trên khoảng D chứa a, ( ) 0f a ≠ và

'( ) 1,f x b x D> > ∀ ∈ thì |xn| tiến dần ñến dương vô cùng khi n tiến dần ñến dương vô cùng.

Phương pháp giải

Page 87: Kỷ yếu hội thảo các trường chuyên dh db bắc bộ lần III

=========================================================== 89

HỘI CÁC TRƯỜNG THTP CHUYÊN KHU VỰC DUYÊN HẢI VÀ ðỒNG BẰNG BẮC BỘ

Hội thảo khoa học môn Toán học lần thứ III - 2010

a) - Nếu phương trình ( )f x x= giải ñược (tìm ñược nghiệm) thì ta giải quyết bài

toán tổng quát tương tự bài toán trên và khi ñó ta tìm ñược giới hạn của dãy số khi n tiến dần

tới dương vô cùng.

- Nếu phương trình ( )f x x= khó giải thì ta giải quyết bài toán tổng quát bằng cách

sử dụng tiêu chuẩn Cauchy. Bài toán sau ñây là một ví dụ cụ thể.

b) Tương tự ý a.

- Khi 0 0 0 0: , ( )a D a a f a a∃ ∈ ≠ = luôn tồn tại 0( ; )n nc x a∈ hoặc 0( ; )na x thỏa mãn:

0 0( ) ( ) '( )n n n

f x f a f c x a− = −

1 1 0 0 0( ) ( ) ... limn

n n n n nx a x a f x f a b x a b a a x+ +⇒ + ≥ − = − > − > > − ⇒ = +∞

- Khi phương trình f(x)=x vô nghiệm, ta có f(x)-x > 0 ∀x∈D hoặc f(x)-x < 0 ∀x∈D suy

ra xn tăng hoặc giảm. Nếu xn có giới hạn thì giới hạn ñó là nghiệm của phương trình f(x) = x,

do ñó lim nx = +∞

Bài toán 24. (Dự bị VMO 2008)

Cho số thực a và dãy số thực (xn) xác ñịnh bởi:

x1 = a và xn+1 = ln(3+cosxn + sinxn) – 2008, *.n N∀ ∈

Chứng minh rằng dãy số (xn) có giới hạn hữu hạn khi n tiến dần ñến dương vô cùng.

Lời giải:

ðặt f(x) = ln(3+sinx+cosx) – 2008, ta có:cos sin

'( ) , R3 sin cos

x xf x x

x x

−= ∀ ∈

+ +.

Mà 2|cossin|,2|sincos| ≤+≤− xxxx , suy ra: .123

2|)('| <=

−≤ qxf

Theo ñịnh lý Lagrange : với mọi cặp hai số thực x, y (x < y), luôn tồn tại ( ; )z x y∈

thỏa mãn: f(x) – f(y) = f’(z)(x-y).

Từ ñó suy ra |f(x) – f(y)| ≤ q|x – y| với mọi x, y thuộc R.

Áp dụng tính chất trên với m > n ≥ N, ta có :

Page 88: Kỷ yếu hội thảo các trường chuyên dh db bắc bộ lần III

=========================================================== 90

HỘI CÁC TRƯỜNG THTP CHUYÊN KHU VỰC DUYÊN HẢI VÀ ðỒNG BẰNG BẮC BỘ

Hội thảo khoa học môn Toán học lần thứ III - 2010

|xm – xn| = |f(xm-1) – f(xn-1)| ≤ q|xm-1- xn-1| ≤ …≤ qn-1|xm-n+1 – x1| ≤ qN-1|xm-n+1 – x1|.

Mặt khác dãy (xn) bị chặn và q < 1 nên với mọi ε > 0 tồn tại N ñủ lớn sao cho:

qN-1|xm-n+1 – x1| < ε.

Như vậy dãy (xn) thoả mãn tiêu chuẩn Cauchy, do ñó (xn) hội tụ.

Bài toán 25. (VMO 2007)

Cho số thực a > 2 và 10 10 1( ) ... 1n n n

nf x a x x x x+ −= + + + + + .

a) Chứng minh rằng với mỗi số nguyên dương n, phương trình ( )nf x a= luôn có

ñúng một nghiệm dương duy nhất. Kí hiệu nghiệm ñó là xn.

b) Chứng minh rằng dãy (xn) có giới hạn bằng 1a

a

− khi n dần ñến vô cùng.

Lời giải:

ðặt ( ) ( )n nF x f x a= − , ta có ( )nF x liên tục, ñồng biến trên [0; )+∞ và

10(0) 1 0, (1) 1 0.n nF a F a n a= − < = + + − > Suy ra phương trình ( )nf x a= luôn có ñúng

một nghiệm xn dương duy nhất.

ðặt 9 *1( ) ( 1)[( 1) 1] ( ) ,n

n n n

ab f b b a a a f b a b x n N

a

−= ⇒ = − − − + ⇒ > ⇒ > ∀ ∈

Theo ñịnh lí Lagrange, luôn tồn tại ( ; )n nc x b∈ thỏa mãn:

( ) ( ) '( )( )n n n n nf b f x f c b x− = − .

Mà '( ) 1nf c > nên 9n( ) ( ) ( 1)[( 1) 1] lim xn

n n n nb x f b f x b a a b− < − = − − − ⇒ =

9n( 1)[( 1) 1] lim xn

nb b a a x b b⇒ − − − − < < ⇒ = (vì (0;1)b ∈ ).

Nhận xét:

Bài toán trên sẽ khó khăn hơn nhiều nếu ñề bài không cho trước giới hạn của dãy số.

Khi ñó câu hỏi ñặt ra là giới hạn ñó bằng bao nhiêu?

Ta có thể trả lời câu hỏi ñó như sau:

Page 89: Kỷ yếu hội thảo các trường chuyên dh db bắc bộ lần III

=========================================================== 91

HỘI CÁC TRƯỜNG THTP CHUYÊN KHU VỰC DUYÊN HẢI VÀ ðỒNG BẰNG BẮC BỘ

Hội thảo khoa học môn Toán học lần thứ III - 2010

Trước hết giới hạn của dãy số phải thuộc khoảng (0; 1), giả sử giới hạn của dãy số là

b ta có: 10 10 1 1 1( ) ( 1) lim ( )

1 1 1n

n nf b b a b f b

b b b= + + − ⇒ = −

− − − (vì (0;1)b ∈ ).

Mà 1 1

( )1n n

af x a a b

b a

−= ⇒ − = ⇒ =

−.

Trong bài toán dạng trên dãy số xác ñịnh là dãy nghiệm thuộc (a; b) của phương

trình ( ) 0nf x = , với giả thiết ( )nf x là hàm số ñồng biến hoặc nghịch biến trên (a;

b), '( )nf x c< với mọi số nguyên dương n và số thực dương x thuộc (a; b), khi giải bài toán

dạng này nói chung ta ñiều khó khăn nhất là xác ñịnh ñược giới hạn của dãy số.

Bài toán 26: (VMO 2002)

Xét phương trình 2

1

1 1

1 2

n

i i x=

=−

∑ , với n là số nguyên dương.

a) Chứng minh rằng với mỗi số nguyên dương n, phương trình nêu trên có một nghiệm

duy nhất lớn hơn 1; kí hiệu nghiệm ñó là xn.

b) Chứng minh rằng lim 4nx = .

Lời giải:

a) Xét 2

1

1 1( )

1 2

n

n

i

f xi x=

= −−

∑ , ta có: ( )nf x liên tục và nghịch biến trên (1; ).+∞

Mà 1

1lim ( ) , lim ( )

2n nxx

f x f x+ →+∞→

= +∞ = − ⇒ ( ) 0nf x = có một nghiệm duy nhất lớn hơn 1.

b) Với mỗi số nguyên dương n ta có:1

(4) 0 (4) ( ) 42(2 1)n n n n nf f f x x

n= − < ⇒ < ⇒ <

+

Theo ñịnh lí Lagrange, luôn tồn tại ( ;4)n nc x∈ thỏa mãn:

(4) ( ) '( )(4 )n n n n nf f x f c x− = − .

Mà 1 9

'( ) 4 9( (4) ( )) 49 2(2 1)n n n n n nf c x f f x x

n< − ⇒ − < − − ⇒ − <

+

Page 90: Kỷ yếu hội thảo các trường chuyên dh db bắc bộ lần III

=========================================================== 92

HỘI CÁC TRƯỜNG THTP CHUYÊN KHU VỰC DUYÊN HẢI VÀ ðỒNG BẰNG BẮC BỘ

Hội thảo khoa học môn Toán học lần thứ III - 2010

94 4 lim 4.

2(2 1) n nx xn

⇒ − < < ⇒ =+

3. BÀI TẬP TỰ GIẢI

1. Giải các phương trình sau.

a) xxx 3)12(log)13(log 32 =+++

b) 2008 2010 2.2009x x x+ =

c) (4 2)(2 ) 6xx+ − =

2. Chứng minh nếu hàm số ( )f x có ñạo hàm cấp 2 trên ñoạn [a; b] và

'( ) '( )f a f b= thì bất phương trình 4

4''( ) ( ) '( )

( )f x f a f b

a b≥ −

− có ít nhất một nghiệm.

3. Tìm 1

1 1

2 1 sin2

n

i

Limin

n

π=

+

4. Cho dãy số thực (xn) xác ñịnh bởi: 1

21 ln 1 2010, 1n n

x a

x x n+

=

= + − ∀ ≥.

Chứng minh rằng xn có giới hạn.

5. Cho phương trình: 1

11.

n

i i nx=

=+

Chứng minh rằng: Với mỗi số nguyên dương n phương trình có duy nhất một

nghiệm dương. Kí hiệu nghiệm ñó là xn, tìm limxn.

6. Chứng minh 1b aa b+ > với mọi , 0a b > .

7. Cho ña thức P(x) và Q(x) = aP(x) + bP’(x) + cP”(x) trong ñó a, b, c là các số

thực thỏa mãn a ≠ 0 và b2 – 4ac > 0. Chứng minh rằng nếu Q(x) vô nghiệm thì P(x) vô

nghiệm.

8. Cho số thực a khác không, ña thức ( ), deg ( ) 1P x P x n= ≥ và ña

thức 2 ( )( ) ( ) '( ) "( ) ... ( )n nQ x P x aP x a P x a P x= + + + + . Chứng minh rằng nếu ( )P x vô nghiệm

thì ( )Q x cũng vô nghiệm.

Page 91: Kỷ yếu hội thảo các trường chuyên dh db bắc bộ lần III

=========================================================== 93

HỘI CÁC TRƯỜNG THTP CHUYÊN KHU VỰC DUYÊN HẢI VÀ ðỒNG BẰNG BẮC BỘ

Hội thảo khoa học môn Toán học lần thứ III - 2010

TỈ SỐ KÉP VÀ PHÉP CHIẾU XUYÊN TÂM

Trường THPT chuyên Thái Bình – Thái Bình Lời nói ñầu: Phép chiếu xuyên tâm và tỉ số kép là một phần rất ñẹp của hình học.Tài liệu nhỏ này xin

ñưa ra một số ví dụ sử dụng phép chiếu xuyên tâm và tỉ số kép trong giải toán hình học.

Phần 1: Sơ lược về lý thuyết: ðịnh lý 1. Cho tứ giác ABCD với các ñiểm chéo S, O, K. Giả sử SO cắt AD tại M, cắt

BC tại N. Khi ñó .

ðịnh lý 2. Cho hai hàng : Khi ñó

song song hoặc ñồng quy.

ðịnh lý 3. Cho hai chùm và :

Giả sử Khi ñó thẳng hàng.

Page 92: Kỷ yếu hội thảo các trường chuyên dh db bắc bộ lần III

=========================================================== 94

HỘI CÁC TRƯỜNG THTP CHUYÊN KHU VỰC DUYÊN HẢI VÀ ðỒNG BẰNG BẮC BỘ

Hội thảo khoa học môn Toán học lần thứ III - 2010

ðịnh lý 4. Cho chùm bốn ñường thẳng Khi ñó

Hệ quả. Cho tứ giác ABCD nội tiếp ñường tròn (O). Khi ñó, với mọi ñiểm M trên (O), tỉ số không ñổi.

ðịnh nghĩa 1. Phép chiếu xuyên tâm. Cho (d). S ở ngoài (d). Với mỗi ñiểm M, SM cắt (d) tại M’(M không thuộc ñường thẳng

qua S song song (d)). Vậy M→M’ là phép chiếu xuyên tâm với tâm chiếu S lên (d) Tiếp theo ta sẽ phát biểu một ñịnh lí quan trọng về phép chiếu xuyên tâm ðịnh lí 5. Phép chiếu xuyên tâm bảo toàn tỉ số kép Phần 2:Các ví dụ:

1.Cho hai tam giác ABC và có

Chứng minh rằng thẳng hàng khi và chỉ khi ñồng quy hoặc song song.

Page 93: Kỷ yếu hội thảo các trường chuyên dh db bắc bộ lần III

=========================================================== 95

HỘI CÁC TRƯỜNG THTP CHUYÊN KHU VỰC DUYÊN HẢI VÀ ðỒNG BẰNG BẮC BỘ

Hội thảo khoa học môn Toán học lần thứ III - 2010

Giải. Giả sử cắt tại . Ta có

Do nên thẳng

hàng.

2.Cho tứ giác ABCD, O là giao ñiểm của hai ñường chéo. ðường thẳng d ñi qua O cắt các ñường thẳng AB, BC, CD, DA tại P, N, Q, M. Chứng minh rằng

(MNPO)=(MNOQ) Giải.

Lần lượt chiếu tâm A và D lên BC.

Chú ý: Có thể yêu cầu chứng minh O là trung ñiểm của MN khi và chỉ khi O là trung ñiểm của PQ.

3.Cho tam giác ABC và các ñiểm M, N trên cạnh BC sao cho M nằm giữa B và N. Gọi là tâm các ñường tròn nội tiếp và bàng tiếp góc A của tam giác ABM; là tâm

ñường tròn bàng tiếp và nội tiếp góc A của tam giác CAN. Chứng minh rằng và

ñồng quy.

S

M

Q

N

P

O

D C

B

A

Page 94: Kỷ yếu hội thảo các trường chuyên dh db bắc bộ lần III

=========================================================== 96

HỘI CÁC TRƯỜNG THTP CHUYÊN KHU VỰC DUYÊN HẢI VÀ ðỒNG BẰNG BẮC BỘ

Hội thảo khoa học môn Toán học lần thứ III - 2010

Giải. Chú ý rằng . Gọi . Sử dụng phép

chiếu tâm O ta suy ra

4.(ðịnh lý Papuyt) Cho hai ñường thẳng và . Trên cho ba ñiểm , trên

cho ba ñiểm . Giả sử

Chứng minh rằng thẳng hàng.

Giải. Lần lượt chiếu xuyên tâm và lên , ta ñược Từ ñó

suy ra ñồng quy và ta có ñiều phải chứng minh.

5.Cho tứ giác ABCD, Trên cạnh AB lấy ñiểm P, trên cạnh

BC lấy ñiểm M. Giả sử AM cắt CD tại N, CP cắt AD tại Q, MP cắt QN tại E. Chứng minh rằng S, K, E thẳng hàng.

Page 95: Kỷ yếu hội thảo các trường chuyên dh db bắc bộ lần III

=========================================================== 97

HỘI CÁC TRƯỜNG THTP CHUYÊN KHU VỰC DUYÊN HẢI VÀ ðỒNG BẰNG BẮC BỘ

Hội thảo khoa học môn Toán học lần thứ III - 2010

Giải. Sử dụng phép chiếu xuyên tâm C, ta ñược . Suy ra

Do , , ,MA NA MB NS S MK ND K MP NQ E≡ ∩ = ∩ = ∩ = nên thẳng hàng.

Lời giải 2. Chiếu xuyên tâm và ta có

Suy ra ñồng qui.

Page 96: Kỷ yếu hội thảo các trường chuyên dh db bắc bộ lần III

=========================================================== 98

HỘI CÁC TRƯỜNG THTP CHUYÊN KHU VỰC DUYÊN HẢI VÀ ðỒNG BẰNG BẮC BỘ

Hội thảo khoa học môn Toán học lần thứ III - 2010

6.Cho tứ giác ABCD, O là giao của hai ñường chéo. Một ñường thẳng d ñi qua O cắt AD, BC, AB, CD tại M, N, P, Q. Giả sử và

. Chứng minh rằng X, Y, Z, T thẳng hàng.

Cách giải 1. Gọi Ta chứng minh Gọi giao ñiểm

của XO với AD, BC là R và G ; giao ñiểm của KO với AD và BC là E và F. Ta có Do nên thẳng hàng.

Cách giải 2. Sử dụng phép chiếu tâm A lên CD và phép chiếu tâm B lên AD ta suy ra

Page 97: Kỷ yếu hội thảo các trường chuyên dh db bắc bộ lần III

=========================================================== 99

HỘI CÁC TRƯỜNG THTP CHUYÊN KHU VỰC DUYÊN HẢI VÀ ðỒNG BẰNG BẮC BỘ

Hội thảo khoa học môn Toán học lần thứ III - 2010

Suy ra EA, CM, KS ñồng quy. Vậy Cách giải 3. Sử dụng tỉ số kép của chùm

Ta có : . Suy ra K, S, X thẳng

hàng. 7.Cho tam giác và ñiểm nằm trong tam giác. Giả sử

ðường thẳng ñi qua , song song với cắt tại Chứng minh rằng

là trung ñiểm của

Giải. Ta có Do ñó

8.Cho tam giác ABC, ñường cao AH. Trên AH lấy ñiểm M, BM cắt AC tại K, CM cắt AB tại E. Chứng minh rằng AH là phân giác của góc

Page 98: Kỷ yếu hội thảo các trường chuyên dh db bắc bộ lần III

=========================================================== 100

HỘI CÁC TRƯỜNG THTP CHUYÊN KHU VỰC DUYÊN HẢI VÀ ðỒNG BẰNG BẮC BỘ

Hội thảo khoa học môn Toán học lần thứ III - 2010

Giải. Áp dụng tính chất ta có . Từ ñó suy ra ñiều phải chứng

minh. 9. (Iran) Cho tam giác và ñiểm nằm trong tam giác. Giả sử

Kẻ Chứng minh rằng là phân giác của góc

Giải. Ta có Do ñó ta có ñpcm.

10. Cho tam giác và ñiểm nằm trong tam giác. Giả sử

Kẻ ðường thẳng ñi qua , song song với cắt tại .

Chứng minh rằng là phân giác của góc

Page 99: Kỷ yếu hội thảo các trường chuyên dh db bắc bộ lần III

=========================================================== 101

HỘI CÁC TRƯỜNG THTP CHUYÊN KHU VỰC DUYÊN HẢI VÀ ðỒNG BẰNG BẮC BỘ

Hội thảo khoa học môn Toán học lần thứ III - 2010

11. Cho tam giác , các ñường cao . Gọi là giao ñiểm của với ,

là trung ñiểm của . ðường thẳng ñi qua song song với cắt tại .

Chứng minh rằng bốn ñiểm cùng thuộc một ñường tròn.

Giải. Chú ý rằng . Sử dụng hệ thức Macloranh ta có ñiều phải chứng

minh.

12. Cho tam giác trực tâm . ðường thẳng ñi qua cắt tại

; ñường thẳng ñi qua , vuông góc với cắt tại . Các

ñiểm chia theo cùng tỉ số . Chứng minh rằng thẳng

hàng.

Page 100: Kỷ yếu hội thảo các trường chuyên dh db bắc bộ lần III

=========================================================== 102

HỘI CÁC TRƯỜNG THTP CHUYÊN KHU VỰC DUYÊN HẢI VÀ ðỒNG BẰNG BẮC BỘ

Hội thảo khoa học môn Toán học lần thứ III - 2010

YX

C2

B2

A2

C1

B1

A1

H

CB

A

Giải. ðể chứng minh thẳng hàng ta dựa vào nhận xét sau :

C0

B0

A0

C2

B2

A2

C1

B1

A1

Cho hai hàng và thỏa mãn : Trên

lấy , trên lấy , trên lấy sao cho

Khi ñó, thẳng hàng.

Trở lại bài toán, ta chứng minh Thật vậy, ta có

trong ñó song song với

Ta chứng minh

Qua , kẻ các ñường thẳng song song với . Ta có :

ðiều này ñúng, do hai chùm trên có các ñường tương ứng vuông góc.

13.(Bài toán con bướm) Cho tứ giác ABCD nội tiếp ñường tròn (O), .

Một ñường thẳng ñi qua K cắt các cạnh AB, CD tại M và N, cắt ñường tròn (O) tại P và Q. Chứng minh rằng K là trung ñiểm của PQ khi và chỉ khi K là trung ñiểm của MN.

Page 101: Kỷ yếu hội thảo các trường chuyên dh db bắc bộ lần III

=========================================================== 103

HỘI CÁC TRƯỜNG THTP CHUYÊN KHU VỰC DUYÊN HẢI VÀ ðỒNG BẰNG BẮC BỘ

Hội thảo khoa học môn Toán học lần thứ III - 2010

O

KQP

NM

D

CB

A

Giải. Ta có . Từ ñó có ñpcm.

14.(ðịnh lý Pascal). Ta có . Từ ñó suy ra ñiều phải chứng

minh.

O

FE

ZY

X

B'

A'C'

C

B

A

Chú ý. Xét các trường hợp ñặc biệt (cho các ñỉnh trùng nhau) ta ñược các kết quả

thú vị. 15.Cho tam giác SAB và ñiểm O nằm trong tam giác. Các ñường thẳng BO, AO cắt SA,

SB tại M và N. Một ñường thẳng qua O cắt các ñoạn MA, NB tại P và Q. Chứng minh rằng

Giải. ðiều phải chứng minh tương ñương với

Xét phép chiếu tâm O ta có Do ñó

Suy ra hoặc .

Giả sử . Khi ñó, (ñpcm).

16. (China TST 2002). Cho tứ giác lồi ABCD. Cho E AB CD, F AD BC, P AC BD= ∩ = ∩ = ∩ . O là chân ñường vuông góc hạ từ P xuống EF. Chứng minh rằng AOD=BOC.

Bg: BD cắt È tại T .(ETFS)=-1.Phép chiếu tâm B lên SC suy ra (APCS)=-1. Phép chiếu xuyên tâm E lên BD,(BPDT)=-1. O(BPDT)=-1 mà OP vuông góc với OT suy ra OP là phân giác góc BOD. O(APCS)=-1 mà OP vuông góc với OS suy ra OP là phân giác góc AOC.Có ðPCM.

QP

NM

O

BA

S

Page 102: Kỷ yếu hội thảo các trường chuyên dh db bắc bộ lần III

=========================================================== 104

HỘI CÁC TRƯỜNG THTP CHUYÊN KHU VỰC DUYÊN HẢI VÀ ðỒNG BẰNG BẮC BỘ

Hội thảo khoa học môn Toán học lần thứ III - 2010

17.(Bucharest 2006) Cho tam giác ABC cân tại A. M là trung ñiểm BC. Tìm quỹ tích các ñiểm P nằm trong tam giác thỏa mãn góc BPM và CPA bù nhau.

Bg: AP cắt ñường tròn ngoại tiếp tam giác BPC và BC lần lượt tại D và S.Giả thiết suy ra

góc BPS=CPM.Áp dụng ñịnh lý Steiner cho hai ñường ñẳng giác PS và PM của tam giác BPC suy ra SB/SC=PB2/PC2.Lại có SB/SC=DB/DC. PB/PC.Suy ra PBDC là tứ giác ñiều hòa.Dẫn ñến tiếp tuyến tại B và Cvà PD ñồng quy tại A’.

Nếu A trùng A’thì P thuộc cung BIC (I là tâm nội tiếp tam giác ABC) Nếu A khác A’ thì P thuộc ñường thẳng AM.

18.Cho tứ giác ABCD. E AB CD, F AD BC,G AC BD= ∩ = ∩ = ∩ . EF AD, AB M, N∩ = . Chứng minh rằng (EMGN) 1= − .

Chứng minh.

Xét phép các phép chiếu: A: E B,G C, M F, N N→ → → → ⇒ ( ) ( )EGMN BCFN=

D: E C,G B, M F, N N (EGMN) (CBFN)→ → → → ⇒ =

( )BCFN (CBFN)

1(BCFN)

(BCFN)

⇒ =

⇔ =

(BCFN) 1⇔ = − (do (BCFN) 1≠ )

Vậy ( )EGMN 1= − (d.p.c.m)

Kết luận :Qua một số ví dụ trên phần nào cho thấy vẻ ñẹp của phép chiếu xuyên tâm và tỉ số kép trong giải toán hình học.Tài liệu còn sơ sài ,chúng tôi kính mong nhận ñược sự thể tất và mong nhận ñược góp ý của quý ñồng nghiệp ñể tập tài liệu này phong phú hơn.

Tài liệu tham khảo: 1) Harmonic_division Virgil Nicula. 2) Tỉ số kép.Phạm Minh Phương 3) Tài liệu giáo khoa chuyên toán 10 Hình học.Nguyễn Minh Hà. 4) Hàng ñiểm ñiều hòa.Kim Luân.Nguyễn ðình Thành Công.Lê Nam Trường.

Page 103: Kỷ yếu hội thảo các trường chuyên dh db bắc bộ lần III

=========================================================== 105

HỘI CÁC TRƯỜNG THTP CHUYÊN KHU VỰC DUYÊN HẢI VÀ ðỒNG BẰNG BẮC BỘ

Hội thảo khoa học môn Toán học lần thứ III - 2010

MỘT SỐ DẠNG TOÁN VỀ DÃY SỐ VÀ GIỚI HẠN Trần Ngọc Thắng - THPT Chuyên Vĩnh Phúc 1 Giới hạn dãy số 1.1 Dãy số ðịnh nghĩa 1.1. Dãy số (thực) là một hàm số xác ñịnh trên tập con của tập số tự nhiên

Với ⊂ �M , thay cho ký hiệu :u → �M

( )n u na

ta thường dùng ký hiệu ( ) ,{ } , ( )n n n n n nu u u∈ ∈M M hay { }n nu

ðịnh nghĩa 1.2. Cho dãy ( )n nu ∈�

• Dãy ( )nu ñược gọi là dãy (ñơn ñiệu) tăng nếu 1n nu u n+≤ ∀ ∈�

• Dãy ( )nu ñược gọi là dãy (ñơn ñiệu) giảm nếu 1n nu u n+≥ ∀ ∈�

• Dãy ( )nu ñược gọi là dãy (ñơn ñiệu) tăng nghiêm ngặt nếu 1n nu u n+< ∀ ∈�

• Dãy ( )nu ñược gọi là dãy (ñơn ñiệu) giảm nghiêm ngặt nếu 1n nu u n+> ∀ ∈�

Nhận xét. • Nếu ( ) , ( )n nx y� � thì ( )n nx y+ �

• Nếu ( ) , ( )n nx y� � thì ( )n nx y+ �

• Nếu ( )nx � thì ( )nx− � . Và nếu ( )nx � thì ( )nx− �

• Nếu hai dãy dương ( ), ( )n nx y cùng tăng (giảm) thì ( )n nx y tăng (giảm).

• Một dãy có thể không tăng, cũng không giảm. Ví dụ ( 1)n

nx n= − ∀ ∈�

ðịnh nghĩa 1.3. Cho dãy số ( )n nx ∈� .

• Dãy ( )nx ñược gọi là bị chặn trên, nếu tồn tại hằng số M sao cho nx M n≤ ∀

• Dãy ( )nx ñược gọi là bị chặn dưới, nếu tồn tại hằng số m sao cho nx m n≥ ∀

• Dãy ( )nx vừa bị chặn trên, vừa bị chặn dưới ñược gọi là bị chặn.

ðịnh lí 1.1. Dãy ( )nx bị chặn khi và chỉ khi tồn tại ghằng số 0c ≥ sao cho | |nu c n≤ ∀

1.2 Giới hạn của dãy số ðịnh nghĩa 1.4. Dãy số ( )nu ñược gọi là hội tụ vềα , ký hiệu lim n

nu α

→∞= , nếu với mọi

0ε > cho trước tùy ý, tìm ñược chỉ số 0n sao cho với mọi 0n n≥ ñều có | |nu α ε− <

Ví dụ 1.1. Chứng minh rằng

1. limn

c c→∞

=

2. 1

lim 0n n→∞

=

3. 1

lim 1n

n

n→∞

+=

ðịnh lí 1.2. (Tính duy nhất của giới hạn) Giới hạn của một dãy hội tụ là duy nhất

Page 104: Kỷ yếu hội thảo các trường chuyên dh db bắc bộ lần III

=========================================================== 106

HỘI CÁC TRƯỜNG THTP CHUYÊN KHU VỰC DUYÊN HẢI VÀ ðỒNG BẰNG BẮC BỘ

Hội thảo khoa học môn Toán học lần thứ III - 2010

ðịnh lí 1.3. (Tính thứ tự của dãy hội tụ) Cho lim n

nx

→∞= l và a ∈� . Khi ñó

• Nếu a < l thì 0 0( : )nn n n a x∃ ∈ ∀ ≥ ⇒ <�

• Nếu a > l thì 0 0( : )nn n n a x∃ ∈ ∀ ≥ ⇒ >�

ðịnh lí 1.4. (Chuyển qua giới hạn trong bất ñẳng thức) Cho lim nn

x→∞

= l và a ∈� .

Khi ñó

• Nếu 0 0( : )nn n n x a∃ ∈ ∀ ≥ ⇒ ≥� thì a≥l

• Nếu 0 0( : )nn n n x a∃ ∈ ∀ ≥ ⇒ ≤� thì a≤l

ðịnh lí 1.5. (ðịnh lý giới hạn kẹp giữa) Cho ba dãy số ( ), ( ), ( )n n nx y z thỏa mãn

• 0 0: n n nn n n z x y∃ ∈ ∀ ≥ ⇒ ≤ ≤�

• các dãy ( ), ( )n ny z cùng hội tụ ñến l

Khi ñó dãy ( )nx hội tụ và lim nn

x→∞

= l

ðịnh lí 1.6. (Tính chất ñại số của dãy hội tụ) Cho hai dãy hội tụ ( ), ( )n nx y và lim ; limn n

n nx a y b

→∞ →∞= = . Khi ñó

• Dãy ( )nx− hội tụ và lim( )nn

x a→∞

− = −

• Dãy (| |)nx hội tụ và lim | | | |nn

x a→∞

=

• Dãy ( )n nx y+ hội tụ và lim( )n nn

x y a b→∞

+ = +

• Dãy ( )n nx y− hội tụ và lim( )n nn

x y a b→∞

− = −

• Dãy ( )nkx hội tụ và lim( )nn

kx ka→∞

=

• Dãy ( · )n nx y hội tụ và lim( · )n nn

x y ab→∞

=

• Với 0b ≠ thì dãy 1

ny

ñược xác ñịnh từ một chỉ số nào ñó, hội tụ và

1 1limn

ny b→∞

=

• Với 0b ≠ thì dãy n

n

x

y

ñược xác ñịnh từ một chỉ số nào ñó, hội tụ và

lim n

nn

x a

y b→∞

=

Ví dụ 1.2. Tìm các giới hạn sau

• 2

2

3 2lim

3 2n

n n

n n→∞

− +

+ −

• 2

3 2

2 3 1lim

3 4 5n

n n

n n→∞

− +

+ −

• 2 1 2

limn

n n

n→+∞

+ − −

• 2 2lim ( 2 1 1)n

n n n n→+∞

+ − − + +

Page 105: Kỷ yếu hội thảo các trường chuyên dh db bắc bộ lần III

=========================================================== 107

HỘI CÁC TRƯỜNG THTP CHUYÊN KHU VỰC DUYÊN HẢI VÀ ðỒNG BẰNG BẮC BỘ

Hội thảo khoa học môn Toán học lần thứ III - 2010

• 0

0

(3 1)lim

(2 3)

n

k

nn

k

k

k

=

→+∞

=

+

+

1.3 Dấu hiệu hội tụ của dãy số 1.3.1 Tiêu chuẩn Weiersstrass ðịnh lí 1.7. Một dãy số ñơn ñiệu và bị chặn thì hội tụ

Cụ thể, một dãy ñơn ñiệu tăng và bị chặn trên thì hội tụ, một dãy ñơn ñiệu giảm và bị chặn dưới thì hội tụ.

Ví dụ 1.3. Cho các dãy số ( ), ( )n nx y ñược xác ñịnh như sau

1 1 1 10, 0, , , 1.2

n nn n n n

x yx a y b x x y y n+ +

+= > = > = = ∀ ≥

Chứng minh rằng các dãy số ( ), ( )n nx y hội tụ và lim limn nx y= .

Lời giải. Ta xét hai trường hợp sau: (i) Nếu a b≥ thì bằng quy nạp ta chỉ ra ñược dãy ( )nx là dãy giảm bị chặn dưới bởi a ,

còn dãy ( )ny là dãy tăng bị chặn trên bởi a . Do ñó theo ñịnh lý 1.7 tồn tại lim , limn nx y và

từ giả thiết chuyển qua giới hạn ta ñược lim limn nx y= .

(ii) Nếu a b≤ tương tự như trường hợp (i). Ví dụ 1.4. Cho dãy số ( )nx ñược xác ñịnh như sau

1 2 2 11, 2, , 1n n n

x x x x x n+ += = = + ∀ ≥ .

Chứng minh rằng dãy số ñã cho có giới hạn và tìm giới hạn ñó. Lời giải. Dễ thấy bằng quy nạp ta chỉ ra ñược ( )nx là dãy số tăng và bị chặn trên bởi 4.

Do ñó theo ñịnh lý 1.7 ta có tồn tại lim nx a= . Từ ñẳng thức 2 1n n nx x x+ += + chuyển qua

giới hạn ta ñược 2a a= nhưng do 0a > nên chỉ lấy 4a = . Vậy lim 4na = .

Bài tập tương tự

Bài tập 1.5. Cho dãy số ( )nx xác ñịnh bởi 1 12, 2 , 1,2,n nx x x n+= = + = …Chứng

minh rằng dãy số ñã cho hội tụ và tìm lim nn

x→∞

.

Bài tập 1.6. Cho dãy số thỏa mãn ñiều kiện

( )1

10 1, 1

4n n nx x x+< < − > .

Chứng minh dãy số trên hội tụ và tìm giới hạn ñó. Bài tập 1.7. (ðịnh lý Cantor) Cho hai dãy số thực ( ), ( )n na b thỏa mãn các ñiều kiện sau:

[ ] [ ]1 1; , ,n n n n n na b a b a b+ +≤ ⊆ với mọi n ∈� và ( )lim 0n nb a− = .

Page 106: Kỷ yếu hội thảo các trường chuyên dh db bắc bộ lần III

=========================================================== 108

HỘI CÁC TRƯỜNG THTP CHUYÊN KHU VỰC DUYÊN HẢI VÀ ðỒNG BẰNG BẮC BỘ

Hội thảo khoa học môn Toán học lần thứ III - 2010

Khi ñó tồn tại số thực c sao cho [ ] { }0

,n n

n

a b c∞

=

=I và lim limn na b c= = .

Bài tập 1.8. (VMO 2005). Cho dãy số thực ( ), 1, 2,3...nx n = ñược xác ñịnh bởi:

1x a= và 3 21 3 7 5n n n nx x x x+ = − + với 1, 2,3,...n =

trong ñó a là một số thực thuộc ñoạn4

0,3

.

Chứng minh rằng dãy số ( )nx có giới hạn hữu hạn và tìm giới hạn ñó.

Bài tập 1.9. (VMO 2002B). Xét phương trình

2 2

1 1 1 1 1... ... 0

2 1 4x x x x k x n+ + + + + + =

− − − −,

trong ñó n là tham số nguyên dương. 1. Chứng minh rằng với mỗi số nguyên dương n , phương trình nêu trên có duy nhất

nghiệm trong khoảng ( )0,1 ; kí hiệu nghiệm ñó là nx .

2. Chứng minh rằng dãy số nx có giới hạn hữu hạn khi n → +∞ .

Bài tập 1.10. Cho số thực a . Cho dãy số ( ),nx n ∈� , ñược xác ñịnh bởi:

0x a= và 1 sinn n nx x x+ = + với mọi n ∈� .

Chứng minh rằng dãy số ( )nx có giới hạn hữu hạn khi n → +∞ và tính giới hạn ñó.

1.3.2 Tiêu chuẩn Cauchy

ðịnh nghĩa 1.5. Dãy ( )nx ñược gọi là dãy Cauchy nếu thỏa mãn ñiều kiện

0, : , , , , m nN m n m n N x xε ε∀ > ∃ ∈ ∀ ∈ ≥ − <� �

ðịnh lí 1.8. Dãy số ( )nx hội tụ khi và chỉ khi ( )nx là dãy Cauchy.

Ví dụ 1.11. Cho hàm số :f →� � thỏa mãn ñiều kiện

( ) ( )f x f y q x y− ≤ − , với mọi ,x y ∈� ,

trong ñó ( )0,1q ∈ là hằng số cho trước. Với c ∈� cho trước và xác ñịnh dãy

( ), 0,1, 2,3...nx n = như sau: 0 1, ( ), 0,1,2,...n nx c x f x n+= = = . Chứng minh rằng dãy số ( )nx

hội tụ và giới hạn của dãy số là nghiệm của phương trình ( )f x x= . Lời giải Trước hết ta chứng minh dãy ( )nx là một dãy Cauchy. Thật vậy, với

, ,m n n m∈ >� ta có:

( )( )1 1 1 1 0... m

n m n m n m n mx x f x f x q x x q x x− − − − −− = − ≤ − ≤ ≤ −

(1) Mặt khác ta có

( )10 1 1 0 1 0 1 0

1... ... 1

1

nn

n n n

qx x x x x x q x x x x

q

−−

−− ≤ − + + − ≤ + + − = −

−.

Từ ñây suy ra 0nx x− bị chặn với mọi n . Kết hợp với (1) ta thu ñược với mọi 0ε >

tồn tại N ∈� sao cho với mọi ,m n N≥ thì n mx x ε− < . Nên dãy ( )nx là một dãy Cauchy

suy ra nó hội tụ.

Page 107: Kỷ yếu hội thảo các trường chuyên dh db bắc bộ lần III

=========================================================== 109

HỘI CÁC TRƯỜNG THTP CHUYÊN KHU VỰC DUYÊN HẢI VÀ ðỒNG BẰNG BẮC BỘ

Hội thảo khoa học môn Toán học lần thứ III - 2010

Từ ñiều kiện của hàm f dễ dàng chứng minh ñược f liên tục và do ñó từ ñẳng thức

1( )n nx f x −= chuyển qua giới hạn ta ñược giới hạn của dãy ( )nx là nghiệm của phương trình

( )f x x= .

Bài tập tương tự

Bài tập 1.12. Cho :f →� � thỏa mãn ñiều kiện với mọi 0ε > ñều tồn tại 0δ > sao

cho: nếu x yε ε δ≤ − < + thì ( ) ( )f x f y ε− < . Xét dãy số xác ñịnh như sau:

0 1, ( ), 0,1,...n nx x f x n+∈ = =� Chứng minh rằng dãy ( )nx hội tụ.

Bài tập 1.13. Cho :f →� � thỏa mãn ñiều kiện ( ) ( ) ( ( ))x f x x f xϕ ϕ− ≤ − , trong ñó

:ϕ →� � là hàm liên tục và bị chặn dưới. Lấy 0x ∈� và lập dãy 1 ( ), 0,1,2,...n nx f x n+ = =

Chứng minh rằng dãy số ( )nx hội tụ.

Bài tập 1.14. Cho :f →� � thỏa mãn ñiều kiện

( )( ) ( ) ( ) ( )f x f y k x f x y f y− ≤ − + − , với mọi ,x y ∈� , trong ñó 1

2k < . Xét dãy số xác

ñịnh như sau: 1 1, ( ), 1n nx x f x n+∈ = ≥� . Chứng minh rằng dãy ( )nx hội tụ và giới hạn của

dãy là nghiệm duy nhất của phương trình ( )f x x= . Bài tập 1.15. Cho :f →� � thỏa mãn ñiều kiện: có số , 0 1k k≤ < sao cho

{ }( ) ( ) max , ( ) , ( ) , .f x f y k x y f x x y f y x y− ≤ − − − ∀ ∈�

Xét dãy số xác ñịnh như sau: 1 1, ( ), 1n nx x f x n+∈ = ≥� . Chứng minh rằng dãy ( )nx hội

tụ và giới hạn của dãy là nghiệm duy nhất của phương trình ( )f x x= .

1.3.3 Nguyên lý kẹp ðịnh lí 1.9. Cho ba dãy số ( ), ( )n na b và ( )nc thỏa mãn: N∃ ∈� sao cho

n n na b c n N≤ ≤ ∀ ≥ và lim limn na c a= = . Khi ñó nlimb a= .

Ví dụ 1.16. (Canada 1985) Dãy số ( )nx thỏa mãn ñiều kiện 11 2x< < và

2 *1

11 , .

2n n nx x x n+ = + − ∀ ∈�

Chứng minh rằng dãy số ñã cho hội tụ. Tìm lim nn

x→∞

Lời giải. Ta sẽ chứng minh bằng quy nạp bất ñẳng thức sau:1

2 , 32n n

x n− < ∀ ≥ . Thật

vậy ta kiểm tra ñược ngay bất ñẳng thức ñúng với 3n = . Giả sử bất ñẳng thức ñúng với

3n ≥ , tức là 1

22n n

x − < . Khi ñó ta có

( )1

1

1 12 2 2 2 2 2 2 2 2

2 21 1 1 1

2 .2 2 2 2

n n n n n

n n n

x x x x x

x

+

+

− = − − − ≤ − − + −

< − < =

Page 108: Kỷ yếu hội thảo các trường chuyên dh db bắc bộ lần III

=========================================================== 110

HỘI CÁC TRƯỜNG THTP CHUYÊN KHU VỰC DUYÊN HẢI VÀ ðỒNG BẰNG BẮC BỘ

Hội thảo khoa học môn Toán học lần thứ III - 2010

Do ñó bất ñẳng thức ñúng ñến 1n + . Mặt khác do 1

lim 02n

= nên từ bất ñẳng thức trên

và nguyên lý kẹp ta có lim 0nx = .

Ví dụ 1.17. Cho dãy các hàm số { }( )nP x xác ñịnh như sau

2

0 1

( )( ) 0, ( ) ( ) , 0;

2n

n n

x P xP x P x P x n x+

−= = + ∀ ≥ ∈� .

Tìm lim ( )nn

P x→∞

.

Lời giải Trước hết ta chứng minh bằng quy nạp bất ñẳng thức

sau: 0 ( ) ,nP x x n≤ ≤ ∀ ∈� .

(1)

Thật vậy, với [0,1]x ∈ suy ra 2 0x x− ≤ nên 10 ( )2

xP x x≤ = ≤ . Như vậy (1) ñúng

với n=1. Giả sử (1) ñúng ñến $n$. Xét hàm số ( )21( )

2f t t x t= + − với [0,1]t ∈ . Dễ thấy hàm

số ( )f t ñồng biến trên [0,1] . theo giả thiết quy nạp ta có 0 ( ) 1nP x x≤ ≤ ≤ với mọi

[0,1]x ∈ (2)

nên 1( ) ( ( )) ( )n nP x f P x f x x+ = ≤ = với mọi [0,1]x ∈ . Mặt khác, từ (2) ta có 2

1( ) 0 ( ) ( ) 0n n nx P x P x P x+− ≥ ⇒ ≥ ≥ . Vậy 10 ( )nP x x+≤ ≤ . Do ñó (1) ñúng ñến 1n + nên

theo nguyên lý quy nạp ta có (1) ñúng với mọi n .

Tiếp theo ta chứng minh 2

( )1nx P x

n− ≤

+ với [0,1],x n∈ ∀ ∈� .

(3)

Thật vậy ta có 11

( )( ) ( ) 1

2n

n n

x P xx P x x P x −

+ − = − −

1 1

0

1

( ) 1 ( ( ) 0)2

2... ( ) 1 1

2 2 2

12 22 2 2

.1 1 1 1

n n

n n

n

n

xx P x do P x

x n x xx P x

n

n x xn

n

n n n n n

− −

+

≤ − − ≥

≤ ≤ − − = −

+ −

≤ = < + + + +

Từ ñó ta thu ñược bất ñẳng thức 2

0 ( )1nx P x

n≤ − <

+ với mọi [0,1]x n∈ ∀ ∈� .

Do 2

lim 01n

=+

nên theo nguyên lý kẹp ta ñược lim ( )nP x x= , với mọi [0,1]x ∈ .

Page 109: Kỷ yếu hội thảo các trường chuyên dh db bắc bộ lần III

=========================================================== 111

HỘI CÁC TRƯỜNG THTP CHUYÊN KHU VỰC DUYÊN HẢI VÀ ðỒNG BẰNG BẮC BỘ

Hội thảo khoa học môn Toán học lần thứ III - 2010

Ví dụ 1.18. Cho { }, , ( , ) 1; 1, 2,...a b a b n ab ab∈ = ∈ + +�å . Kí hiệu

nr là số cặp số

( , )u v ∈ ×� �å å sao cho n au bv= + . Chứng minh rằng

1lim n

n

r

n ab→∞= .

Lời giải Xét phương trình au bv n+ = (1). Gọi 0 0( , )u v là một nghiệm nguyên dương

của (1). Giả sử ( , )u v là một nghiệm nguyên dương khác 0 0( , )u v của (1). Ta có

0 0 ,au bv n au bv n+ = + = suy ra 0 0( ) ( ) 0a u u b v v− + − = do ñó tồn tại k nguyên dương sao

cho 0 0,u u kb v v ka= + = − . Do v là số nguyên dương nên 00

11

vv ka k

a

−− ≥ ⇔ ≤ . (2)

Ta nhận thấy số nghiệm nguyên dương của phương trình (1) bằng số các số k nguyên

dương cộng với 1. Do ñó 0 01 11 1n

v unr

a ab b a

− = + = − − +

. Từ ñó ta thu ñược bất ñẳng

thức sau:

0 01 11.n

u un nr

ab b a ab b a− − ≤ ≤ − − +

Từ ñó suy ra

0 01 1 1 1 1.nu r u

ab nb na n ab nb na n− − ≤ ≤ − − +

Từ ñây áp dụng nguyên lý kẹp ta có ngay 1

lim n

n

r

n ab→∞= .

Ví dụ 1.19. Tìm giới hạn của dãy số ( )nx biết

1 2 1 3 1 1 ( 1) 1nx n n= + + + + − +L

Lời giải 1.

Xét hàm số ( ) 1 1 (1 )f x x x= + + + L ta chứng minh 1

( ) 2( 1)2

xf x x

+≤ ≤ + . Từ ñó

1 1

2 22 (1 ) ( ) 2 (1 )n n

x f x x−

+ ≤ ≤ + .

Từ ñó, thay 2x = ñược 1 1

2 23·2 3·2n n

nx−

< < . Từ ñó, theo nguyên lý kẹp, suy ra

lim 3nn

x→∞

= .

Lời giải 2. Với1 1m n≤ ≤ − , ñặt 1 1 (1 ) 1 1 ( 1) 1ma m m n n= + + + + + − +L ta có 2 2 2 2

1 1

2 21

1 ( 1) 2

( 1) ( ( 2))

m m m m

m m

a ma a m ma m m

a m m a m

+ +

+

= + − + = − −

⇔ − + = +.

Suy ra

1 21

| || ( 1) | | 2 |

| ( 1) | 2m m

m m

m

m a a ma m a m

a m m

+ ++

−− + ≤ ≤ − +

+ + +.

Từ ñó 2 1

1 1| 3 | | | | 1 ( 1) 1 | 0 ( )

1 1n

n na a n n n n n

n n−

− −− ≤ − < + − + − → → ∞

+ +

Lời giải 3. ðể ý rằng

Page 110: Kỷ yếu hội thảo các trường chuyên dh db bắc bộ lần III

=========================================================== 112

HỘI CÁC TRƯỜNG THTP CHUYÊN KHU VỰC DUYÊN HẢI VÀ ðỒNG BẰNG BẮC BỘ

Hội thảo khoa học môn Toán học lần thứ III - 2010

n&3 1 2·4 1 2· 16 1 2 1 3 25 1 2 1 3 1 4 36= + = + = + + = + + + bằng quy nạp,

dễ dàng chứng minh ñược

21 2 1 3 1 1 ( 2) 3n n+ + + + + =L

Suy ra 3nx ≤

(1)

Nhận xét. Cho 1α > . Khi ñó 1 · 1 0x x xα α+ ≤ + ∀ ≥ . Áp dụng nhận xét trên với , 2x n nα= = + ñược

21 ( 2) 2· 1n n n n+ + ≤ + + .

Từ ñó

2 41 ( 1) 1 ( 2) 1 2·( 1)· 1 2· 1 ( 1) 1n n n n n n n n n+ − + + ≤ + + − + ≤ + + − + .

Do ñó, bằng quy nạp, thu ñược 23 ( 2)

n

nn x−

≤ + (2) Từ (1),(2) và nguyên lý kẹp, suy ra lim 3n

nx

→∞= .

Bài tập tương tự

Bài toán 1.20. Cho , 2α α∈ >� , dãy số ( )na

+⊂ � thỏa mãn ñiều kiện

1 2 1... , 2.n na a a a nα

−= + + + ∀ ≥

Chứng minh rằng lim 0na

n= .

Bài tập 1.21. Cho dãy số dương ( )na thỏa mãn ñiều kiện 3 *

1 1 2 ... , .n na a a a n+ ≤ + + + ∀ ∈�

Chứng minh rằng với mọi 1

2α > ta luôn có lim 0na

= .

Bài tập 1.22. (VMO 2002A). Xét phương trình

2 2

1 1 1 1 1... ...

1 4 1 1 1 2x x k x n x+ + + + + =

− − − −,

trong ñó n là tham số nguyên dương. 1. Chứng minh rằng với mỗi số nguyên dương n , phương trình nêu trên có duy nhất

nghiệm lớn hơn 1; kí hiệu nghiệm ñó là nx .

2. Chứng minh rằng dãy số nx có giới hạn bằng 4 khi n → +∞ .

Bài tập 1.23. (Matxcơva 2000). Ký hiệu nx là nghiệm của phương trình

1 1 1... 0

1x x x n+ + + =

− −,

thuộc khoảng (0,1)

1. Chứng minh dãy ( )nx hội tụ;

2. Hãy tìm giới hạn ñó. Bài tập 1.24. (VMO 2007) Cho số thực 2a > và 10 10( ) ... 1n n

nf x a x x x+= + + + +

Page 111: Kỷ yếu hội thảo các trường chuyên dh db bắc bộ lần III

=========================================================== 113

HỘI CÁC TRƯỜNG THTP CHUYÊN KHU VỰC DUYÊN HẢI VÀ ðỒNG BẰNG BẮC BỘ

Hội thảo khoa học môn Toán học lần thứ III - 2010

1. Chứng minh rằng với mỗi số nguyên dương n , phương trình ( )nf x a= luôn có ñúng

một nghiệm dương duy nhất. 2. Gọi nghiệm ñó là nx , chứng minh rằng dãy ( )nx có giới hạn hữu hạn khi n dần ñến

vô cùng. 1.4. Khảo sát sự hội tụ của dãy số dạng 1 ( )n nx f x+ =

ðể khảo sát sự hội tụ của dãy số có dạng 1 ( )n nx f x+ = , ta thường xét hàm số

( )y f x= và sử dụng một số kết quả sau

ðịnh lí 1.10. Cho dãy số ( )nx ⊂ � xác ñịnh như sau: 1 1, ( ), 1, 2,...n nx a x f x n+= = = . Khi

ñó 1. Nếu ( )f x là hàm số ñồng biến thì dãy số ( )nx ñơn ñiệu.

2. Nếu ( )f x là hàm số nghịch biến thì dãy số ( )nx có chứa hai dãy con 2 2 1( ), ( )k kx x +

ñơn ñiệu ngược chiều. 3. Khi ( )f x là hàm số nghịch biến và dãy ( )nx bị chặn thì 2 2 1lim ,limk k

k kx a x b+

→∞ →∞∃ = =

và do ñó dãy ñã cho hội tụ khi và chỉ khi a b= . Ví dụ 1.25. (VMO 1998A). Cho số thực 1a ≥ . Xét dãy số ( ), 1, 2,...nx n = ñược xác

ñịnh bởi 2

1 1, 1 ln1 ln

nn

n

xx a x

x+

= = +

+ với 1, 2,3,...n =

Chứng minh rằng dãy số trên có giới hạn hữu hạn và tìm giới hạn ñó. Lời giải

Xét dãy số ( )nx với 1 ( 1)x a a= ≥ và 2

1 1 ln , 1,2,...1 ln

nn

n

xx n

x+

= + =

+

(i) Nếu 1a = thì 1( )nx n= ∀ suy ra lim 1nn

x→+∞

= .

(ii) Nếu 1a > . Ta chứng minh bằng quy nạp 1nx > với mọi *n ∈� . Giả sử với n sao

cho 1nx > . Ta nhận thấy 21 1 1 ln 0n n nx x x+ > ⇔ − − > . Dễ thấy hàm số 2( ) 1 lnf x x x= − −

ñồng biến trên [1; )+∞ . Mặt khác 1nx > suy ra 1 1nx + > . Vậy 1 1nx n> ∀ ≥ .

Tiếp theo ta chứng minh với 1 1nx n> ∀ ≥ thì 1 1n nx x n+> ∀ ≥ . Xét hàm số 2

( ) 1 ln1 ln

xg x x

x

= − −

+ trên [1; )+∞ . Bằng cách khảo sát hàm số này ta chỉ ra ñược ( )g x

ñồng biến trên [1; )+∞ mà (1) 0g = , suy ra ( ) 0 1g x x> ∀ > và ( ) 0 1g x x= ⇔ = . Do ñó nếu

1 1nx n> ∀ ≥ thì 1 1n nx x n+> ∀ ≥ . Do vậy dãy ( )nx là dãy số giảm và bị chặn dưới bởi 1, nên

tồn tại limn

b→+∞

= . Dễ thấy 1b ≥ và từ hệ thức truy hồi chuyển qua giới hạn ta ñược

2 2

1 ln 1 ln 01 ln 1 ln

b bb b

b b

= + ⇔ − − =

+ + .

Theo kết quả khảo sát của hàm ( )g x ở trên thì ( ) 0 1g b b= ⇔ = . Vậy lim 1nn

x→+∞

= .

Ví dụ 1.26. Cho dãy số ( )nx thỏa mãn ñiều kiện

1 1 22,9; 3 , 1,2,3,

1n

n

n

xx x n

x+= = + = …

Page 112: Kỷ yếu hội thảo các trường chuyên dh db bắc bộ lần III

=========================================================== 114

HỘI CÁC TRƯỜNG THTP CHUYÊN KHU VỰC DUYÊN HẢI VÀ ðỒNG BẰNG BẮC BỘ

Hội thảo khoa học môn Toán học lần thứ III - 2010

Chứng minh rằng dãy số trên có giới hạn và tìm giới hạn ñó. Lời giải

Xét hàm số 2

( ) 31

xf x

x= +

− với (1, )x ∈ +∞ . Dễ thấy ( )f x là hàm số nghịch

biến trên (1, )+∞ .

(i) Ta chứng minh dãy ( )nx bị chặn. Ta sẽ chứng minh bằng quy nạp

*33 3

2nu n< < + ∀ ∈� (1). Thật vậy

Với 1n = thì bất ñẳng thức trên luôn ñúng. Giả sử bất ñẳng thức trên ñúng ñến n , tức là

33 3

2nu< < + . Ta có 1 ( )n nu f u+ = và f là nghịch biến trên (1, )+∞ nên

1

3( 3) 3

2nu f+ < = + . Mặt khác do 3 nu< nên từ hệ thức 1 ( )n nu f u+ = ta có 13 nu +< .

Vậy (1) ñược chứng minh. (ii) Từ ñó suy ra 2 2 1lim , limn n

n na x b x +

→+∞ →+∞∃ = ∃ = , trong ñó ,a b là nghiệm của hệ phương

trình

( )

( )

a f b

b f a

=

=.

(iii) Xét hàm số ( ) ( ( ))g x f f x x= − , với 3

3 32

x< < + , có

( ) ( ). ( ( )) 1g x f x f f x′ ′ ′= − . Do 3

3 ( ) 32

f x< < + và ( ) 0f x′ < với mọi

33 3

2x< < + nên ( ) 0g x′ < với mọi

33 3

2x< < + , cùng với

3( 3). ( ) 0

2g g <

suy ra phương trình ( ) 0g x = có nghiệm duy nhất. Do ñó dãy ( )nx hội tụ.

Ví dụ 1.27. (VMO 2008) Cho dãy số ( )nx xác ñịnh như sau

1 2

2

0, 2

12 , 1,2,...

2nx

n

x x

x n−

+

= =

= + =

Chứng minh rằng dãy ( )nx hội tụ và tìm lim nn

x→+∞

.

Lời giải 1. Bằng quy nạp, dễ dàng chứng minh ñược 1 3

22 2n

x n< < ∀ > . Xét hàm số

1 1 3( ) 2 , ;

2 2 2x

f x x−

= + ∈

. Ta có 1 1 3

( ) 2 ·ln 0 ;2 2 2

xf x x

− ′ = < ∀ ∈

và với mọi

1 3;

2 2x

thì 3

1 12 ; (0;1)

4 2x− ∈ ⊂

. Do ñó ln 2

| ( ) | 12

f x u′ < = < .

Mặt khác, theo ñịnh lý Lagrange thì với mọi 1 3

2 2x y< ≤ < ñều tồn tại ( ; )t x y∈ sao cho

2 2 ( )( )x yf t x y

− − ′− = − . Vậy

Page 113: Kỷ yếu hội thảo các trường chuyên dh db bắc bộ lần III

=========================================================== 115

HỘI CÁC TRƯỜNG THTP CHUYÊN KHU VỰC DUYÊN HẢI VÀ ðỒNG BẰNG BẮC BỘ

Hội thảo khoa học môn Toán học lần thứ III - 2010

2 3

4 5

1 2 3

24 5

| | 2 2 | ·| |

·| 2 2 | ·

|

| |

n n

n n

x x

n n n n

x x

n n

x x u x x

u u x x

− −

− −

− −

− − −

− −

− −

− = − < −

= − < − LL

Từ ñó

2 2 1 2 1| | | | 0 ( )n

n nx x u x x n−− < − → → +∞ .

Từ ñó, theo ñịnh lý Cauchy, dãy $(x_n)$ hội tụ về α là nghiệm của phương trình 1

22

αα −= +

Giải phương trình này, thu ñược 1α = . Vậy, lim 1n

nx

→+∞= .

Lời giải 2. Bằng quy nạp, dễ dàng chứng minh ñược 1 3

22 2nx n< < ∀ > .

Xét hàm số 1 1 3

( ) 2 , ;2 2 2

xf x x

− = + ∈

. Ta có

1 1 3( ) 2 ·ln 0 ;

2 2 2x

f x x−

′ = < ∀ ∈

. Do

ñó hàm 1 3

( ), ;2 2

y f x x

= ∈

là hàm giảm. Vậy, mỗi dãy ( ) ( )2 2 1,k kx x + chứa hai dãy con

ñơn ñiệu ngược chiều. Từ ñó, do 1 3

22 2n

x n< < ∀ > suy ra bốn dãy con

4 4 1 4 2 4 3( ), ( ), ( ), ( )k k k kx x x x+ + + hội tụ theo thứ tự về , , ,α β γ δ .

Xét hệ phương trình

( )

( )

( )

( )

f

f

f

f

α γ

β δ

γ α

δ β

=

=

= =

Giải hệ thu ñược 1α β γ δ= = = = . Vậy lim 1nn

x→+∞

= .

Lời giải 3. Bằng quy nạp, dễ dàng chứng minh ñược 1 3

22 2n

x n< < ∀ > .

Xét hàm số 1 1 3

( ) 2 , ;2 2 2

xf x x

− = + ∈

. Ta có

1 1 3( ) 2 ·ln 0 ;

2 2 2x

f x x−

′ = < ∀ ∈

và với

mọi 1 3

;2 2

x

thì 3

1 12 ; (0;1)

4 2x− ∈ ⊂

. Do ñó ln 2

| ( ) | 12

f x u′ < = < .

Mặt khác, theo ñịnh lý Lagrange thì với mọi 1 3

2 2x y< ≤ < ñều tồn tại ( ; )t x y∈ sao cho

2 2 ( )( )x yf t x y

− − ′− = − . Vậy, với mọi 1 3

, ;2 2

x y

tồn tại ln 2

(0;1)2

u = ∈ sao cho

| ( ) ( ) | . | |f x f y u x y− = − . Suy ra hàm f là hàm co. Bởi vậy, hai dãy con 2 2 1( ), ( )k kx x + (ñều

cho bởi hệ thức truy hồi 2 ( )n nx f x+ = hội tụ. Bằng việc giải phương trình giới hạn, thu ñược

lim 1nn

x→+∞

= .

Page 114: Kỷ yếu hội thảo các trường chuyên dh db bắc bộ lần III

=========================================================== 116

HỘI CÁC TRƯỜNG THTP CHUYÊN KHU VỰC DUYÊN HẢI VÀ ðỒNG BẰNG BẮC BỘ

Hội thảo khoa học môn Toán học lần thứ III - 2010

Bài tập tương tự

Bài tập 1.28. Cho dãy số ( )nx xác ñịnh như sau 0 1 21, , 0

1n

n

n

xx x n

x+= = ≥

+. Tìm lim n

nx

→∞.

Bài tập 1.29. Cho trước 0a > . Xét dãy số ( )nx xác ñịnh như sau:

0

2

1

0

1= , 0,1,2,...

2n n

n

x

ax x n

x+

>

+ =

Khảo sát sự hội tụ của dãy.

Bài tập 1.30. Khảo sát sự hội tụ của dãy 0 1

1( ) : 1, , 0

2n n

n

x x x nx

+= = ≥+

.

Bài tập 1.31. Khảo sát sự hội tụ của dãy 0 1 2

2( ) : 0, , 0

1n n

n

x x x nx

+≥ = ≥+

.

Bài tập 1.32. Khảo sát sự hội tụ của dãy 0 1 2

6( ) : 0, , 0

2n n

n

x x x nx

+≥ = ≥+

.

Bài tập 1.33. Khảo sát sự hội tụ của dãy 0 1

2( ) : 1, 1 , 0n n

n

x x x nx

+= = − ≥ .

Bài tập 1.34. Khảo sát sự hội tụ của dãy 2

0 1

3( ) : 0, , 0

2( 1)n

n n

n

xx x x n

x+

+> = ≥

+.

Bài tập 1.35. Khảo sát sự hội tụ của dãy 30 1( ) : , 7 6, 0n n nx x x x n+∈ = − ≥� .

Bài tập 1.36. Khảo sát sự hội tụ của dãy 0 1

1( ) : 0, 1, 0n n n

n

x x x x nx

+> = + − ≥ .

Bài tập 1.37. Khảo sát sự hội tụ của dãy 20 1( ) : , 2 , 0n n n nx x x x x n+∈ = + ≥� .

Bài tập 1.38. Khảo sát sự hội tụ của dãy 0 1( ) : ( 1;0), 1 ( 1) 1 , 0n

n n nx x x x n+∈ − = + − + ≥ .

Bài tập 1.39. Khảo sát sự hội tụ của dãy 0 1 1 0( ) : 0, , 0n n n nx x x x x x n+ −> = + + ≥L .

Bài tập 1.40. Khảo sát sự hội tụ của dãy 1 1( ) : 2, 2 1n n nx x x x n+= = ∀ ≥ .

Bài tập 1.41. Khảo sát sự hội tụ của dãy 230 1 2 1( ) : 1, , , 0n n n nx x x a x x x n+ += = = ≥ .

Bài tập 1.42. Cho dãy số ( )nx xác ñịnh như sau

( )1

2 21 1 2 1.

n n n

x

x x a x a n+

= + − + ∀ ≥

Tìm tất cả các giá trị thực của tham số a sao cho dãy ñã cho hội tụ. Khi ñó, tìm lim nn

x→∞

.

Bài tập 1.43. (VMO 2005B). Cho dãy số thực ( ), 1,2,3...nx n = ñược xác ñịnh bởi

1x a= và 3 21 3 7 5n n n nx x x x+ = − + với mọi 1, 2,3,...n = , trong ñó a là một số thực thuộc

ñoạn 4

0,3

.

Chứng minh rằng dãy số ( )nx có giới hạn hữu hạn và tìm giới hạn ñó.

Bài tập 1.44. (VMO 2005A). Cho dãy số thực ( ), 1,2,3...nx n = ñược xác ñịnh bởi

1x a= và 3 21 3 7 5n n n nx x x x+ = − + với mọi 1, 2,3,...n = , trong ñó a là một số thực.

Page 115: Kỷ yếu hội thảo các trường chuyên dh db bắc bộ lần III

=========================================================== 117

HỘI CÁC TRƯỜNG THTP CHUYÊN KHU VỰC DUYÊN HẢI VÀ ðỒNG BẰNG BẮC BỘ

Hội thảo khoa học môn Toán học lần thứ III - 2010

Hãy tìm tất cả các giá trị của a ñể dãy số ( )nx có giới hạn hữu hạn. Hãy tìm giới hạn ñó

trong các trường hợp ñó.\hbt Bài tập 1.45. (VMO 2001A). Với mỗi cặp số thực ( , )a b , xét dãy số ( ),nx n ∈� , ñược

xác ñịnh bởi

0x a= và 1 .sinn n nx x b x+ = + với mọi n ∈� .

(1) Cho 1b = . Chứng minh rằng với mọi số thực a , dãy ( )nx có giới hạn hữu hạn khi

n → +∞ . Hãy tính giới hạn ñó theo a . (2) Chứng minh rằng với mỗi số thực 2b > cho trước, tồn tại số thực a sao cho dãy

( )nx tương ứng không có giới hạn hữu hạn khi n → +∞ .

Bài tập 1.46. (VMO 2000A). Cho c là số thực dương. Dãy số ( ), 0,1,2,...nx n = ñược

xây dựng theo cách sau:

1 , 0,1,2,...n n

x c c x n+ = − + = nếu các biểu thức trong căn là không âm.

Tìm tất cả các giá trị của c ñể với mọi giá trị ban ñầu ( )0 0;x c∈ dãy ( )nx ñược xác

ñịnh với mọi giá trị n và tồn tại giới hạn hữu hạn lim nx khi n → +∞ .

Bài tập 1.47. (VMO 1998B). Cho số thực a . Xét dãy số ( ), 1, 2,3,...nx n = ñược xác ñịnh bởi

( )2

1 1 2

3,

3 1n n

n

n

x xx a x

x+

+= =

+ với n=1, 2, 3, ...

Chứng minh rằng dãy số trên có giới hạn hữu hạn và tìm giới hạn ñó. Bài tập 1.48. (VMO 1994B). Cho số thực a . Xét dãy số ( ), 0,1,2,...nx n = ñược xác

ñịnh bởi 3

0 1 1, 6 6sinn n n

x a x x x− −= = − với mọi n=1, 2, 3,...

Chứng minh rằng dãy số trên có giới hạn hữu hạn khi n dần tới dương vô cực và tìm giới hạn ñó.

Bài tập 1.49. (VMO 1994A). Cho số thực a . Xét dãy số ( ), 0,1,2,...nx n = ñược xác

ñịnh bởi

0 1 12

4, arccos .arcsin

2n n nx a x x xπ

π− −

= = +

với mọi n=1, 2, 3,...

Chứng minh rằng dãy số trên có giới hạn hữu hạn khi n dần tới dương vô cực và tìm giới hạn ñó.

1.5. ðịnh lý trung bình Cesaro và dãy số dạng 1

a

n n nx x x+ = ±

ðây là trường hợp ñặc biệt của dãy số dạng 1 ( )n nx f x+ = . Tuy nhiên, chúng ta không

ñặt vấn ñề khảo sát sự hội tụ của những dãy dạng này, bởi vì giới hạn của chúng hoặc là 0

hoặc là ∞ ; mà ở ñây chúng ta quan tâm tới tất cả các số β sao cho dãy nx

hội tụ. Với

những dãy số dạng này, ñịnh lý trung bình Cesaro tỏ ra rất hữu hiệu.

ðịnh lí 1.11. Nếu dãy số ( )nx có giới hạn hữu hạn là a thì dãy số các trung bình

1 2 ...n

x x x

n

+ + +

cũng có giới hạn là a .

Chứng minh.

Page 116: Kỷ yếu hội thảo các trường chuyên dh db bắc bộ lần III

=========================================================== 118

HỘI CÁC TRƯỜNG THTP CHUYÊN KHU VỰC DUYÊN HẢI VÀ ðỒNG BẰNG BẮC BỘ

Hội thảo khoa học môn Toán học lần thứ III - 2010

Không mất tính tổng quát ta có thể giả sử 0a = . Với mọi 0ε > tồn tại *N ∈� sao

cho với mọi n N≥ thì 2n

< và 1 2 ...

2N

u u u

n

ε+ + +< . Từ ñó ta có

( )1 2 1 2 1... ... ...

2 2n N N n

n Nu u u u u u u un N

n n n n

ε εε+

−+ + + + + + + +≤ + < + < ∀ ≥ .

Ví dụ 1.50. Nếu ( )1lim n nn

x x a+→∞

− = thì lim .n

n

xa

n→∞=

Lời giải. ðặt 1n n nu x x −= − . Khi ñó dễ thấy dãy ( )nu thỏa mãn ñiều kiện của ðịnh lý

Cesaro nên ta có 1 ...lim n

n

u ua

n→∞

+ += hay lim n

n

xa

n→∞= .

Ví dụ 1.51. Chứng minh rằng nếu dãy số dương ( )na hội tụ về a dương thì

1 2lim ...nn

na a a a

→∞= .

Lời giải. Ta có lim ln lnn

na a

→∞= . Áp dụng ðịnh lý Cesaro, ta có:

1ln ... lnlim lnn

n

a aa

n→∞

+ += hay 1 2lim ...n

nn

a a a a→∞

= .

Ví dụ 1.52. Cho dãy số dương ( )na . Chứng minh rằng 1lim 0n

nn

aa

a

+

→∞= > thì lim n

nn

a a→∞

= .

Lời giải. ðặt 1 , 2nn

n

ab n

a

+= ≥ . Dễ thấy dãy ( )nb thỏa mãn ví dụ 1.50 nên

1 2lim ...nn

nb b b a

→∞= hay lim n

nn

a a→∞

= .

Bài tập tương tự

Bài tập 1.53. Cho dãy ( )nx xác ñịnh bởi 21 11/ 2, n n nx x x x+= = − . Chứng minh rằng

lim 1.nn

nx→∞

=

Bài tập 1.54. Cho dãy ( )nx xác ñịnh bởi 1 11, sinn nx x x+= = . Chứng minh rằng

lim 1.nn

nx→∞

=

Bài tập 1.55. (TST VN 1993). Dãy số { }nx xác ñịnh bởi 1 1

11, n n

n

x x xx

+= = + . Hãy

tìm tất cả các số α ñể dãy số ( )na

n

α

có giới hạn hữu hạn khác 0.

Bài tập 1.56. Cho dãy số xác ñịnh bởi ( )1 10, 1 sin 1 , 1n na a a n+= = − − ≥ . Tính

1

1lim

n

kn

k

an→∞

=

∑ .

Page 117: Kỷ yếu hội thảo các trường chuyên dh db bắc bộ lần III

=========================================================== 119

HỘI CÁC TRƯỜNG THTP CHUYÊN KHU VỰC DUYÊN HẢI VÀ ðỒNG BẰNG BẮC BỘ

Hội thảo khoa học môn Toán học lần thứ III - 2010

Bài tập 1.57. Xét dãy số ( )nx xác ñịnh bởi 1 13

11, 1n n

n

x x x nx

+= = + ∀ ≥ . Chứng minh

rằng tồn tại ,a b sao cho lim 1n

bn

x

an→∞= .

2 Bài toán dãy số qua các kì thi IMO 2.1 IMO 2009 Bài 2.1.1 (IMO 2009) . Giả sử 1 2 3, , ,...s s s là một dãy tăng ngặt các số nguyên dương

sao cho các dãy con 1 2 3, , ,...s s ss s s và

1 2 31 1 1, , ,...s s ss s s+ + + ñều là cấp số cộng. Chứng minh rằng

1 2 3, , ,...s s s cũng là một cấp số cộng.

Bài 2.1.2 (Mở rộng IMO 2009). Cho k là một số nguyên dương cho trước. Giả sử

1 2 3, , ,...s s s là một dãy tăng nghặt các số nguyên dương sao cho các dãy con 1 2 3, , ,...s s ss s s và

1 2 3, , ,...s k s k s ks s s+ + + ñều là cấp số cộng. Chứng minh rằng 1 2 3, , ,...s s s cũng là một cấp số cộng.

Chứng minh.

Gọi D và E lần lượt là công sai của các cấp số cộng 1 2 3, , ,...s s ss s s

và1 2 3

, , ,...s k s k s ks s s+ + + . ðặt 1s

A s D= − và 1s kB s E+= − . Theo công thức tính số hạng tổng

quát của một cấp số cộng và với số nguyên dương n ta có

1 1( 1) , ( 1) .

n ns s s k s ks s n D A nD s s n E B nE+ += + − = + = + − = +

Từ dãy 1 2 3, , ,...s s s là một dãy tăng ngặt, nên với mọi số nguyên dương n và với chú ý

n n ks k s ++ ≤ ta có

1 ,n n n ks s k ss k s s

+++ − < ≤

từ ñó ta thu ñược 1 ( 1) ,A nD k B nE A n D+ + − < + ≤ + +

ñiều này tương ñương với 0 1 ( ) ,k B A n E D kD< − + − + − ≤

nếu D khác E thì cho n → ∞ ta thấy mâu thuẫn với bất ñẳng thức trên nên D E= và do ñó

0 1 .k B A kD≤ − + − ≤ (1) ðặt { }1min : 1, 2,...n nm s s n+= − = . Khi ñó

1 1 1 1( ) ( )s k s s k sB A s E s D s s km+ +− = − − − = − ≥ (2)

1 11 1( ) ( ) ( ).

s k ss s B D A DkD A s k D A s D s s s s m B A

+ + += + + − + = − = − ≥ − (3)

Ta xét hai trường hợp (a) B A kD− = . Khi ñó, với mỗi số nguyên dương , ( )

n n ks k sn s B nD A n k D s++ = + = + + = , từ ñây kết hợp

với dãy 1 2 3, , ,...s s s là một dãy tăng ngặt ta có n k ns s k+ = + . Mặt khác do

1 ...n n n k ns s s s k+ +< < < = + nên 1 1n ns s+ = + và do ñó 1 2 3, , ,...s s s là một cấp số cộng với công

sai bằng 1.

Page 118: Kỷ yếu hội thảo các trường chuyên dh db bắc bộ lần III

=========================================================== 120

HỘI CÁC TRƯỜNG THTP CHUYÊN KHU VỰC DUYÊN HẢI VÀ ðỒNG BẰNG BẮC BỘ

Hội thảo khoa học môn Toán học lần thứ III - 2010

(b) B A kD− < . Chọn số nguyên dương N sao cho 1N Ns s m+ − = . Khi ñó

( ) (( ( 1) ) ( ))m A B D k m A N D B ND k− + − = + + − + +

1( 1) s s kN N

A N D B ND k s s ks s s s

+++ + + + +≤ − = −

1 1( ) ( ( ) ) ( )N N N NA s D B s k D s s D A B kD+ += + − + + = − + − −

,mD A B kD= + − − do vậy

( ) ( ( )) 0.B A km kD m B A− − + − − ≤ (4) Từ các bất ñẳng thức (2), (3) và (4) ta thu ñược các ñẳng thức sau:

B A km− = và ( )kD m B A= − .

Giả sử tồn tại số nguyên dương n sao cho 1n ns s m+ > + . Khi ñó

11( 1) ( )n nn n s sm m m s s s s

+++ ≤ − ≤ −

2( )( ( 1) ) ( )

m B AA n D A nD D m

k

−= + + − + = = = ,

vô lý. Vì vậy ñiều giả sử là sai nên 1n ns s m+ = + với mọi n ∈� hay dãy 1 2, ,...s s là một cấp số

cộng có công sai bằng m . Nhận xét

Bây giờ ta thay cấp số cộng bởi cấp số nhân khi ñó bài toán trên còn ñúng không?

Bài 2.1.3. Giả sử 1 2 3, , ,...s s s là một dãy tăng nghặt các số nguyên dương sao cho các

dãy con 1 2 3, , ,...s s ss s s và

1 2 31 1 1, , ,...s s ss s s+ + + ñều là cấp số nhân. Chứng minh rằng 1 2 3, , ,...s s s

cũng là một cấp số nhân. Bài 2.1.4. (Mở rộng của bài toán 2.1.3) Cho k là một số nguyên dương. Giả sử

1 2 3, , ,...s s s là một dãy tăng nghặt các số nguyên dương sao cho các dãy con 1 2 3, , ,...s s ss s s và

1 2 3, , ,...s k s k s ks s s+ + + ñều là cấp số nhân. Chứng minh rằng 1 2 3, , ,...s s s cũng là một cấp số nhân.

IMO 2010 Bài 2.2.1

Cho 1 2 3, , ,...a a a là một dãy số thực dương. Giả sử với số nguyên dương s cho trước,

ta có

{ }max :1 1n k n ka a a k n−= + ≤ ≤ − ,

với mọi n s> . Chứng minh rằng tồn tại các số nguyên dương l và N , với l s≤ và thỏa mãn n l n la a a −= + với mọi n N≥ .

Chứng minh.

Từ ñiều kiện bài toán và với mỗi na ( n s> ) ta có ñẳng thức sau

1 2n j ja a a= + với

1 2 1 2, ,j j n j j n< + = nếu 1j s> thì ta có thể viết ñược 1j

a giống như na . Cuối cùng, ta có

thể viết ñược ñẳng thức dưới ñây

1 2... ,

kn i i ia a a a= + + + (1)

Page 119: Kỷ yếu hội thảo các trường chuyên dh db bắc bộ lần III

=========================================================== 121

HỘI CÁC TRƯỜNG THTP CHUYÊN KHU VỰC DUYÊN HẢI VÀ ðỒNG BẰNG BẮC BỘ

Hội thảo khoa học môn Toán học lần thứ III - 2010

1 21 , ... , 1, 2,...,j ki s i i i n j k≤ ≤ + + + = = . (2)

Cố ñịnh chỉ số 1 l s≤ ≤ sao cho

1min .l i

i s

a am

l i≤ ≤= =

Ta xác ñịnh dãy { }nb với n nb a mn= − , 0lb = .

Ta sẽ chứng minh với mọi n thì 0nb ≥ , và dãy { }nb thỏa mãn các tính chất giống như

dãy { }nb .

Thật vậy nếu n s≤ thì ta có ngay 0nb ≥ theo cách xác ñịnh của m . Bây giờ ta xét nếu

n s> và sử dụng phương pháp quy nạp cùng với ñánh giá sau

1 1 1 1max ( ) max ( )n k n k k n k

k n k nb a a nm b b nm nm− −

≤ ≤ − ≤ ≤ −= + − = + + −

1 1max ( ),k n k

k nb b −

≤ ≤ −= +

ta thu ñược 0nb ≥ .

Nếu 0kb = với mọi 1 k s≤ ≤ , khi ñó 0nb = với mọi n , vì vậy na mn= , và trường hợp

này là tầm thường. Nếu tồn tại 1 1k n≤ ≤ − sao cho nb khác 0, ta xác ñịnh

{ }1max , min :1 , 0 .

i i ii s

M b b i s bε≤ ≤

= = ≤ ≤ >

Khi ñó với n s> ta ñạt ñược

1 1max ( ) ,n k n k l n l n l

k nb b b b b b− − −

≤ ≤ −= + ≥ + =

vì vậy

2 ... 0.n n n l n lM a b b b− −≥ ≥ ≥ ≥ ≥ ≥

Ta có dãy ( )nb cũng có tính chất (1), (2) giống như dãy ( )na , ta có với mỗi nb chứa

trong tập

{ } [ ]1 2 1... :1 ,..., 0,

ki i i kT b b b i i s M= + + + ≤ ≤ ∩ .

Ta chứng minh tập này chỉ có hữu hạn phần tử. Thật vậy, với mọi x T∈ , biểu diễn ñược

1 2 1... (1 ,..., )ki i i kx b b b i i s= + + + ≤ ≤ . Khi ñó chỉ có tối ña

M

ε số

jib khác 0 (vì nếu ngược lại

thì .M

x Mεε

> = ñiều này vô lý). Vì vậy x chỉ có thể biểu thành tổng của k số ji

b với

Mk

ε≤ , và do ñó tập này chỉ có hữu hạn.

Từ ñó ta có ngay dãy nb là một dãy tuần hoàn với chu kì l từ một chỉ số N trở ñi, có

nghĩa là

n n l n l lb b b b− −= = + với n N l> + ,

và do ñó ( ( ) ) ( )n n n l l n l la b nm b n l m b lm a a− −= + = + − + + = + với mọi n N l> + .

Từ bài toán này ta có thể xây dựng ñược một số dạng bài tập sau và ñiều kiện dãy số dương là không cần thiết.

Bài 2.2.2

Cho 1 2 3, , ,...a a a là một dãy số thực. Giả sử với số nguyên dương s cho trước, ta có

{ }min :1 1n k n ka a a k n−= + ≤ ≤ −

Page 120: Kỷ yếu hội thảo các trường chuyên dh db bắc bộ lần III

=========================================================== 122

HỘI CÁC TRƯỜNG THTP CHUYÊN KHU VỰC DUYÊN HẢI VÀ ðỒNG BẰNG BẮC BỘ

Hội thảo khoa học môn Toán học lần thứ III - 2010

với mọi n s> . Chứng minh rằng tồn tại các số nguyên dương l và N , với l s≤ và thỏa mãn n l n la a a −= + với mọi n N≥ .

Bài 2.2.3

Cho 1 2 3, , ,...a a a là một dãy số thực dương. Giả sử với mỗi số nguyên dương s , ta có

{ }max . :1 1n k n ka a a k n−= ≤ ≤ −

với mọi n s> . Chứng minh rằng tồn tại các số nguyên dương l và N , với l s≤ và thỏa mãn .n l n la a a −= với mọi n N≥ .

Chứng minh.

ðặt lnn nb a= thì dãy 1 2 3, , ,...b b b là một dãy số thực và với cách chứng minh tương

tự như bài 2.2.1 ta sẽ thu ñược kết quả bài toán trên.

Page 121: Kỷ yếu hội thảo các trường chuyên dh db bắc bộ lần III

=========================================================== 123

HỘI CÁC TRƯỜNG THTP CHUYÊN KHU VỰC DUYÊN HẢI VÀ ðỒNG BẰNG BẮC BỘ

Hội thảo khoa học môn Toán học lần thứ III - 2010

SỬ DỤNG CÔNG CỤ SỐ PHỨC ðỂ GIẢI CÁC BÀI TOÁN HÌNH HỌC PHẲNG

Trường THPT chuyên Hạ Long Ta biết rằng mỗi số phức ñược biểu diễn bởi một ñiểm trong mặt phẳng phức. Do ñó

cũng như phương pháp toạ ñộ, khi ñồng nhất mỗi ñiểm trong mặt phẳng bởi một số phức thì bài toán trong hình học phẳng thành bài toán với số phức mà ta biết rằng các công thức về khoảng cách và góc có thể ñưa về các công thức ñơn giản ñối với số phức. Do vậy ta có thể sử dụng số phức ñể giải các bài toán hình học từ ñơn giản ñến phức tạp. Trong bài này, ta quy ước mỗi ñiểm A ñược biểu diễn nó trong mặt phẳng phức, do ñó ta có các khái niệm tương ứng là ñường thẳng ab, tam giác abc…ðể sử dụng ñược công cụ này ta cần nắm ñược các công thức và ñịnh lý sau:

1. Các công thức và ñịnh lý: ðịnh lý 1.1

ðường thẳng ab//cd khi và chỉ khi a b c d

a b c b

− −=

− −.

Các ñiểm a, b, c thẳng hàng khi và chỉ khi a b a c

a b a c

− −=

− −

ðường thẳng ab vuông góc với cd khi và chỉ khi a b c d

a b c b

− −= −

− −

Gọi ϕ là góc acb theo chiều dương từ a ñến b thì ic b c ae

c b c a

ϕ− −=

− −.

ðịnh lý 1.2 Trên ñường tròn ñơn vị, ta có các tính chất sau:

Hai ñiểm a, b thuộc ñường tròn ñơn vị thì a b

aba b

−= −

ðiểm c nằm trên dây cung ab thì a b c

cab

+ −= .

Giao của hai tiếp tuyến tại hai ñiểm a, b là ñiểm 2ab

a b+.

Chân ñường cao hạ từ một ñiểm c bất kì xuống dây ab của ñường tròn là ñiểm 1

( )2

a b c abc+ + − .

Giao ñiểm của hai dây cung ab và cd là ñiểm ( ) ( )ab c d cd a b

ab cd

+ − +

−.

ðịnh lý 1.3

4 ñiểm a, b, c, d cùng thuộc một ñường tròn khi và chỉ khi :a c a d

b c b d

− −∈

− −� .

ðịnh lý 1.4

Tam giác abc và tam giác pqr ñồng dạng và cùng hướng khi và chỉ khi a c p r

b c q r

− −=

− −.

ðịnh lý 1.5 Diện tích có hướng của tam giác abc là

( )4

iS ab bc ca ab bc ca= + + − − − .

Page 122: Kỷ yếu hội thảo các trường chuyên dh db bắc bộ lần III

=========================================================== 124

HỘI CÁC TRƯỜNG THTP CHUYÊN KHU VỰC DUYÊN HẢI VÀ ðỒNG BẰNG BẮC BỘ

Hội thảo khoa học môn Toán học lần thứ III - 2010

ðịnh lý 1.6 ðiểm c chia ñoạn thẳng ab theo tỉ số 1λ ≠ khi và chỉ khi 1

a bc

λ

λ

−=

−.

ðiểm G là trọng tâm tam giác abc khi và chỉ khi 3

a b cg

+ += .

Với H là trực tâm và O là tâm ngoại tiếp thì h+2o=a+b+c. ðịnh lý 1.7 Giả sử ñường tròn ñơn vị nội tiếp tam giác abc và tiếp xúc với các cạnh bc,

ca, ab của tam giác abc tại p, q, r thì

a. 2 2 2

, ,qr rp pq

a b cq r r p p q

= = =+ + +

b. Với h là trực tâm tam giác abc ta có 2 2 2 2 2 22( ( ))

( )( )( )

p q q r r p pqr p q rh

p q q r r p

+ + + + +=

+ + +

c. Với tâm ñường tròn bàng tiếp o, tương tự ta có: 2 ( )

( )( )( )

pqr p q ro

p q q r r p

+ +=

+ + +.

ðịnh lý 1.8 Cho tam giác abc nội tiếp ñường tròn ñơn vị, khi ñó tồn tại các số u, v, w

sao cho 2 2 2, , wa u b v c= = = và –uv,-vw,-wu là trung ñiểm của các cung ab, bc, ca không chứa các ñỉnh ñối diện. Khi ñó tâm ñường tròn nội tiếp i có i=-(uv+vw+wu).

ðịnh lý 1.9 Nếu tam giác có một ñỉnh trùng với gốc toạ ñộ và các ñỉnh còn lại là x, y thì

Trực tâm là ñiểm ( )( )xy x y x y

hx y xy

+ −=

Tâm ñường tròn ngoại tiếp là ñiểm ( )xy x y

oxy x y

−=

−.

Ta bắt ñầu với một số ví dụ như sau: 1. Cho tam giác ABC tâm ñường tròn ngoại tiếp O, trực tâm H. Q là ñiểm ñối xứng với

H qua O. Kí hiệu A’;B’;C’ là trọng tâm các tam giác BCQ, ACQ, ABQ. Chứng minh rằng:

4' ' '

3AA BB CC R= = = .

Giải: Giả sử bán kính ñường tròn ngoại tiếp bằng 1 và tâm ñường tròn ngoại tiếp trùng với

gốc toạ ñộ. Giả sử các ñiểm A, B, C biểu diễn bởi các số a, b, c (|a|=|b|=|c|=1) khi ñó trực tâm h=a+b+c và do O là trung ñiểm của HQ nên q=-a-b-c. Do A’ là trọng tâm tam giác BCQ

nên A’=(b+c+q)/3=-a/3. Ta có 4 4

AA'3 3 3

aa R= + = = . Làm tương tự ta suy ra ñpcm.

2. Cho tứ giác ABCD nội tiếp và A’, B’, C’, D’ lần lượt là trực tâm các tam giác BCD, CDA, DAB, ABC. Chứng minh rằng hai tứ giác ABCD và A’B’C’D’ ñồng dạng.

Giải: Xét 4 số phức a, b, c, d trên ñường tròn ñơn vị. Khi ñó a’=b+c+d; b’=c+d+a…. Khi ñó dễ thấy a’-b’=a-b, b’-c’=b-c; c’-a’=c-a nên suy ra tam giác ABC ñồng dạng với

tam giác A’B’C’. Làm tương tự với các tam giác còn lại và dễ suy ra tứ giác ABCD và A’B’C’D’ ñồng dạng.

3. Cho tam giác ABC, về phía ngoài tam giác dựng các hình vuông BCDE, CAFG, ABHI. Dựng các hình bình hành DCGQ, EBHP. Chứng minh rằng tam giác APQ vuông cân.

Page 123: Kỷ yếu hội thảo các trường chuyên dh db bắc bộ lần III

=========================================================== 125

HỘI CÁC TRƯỜNG THTP CHUYÊN KHU VỰC DUYÊN HẢI VÀ ðỒNG BẰNG BẮC BỘ

Hội thảo khoa học môn Toán học lần thứ III - 2010

Ta sử dụng một kết quả dễ chứng minh như sau: Nếu b là ảnh của a qua phép quay

tâm c góc quay ϕ thì ( )ib c e a c

ϕ− = − .

Giải (phụ thuộc vào góc quay của hình vẽ)

Vì H là ảnh của A qua phép quay tâm B góc quay 2

π nên

2 ( ) ( )i

h b e a b i a bπ

− = − = − do ñó (1 )h i b ia= − + . Tương tự (1 )e i b ic= + − . Do

EBHP là hình bình hành nên b+p=h+e nên tính ñược p=b+ia-ic. Tương tự tính ñược q=-ia+ib+c Khi ñó p-a=b+(i-1)a-ic và q-a=(-i-1)a+ib+c

Dễ thấy p-a=-i(q-a) nên p là ảnh của q qua phép quay tâm A góc quay 2

π− .

Bài tập: 4. Cho tứ giác ABCD. Về phía ngoài tứ giác dựng các tam ñều BCM, CDN, DAP. I, E,

F là trung ñiểm của AB, MN, NP. Chứng minh rằng tam giác IEF cân. 5. Cho tứ giác lồi ABCD với AC=BD. Dựng phía ngoài tứ giác các tam giác ñều cạnh

AB, BC, CD, DA và gọi G1;G2;G3;G4 là trọng tâm các tam giác ñó. Chứng minh rằng G1G3 vuông góc với G2G4.

Với ña giác ñều n cạnh 0 1 1...

nA A A

− ta có thể giả sử chúng nội tiếp trong ñường tròn

ñơn vị. Khi ñó ta còn có thể chọn ñược các ñỉnh của nó là các căn bậc n của ñơn vị tức là 2k

in

ia e

π

= với 0 1i n≤ ≤ − . Như vậy các ñỉnh của nó có thể viết dưới dạng i

ia ε= với

0 1i n≤ ≤ − và 1a ε= .

6. Cho ña giác ñều 7 cạnh 0 1 6...A A A . Chứng minh rằng 0 1 0 2 0 3

1 1 1

A A A A A A= +

A0

A1A2

A3

A4

A5

A6

A1'

A2'

Giải: Giả sử các ñỉnh của ña giác lồi trên là i

ia ε= với 0 6i≤ ≤ và

2

7i

ε = .

Page 124: Kỷ yếu hội thảo các trường chuyên dh db bắc bộ lần III

=========================================================== 126

HỘI CÁC TRƯỜNG THTP CHUYÊN KHU VỰC DUYÊN HẢI VÀ ðỒNG BẰNG BẮC BỘ

Hội thảo khoa học môn Toán học lần thứ III - 2010

Khi ñó dễ chứng minh ñược �1 0 3

2

7A A A

π= và do ñó ảnh của 1A qua phép quay tâm 0A

góc quay 2

7

π là ñiểm 1 'A thuộc ñoạn 0 3A A và

2' ' 2 2 271 1 11 ( 1) ( 1) 1 ( 1) 1

i

a e a a w a w wπ

− = − ⇒ = − + = − + trong ñó 7wi

= .

Tương tự �2 0 3 7A A A

π= và ảnh của của 2A qua phép quay tâm 0A góc quay

7

π là ñiểm

2 'A thuộc ñoạn 0 3A A và ' ' 472 2 21 ( 1) ( 1) 1

i

a e a a w wπ

− = − ⇒ = − + .

Do các ñiểm A0;A1’,A2’,A3 thẳng hàng nên ta chỉ cần chứng minh hệ thức:

( )2 2 4 6

1 1 1

( 1) 1 1w w w w w= +

− − − với 7w

i

= . Hệ thức này xin dành cho bạn ñọc.

7. Cho ña giác ñều 15 cạnh 0 1 14...A A A . Chứng minh hệ thức

0 1 0 2 0 4 0 7

1 1 1 1

A A A A A A A A= + + .

8. Cho ña giác ñều n cạnh nội tiếp ñường tròn bán kính r. Chứng minh rằng với mọi

ñiểm P nằm trên ñường tròn và với mọi số tự nhiên m<n thì 1

2 2

20

nm m m

k mk

PA C nr−

=

=∑ .

9. Cho tam giác ABC và hai ñiểm phân biệt M, N sao cho AM:BM:CM=AN:BN:CN. Chứng minh rằng MN ñi qua tâm ñường tròn ngoại tiếp tam giác ABC.

10. Cho P là một ñiểm tùy ý trên cung �0 1nA A

− của ñường tròn ngoại tiếp ña giác ñều

0 1 1...n

A A A−

. Gọi hi là khoảng cách từ P ñến các ñường thẳng Ai-1Ai với i=1,2,..,n. Chứng

minh rằng 1 1

1 1 1...

n nh h h

+ + = .

Với tam giác ñều, ta có thể ñưa ra một ñiều kiện cần và ñủ ñể một tam giác là tam giác ñều như sau:

A2

A0

A1

Xét tam giác ñều 0 1 2A A A với i

ia ε= ,i=0,1,2 và

2 2cos sin

3 3i

π πε = + và tam giác

ñều ABC bất kì với nó: Theo ñịnh lý 1.4 ta có hệ thức:

Page 125: Kỷ yếu hội thảo các trường chuyên dh db bắc bộ lần III

=========================================================== 127

HỘI CÁC TRƯỜNG THTP CHUYÊN KHU VỰC DUYÊN HẢI VÀ ðỒNG BẰNG BẮC BỘ

Hội thảo khoa học môn Toán học lần thứ III - 2010

1 0

2

2 0

1 1

1 1

a c a a

b c a a

ε

ε ε

− − −= = =

− − − +(1) nếu hai tam giác trên cùng hướng

1 0

2

2 0

1 1

1 1

a b a a

c b a a

ε

ε ε

− − −= = =

− − − + (2) nếu hai tam giác trên ngược hướng.

Hai hệ thức trên có thể viết lại là 2 2(1) 0 0a b c a b cε ε ε ε⇔ + + = ⇔ + + =

22 2(2) 0 0 0a b c a b c a b cε ε ε ε ε ε⇔ + + = ⇔ + + = ⇔ + + =

Do vậy ñiều kiện cần và ñủ ñể tam giác ABC ñều là 2 0a wb w c+ + = trong ñó w là

một căn bậc 3 khác 1 của ñơn vị. Chú ý rằng nếu 2

3wi

= thì tam giác trên có hướng

dương còn nếu 2

( )3w

i

= thì tam giác trên có hướng âm. Áp dụng ñiều kiện trên ta có thể giải các bài toán sau: 11. Cho tam giác ABC, về phía ngoài tam giác dựng các tam giác ñều MBC, NCA,

PAB. Gọi G, H, I là trọng tâm của các tam giác trên. Chứng minh rằng tam giác GHI ñều. Xét tam giác ABC có hướng dương thì các tam giác MCB, NAC, PBA trên cũng có

hướng dương. Theo các hệ thức trên ta có 2

2

2

0

0

0

m c b

n a c

p b a

ε ε

ε ε

ε ε

+ + =

+ + =

+ + =

ðể chứng minh tam giác GHI ñều ta chứng minh 2 0g h iε ε+ + = trong ñó

3

m b cg

+ += …. Việc chứng minh xin dành cho bạn ñọc.

12. Trong mặt phẳng cho tam giác A1A2A3 và ñiểm P0. Kí hiệu các ñiểm Aa=As-3 với

mọi số tự nhiên s>3. Xét dãy các ñiểm P0;P1;… cho bởi Pk+1 là ảnh của Pk qua phép quay tâm Ak+1 góc quay 1200. Chứng minh rằng nếu P1986=P0 thì tam giác A1A2A3 là tam giác ñều.

Trong các bài toán về ña giác nội tiếp ta có thể giả sử chúng nội tiếp trong ñường tròn

ñơn vị. Sau ñây là một số ví dụ 13. Cho H là trực tâm tam giác ABC và P là một ñiểm tuỳ ý trên ñường tròn ngoại tiếp.

E là chân ñường cao kẻ từ B và dựng các hình bình hành PAQB và PARC. X là giao ñiểm của AQ và HR. Chứng minh rằng EX//AP.

Page 126: Kỷ yếu hội thảo các trường chuyên dh db bắc bộ lần III

=========================================================== 128

HỘI CÁC TRƯỜNG THTP CHUYÊN KHU VỰC DUYÊN HẢI VÀ ðỒNG BẰNG BẮC BỘ

Hội thảo khoa học môn Toán học lần thứ III - 2010

A

B C

H

E

O

P

Q R

X

Giải: Xét tam giác ABC nội tiếp trong ñường tròn ñơn vị, khi ñó h=a+b+c và 1

( )2

ace a b c

b= + + − . Do APBQ là hình bình hành nên q=a+b-p, tương tự r=a+c-p

Do x, a, q thẳng hàng nên x a a q p b

pbx a a q p b

− − −= = = −

− − − (p, b thuộc ñường tròn ñơn

vị). Do ñó 2 axx a pb a

x apb abp

− − + −= + =

−(1). Tương tự các ñiểm h,r,x thẳng hàng nên ta

tính ñược x h

pbx h

−=

− nên

bp bpx a b c p

a cxbp

− − − + + += (2).

Từ (1) và (2) ta tính ñược 1

(2 )2

bpx a b c p

c= + + − − .

ðể chứng minh XE//AP ta chứng minh e x a p

ape x a b

− −= = −

− −.

Ta có 2 21 ( )( )

( )2 2 2

bp ac bcp b p abc ac b c bp ace x p a

c b bc bc

+ − − + −− = + − − = =

1 1 1 1( )( )

1.

1 1 22 .

bp acb c bp ace x

ap bc

b c

+ −−

− = = − nên ta có ñiều phải chứng minh.

14. Cho tứ giác ABCD nội tiếp, P và Q là các ñiểm ñối xứng với C qua AB và AD. Chứng minh rằng PQ ñi qua trực tâm tam giác ABD.

Page 127: Kỷ yếu hội thảo các trường chuyên dh db bắc bộ lần III

=========================================================== 129

HỘI CÁC TRƯỜNG THTP CHUYÊN KHU VỰC DUYÊN HẢI VÀ ðỒNG BẰNG BẮC BỘ

Hội thảo khoa học môn Toán học lần thứ III - 2010

A

D C

B

Q

P

Giả sử tứ giác ABCD nội tiếp trong ñường tròn ñơn vị. Khi ñó ab

p a bc

= + −

,ad

q a dc

= + + và h=a+b+d.

1 1 1 1 1

aba b a b d

p h abdcc cp h

a b ab a b d

+ − − − −−

= =− + − − − −

. Tương tự q h abd

cq h

−=

−. Do ñó P, Q, H

thẳng hàng. 15. Tam giác ABC trực tâm H nội tiếp ñường tròn (O) bán kính R. D là ñiểm ñối xứng

với A qua BC, E là ñiểm ñối xứng với B qua CA, F ñối xứng với C qua AB. Chứng minh rằng các ñiểm D, E, F thẳng hàng khi và chỉ khi OH=2R.

16. Cho lục giác ABCDEF nội tiếp. Chứng minh rằng các giao ñiểm của AB và DE, BC và EF, CD và FA thẳng hàng.

17. Cho tứ giác ABCD nội tiếp, AB cắt CD tại E, AD cắt BC tại F, AC cắt BD tại G. Chứng minh rằng O là trực tâm tam giác EFG.

18. Cho tứ giác ABCD nội tiếp và K, L, M, L là trung ñiểm của AB, BC, CD, DA. Chứng minh rằng các trực tâm tam giác AKN, BKL, CLM, DMN tạo thành các ñỉnh của một hình bình hành.

Sử dụng ñịnh lý 1.7 ta có thể giải ñược một số bài toán liên quan ñến ñường tròn nội tiếp ña giác

19. ðường tròn (I) nội tiếp tam giác ABC và tiếp xúc với các cạnh của tam giác tại P, Q, R. Gọi H là giao ñiểm của PR và AC. Chứng minh rằng IH vuông góc với BQ.

20. Cho ñường tròn (O) nội tiếp tứ giác ABCD và tiếp xúc với các cạnh AB, BC, CD, DA tại K, L, M, N. KL cắt MN tại S. Chứng minh rằng OS vuông góc với BD.

Trên ñây là một số ứng dụng ñơn giản của số phức ñối với những bài toán hình học phẳng. Hy vọng sau bài viết này, cùng với phương pháp toạ ñộ trong mặt phẳng chúng ta có thêm một cách nhìn nữa về cách giải cho các bài toán hình học thông thường.

Tài liệu tham khảo

- Complex number in Geometry Marko Radovanovic -Tạp chí Mathematical Excalibur Vol. 1,No.3,May-Jun,95 - Một số tài liệu trên mạng.

Page 128: Kỷ yếu hội thảo các trường chuyên dh db bắc bộ lần III

=========================================================== 130

HỘI CÁC TRƯỜNG THTP CHUYÊN KHU VỰC DUYÊN HẢI VÀ ðỒNG BẰNG BẮC BỘ

Hội thảo khoa học môn Toán học lần thứ III - 2010

BẤT BIẾN TRONG CÁC BÀI TOÁN LÝ THUYẾT TRÒ CHƠI Phạm Minh Phương

Giáo viên trường THPT chuyên ðại học Sư phạm Hà Nội Bất biến là khái niệm quan trọng của toán học. Nói một cách ñơn giản thì bất biến là ñại

lượng hay tính chất không thay ñổi trong khi các trạng thái biến ñổi. Người ta sử dụng bất biến ñể phân loại các vật trong một phạm trù nào ñó. Hai vật thuộc cùng một loại nếu nó có cùng tính chất H và nếu vật A có tính chất H, vật B không có tính chất H thì B không cùng loại với A.

Trong chuyên ñề này chúng tôi xin giới thiệu về ứng dụng của bất biến trong các bài toán về thuật toán của lý thuyết trò chơi. ðây là dạng toán thường gặp trong các kì thi Olympic.

1. Một số khái niệm của lý thuyết trò chơi 1. Thuật toán

Cho tập A ≠ ∅ và ta gọi là không gian các trạng thái, mỗi phần tử của A là một trạng thái. Khi ñó, mỗi ánh xạ: :T A A→ gọi là một thuật toán (ôtômat).

2. Các bài toán về thuật toán

Bài toán 1 (Bài toán tìm kiếm thuật toán). Cho trạng thái ban ñầu 0α và trạng

thái kết thúc nα . Hỏi có hay không thuật toán T trên A sao cho khi thực hiện T hữu

hạn lần ta thu ñược nα ?

0 1 2 ...T T T T

nα α α α→ → → →

Bài toán 2. Cho thuật toán T trên A và trạng thái ban ñầu α . a) Xét trạng thái .Aβ ∈ Hỏi có thể nhận ñược β từ α sau hữu hạn lần thực hiện

thuật toán T hay không?

b) Tìm tập hợp α gồm tất cả các trạng thái có thể nhận ñược từ α sau hữu hạn bước thực hiện thuật toán T:

( ){ }: nA Tα β β α= ∈ =

3. Hàm bất biến

Cho thuật toán T trên A và I là một tập hợp khác rỗng mà ta gọi là không gian các mẫu bất biến.

Khi ñó, ánh xạ :H A I→ gọi là hàm bất biến trên A nếu

, : ( ) ( ).a b A b a H b H a∀ ∈ ∈ ⇒ =

2. Một số bài toán minh hoạ Bài toán 1. Hai người chơi cờ. Sau mỗi ván người thắng ñược 2 ñiểm, người thua ñược

0 ñiểm, nếu hoà thì mỗi người ñược 1 ñiểm. Hỏi sau một số ván liệu có thể xảy ra trường hợp một người ñược 7 ñiểm và người kia ñược 10 ñiểm ñược không?

Lời giải. Gọi ( )S n là tổng số ñiểm của cả hai người sau ván thứ n. Ta có ( )S n bất biến

theo modun 2. Do ñó

( ) ( ) ( )0 0 mod 2 , 0.S n S n≡ ≡ ∀ ≥

Vậy không thể xảy ra trường hợp một người ñược 7 ñiểm và người kia ñược 10 ñiểm.

Page 129: Kỷ yếu hội thảo các trường chuyên dh db bắc bộ lần III

=========================================================== 131

HỘI CÁC TRƯỜNG THTP CHUYÊN KHU VỰC DUYÊN HẢI VÀ ðỒNG BẰNG BẮC BỘ

Hội thảo khoa học môn Toán học lần thứ III - 2010

Bài toán 2. Thực hiện trò chơi sau: Lần ñầu viết lên bảng cặp số ( )2; 2 . Từ lần thứ

hai, nếu trên bảng có cặp số ( );B a b= thì ñược phép viết thêm cặp số

( ) ; .2 2

a b a bT B

+ − =

Hỏi có thể viết ñược cặp số ( )1;1 2+ hay không?

Lời giải. Giả sử ở bước thứ n ta viết cặp số ( ); .n na b Khi ñó tổng ( ) 2 2n nS n a b= + là ñại

lượng bất biến. Do ñó

( ) ( )2

2 2 20 0 6 1 1 2 , 0.S n a b n= + = ≠ + + ∀ ≥

Vậy không thể viết ñược cặp số ( )1;1 2+ .

Bài toán 3. Trên bảng có hai số 1 và 2. Thực hiện trò chơi sau: Nếu trên bảng có hai số a và b thì ñược phép viết thêm số .c a b ab= + + Hỏi bằng cách ñó có thể viết ñược các số 2001 và 11111 hay không?

Lời giải. Dãy các số viết thêm là: 5; 11; 17; ... Dễ dàng chứng minh ñược dãy các số ñược viết thêm ñều chia cho 3 dư 2. Bất biến trên

cho phép ta loại trừ số 2001 trong dãy các số ñược viết thêm. Tuy nhiên, bất biến ñó không cho phép ta loại trừ số 11111. Ta ñi tìm một bất biến khác. Quan sát các số viết ñược và quy tắc viết thêm số, ta có

( )( )1 1 1c a b ab c a b= + + ⇒ + = + +

và nếu cộng thêm 1 vào các số thuộc dãy trên ta có dãy mới: 6; 12; 18; ...

Như vậy, nếu cộng thêm 1 vào các số viết thêm thì các số này ñều có dạng: 2 .3 .m n Do số 11111 1 11112 3.8.463+ = = nên 11111 không thuộc dãy các số ñược viết thêm.

Bài toán 4 (VMO – 2006). Xét bảng ô vuông ( ), 3 .m n m n× ≥ Thực hiện trò chơi sau:

mỗi lần ñặt 4 viên bi vào 4 ô của bảng, mỗi ô một viên bi, sao cho 4 ô ñó tạo thành một trong các hình dưới ñây:

Hỏi sau một số lần ta có thể nhận ñược bảng mà số bi trong các ô bằng nhau ñược không

nếu: a) 2004, 2006?m n= =

b) 2005, 2006?m n= =

Lời giải a) Bảng ñã cho có thể chia thành các hình chữ nhật 4 2× nên có thể nhận ñược trạng

thái mà số bi trong các ô bằng nhau.

b) Tô màu các ô như hình vẽ

Page 130: Kỷ yếu hội thảo các trường chuyên dh db bắc bộ lần III

=========================================================== 132

HỘI CÁC TRƯỜNG THTP CHUYÊN KHU VỰC DUYÊN HẢI VÀ ðỒNG BẰNG BẮC BỘ

Hội thảo khoa học môn Toán học lần thứ III - 2010

Dễ thấy, mỗi lần ñặt bi có 2 viên ñược ñặt vào các ô màu ñen và 2 viên ñược ñặt vào ô màu trắng. Do ñó, nếu gọi ( )S n là số bi trong các ô màu ñen và ( )T n là số bi trong

các ô màu trắng sau lần ñặt bi thứ n thì ( ) ( )S n T n− là ñại lượng bất biến. Ta có

( ) ( ) ( ) ( )0 0 0, 0.S n T n S T n− = − = ∀ ≥

Vì 2005m = là số lẻ nên nếu nhận ñược trạng thái mà số bi trong các ô bằng nhau thì

( ) ( ) 2005,S n T n m− = =

vô lý. Bài toán 5 (IMO – 2004). Ta ñịnh nghĩa viên gạch hình móc câu là hình gồm 6 ô vuông

ñơn vị như hình vẽ dưới ñây, hoặc hình nhận ñược do lật hình ñó (sang trái, sang phải, lên trên, xuống dưới) hoặc hình nhận ñược do xoay hình ñó ñi một góc:

Hãy xác ñịnh tất cả các hình chữ nhật m n× , trong ñó m, n là các số nguyên dương sao cho có thể lát hình chữ nhật ñó bằng các viên gạch hình móc câu?

Lời giải. Dễ thấy { }, 1;2;5 .m n ∉ Chi hình chữ nhật ñã cho thành các m n× ô vuông và

ñánh số các hàng, các cột từ dưới lên trên, từ trái sang phải. Ta gọi ô ( );p q là ô nằm ở giao

của hàng thứ p và cột thứ q. Hai viên gạch hình móc câu chỉ có thể ghép lại ñể ñược một trong hai hình dưới ñây:

Do ñó, ñể lát ñược hình chữ nhật m n× thì .m n phải chia hết cho 12. Nếu ít nhất một

trong hai số m, n chia hết cho 4 thì có thể lát ñược. Thật vậy, giả sử ñược m chia hết cho 4. Ta có thể viết n dưới dạng: 3 4n a b= + , do ñó có thể lát ñược.

Xét trường hợp m, n ñều không chia hết cho 4. Ta chứng minh trường hợp này không thể lát ñược. Giả sử ngược lại, khi ñó m, n ñều chia hết cho 2 nhưng không chia hết cho 4. Ta tạo bất biến như sau: Xét ô ( );p q . Nếu chỉ một trong hai toạ ñộ p, q chia hết cho 4 thì

ñiền số 1 vào ô ñó. Nếu cả hai toạ ñộ p, q chia hết cho 4 thì ñiền số 2. Các ô còn lại ñiền số 0. Với cách ñiền số như vậy ta thu ñược bất biến là tổng các số trong hình (H1) và tổng các số trong hình (H2) ñều là số lẻ. Do m, n chắn nên tổng các số trong toàn bộ hình chữ nhật m n× là số chẵn. ðể lát ñược thì tổng số hình (H1) và (H2) ñược sử dụng phải là số chẵn. Khi ñó, .m n chia hết cho 24, vô lý.

3. Bài tập Bài tập 1. Một con robot nhảy trong mặt phẳng toạ ñộ theo quy tắc sau: Xuất phát từ

ñiểm ( );x y , con robot nhảy ñến ñiểm ( )'; 'x y xác ñịnh như sau:

2' , ' .

2

x y xyx y

x y

+= =

+

Chứng minh rằng, nếu ban ñầu con robot ñứng ở ñiểm ( )2009; 2010 thì không bao giờ

con robot nhảy vào ñược trong ñường tròn (C) có tâm là gốc toạ ñộ O và bán kính 2840.R =

(H1)

(H2)

Page 131: Kỷ yếu hội thảo các trường chuyên dh db bắc bộ lần III

=========================================================== 133

HỘI CÁC TRƯỜNG THTP CHUYÊN KHU VỰC DUYÊN HẢI VÀ ðỒNG BẰNG BẮC BỘ

Hội thảo khoa học môn Toán học lần thứ III - 2010

Bài tập 2. Ở 6 ñỉnh của một lục giác lồi có ghi 6 số chẵn liên tiếp theo chiều kim ñồng hồ. Thực hiện thuật toán sau: mỗi lần chọn một cạnh và cộng thêm mỗi số trên cạnh ñó với cùng một số nguyên nào ñó. Hỏi có nhận ñược hay không trạng thái mà 6 số ở 6 ñỉnh bằng nhau?

Bài tập 3. Một dãy có 19 phòng. Ban ñầu mỗi phòng có một người. Sau ñó, cứ mỗi ngày có hai người nào ñó chuyển sang hai phòng bên cạnh nhưng theo hai chiều ngược nhau. Hỏi sau một số ngày có hay không trường hợp mà:

a) Không có ai ở phòng có thứ tự chẵn?

b) Có 10 người ở phòng cuối dãy?

Bài tập 4 (ðề thi chọn ñội tuyển Bắc Ninh năm 2007) Trên bàn có 2007 viên bi bồm 667 bi xanh, 669 bi ñỏ, 671 bi vàng. Thực hiện thuật toán

sau: Mỗi lần lấy ñi hai viên bi khác màu và ñặt thêm hai viên bi có màu còn lại. Hỏi có thể nhận ñược trạng thái mà trên bàn chỉ còn lại các viên bi cùng màu ñược không?

Bài tập 5 (VMO – 1991). Cho bảng 1991 1992.× Kí hiệu ô ( );m n là ô nằm ở giao của

hàng thứ m và cột thứ n. Tô màu các ô của bảng theo quy tắc sau: Lần thứ nhất: Tô ba ô: ( ) ( ) ( ); , 1; 1 , 2; 2 .r s r s r s+ + + +

Từ lần thứ hai: mỗi lần tô ñúng ba ô chưa có màu nằm cạnh nhau trên cùng một hàng hoặc trên cùng một cột.

Hỏi có thể tô hết tất cả các ô của bảng ñược không? Bài tập 6 (VMO – 1992). Tại mỗi ñỉnh của ña giác lồi 1 2 1993...A A A ta ghi một dấu cộng

(+) hoặc một dấu trừ (-) sao cho trong 1993 dấu ñó có cả dấu (+) và dấu (-). Thực hiện việc thay dấu như sau: mỗi lần thay dấu ñồng thời tại tất cả các ñỉnh của ña giác ñã cho theo quy tắc:

- Nếu dấu tại iA và 1iA + là như nhau thì dấu tại iA ñược thay bằng dấu (+).

- Nếu dấu tại iA và 1iA + khác nhau thì dấu tại iA ñược thay bằng dấu (-).

(Quy ước: 1994A là 1.A )

Chứng minh rằng, tồn tại số 2k ≥ sao cho sau khi thực hiện liên tiếp k lần thay dấu ta ñược ña giác 1 2 1993...A A A mà dấu tại mỗi ñỉnh trùng với dấu tại chính ñỉnh ñó ngay sau lần

thay dấu thứ nhất. Bài tập 7 (Shortlist). Cho k, n là các số nguyên dương. Xét một bảng ô vuông vô hạn,

ñặt 3k n× quân cờ trong hình chữ nhật 3k n× . Thực hiện trò chơi sau: mỗi quân cờ sẽ nhảy ngang hoặc dọc qua một ô kề với nó và có chứa quân cờ, ñể ñến ô trống kề với ô nó vừa nhảy qua. Sau khi làm như trên ta loại bỏ quân cờ ở ô bị nhảy qua ra khỏi bàn cờ. Chứng minh rằng, với cách chơi ñó trên bảng ô vuông sẽ không bao giờ còn lại ñúng một quân cờ.

Bài tập 8 (Belarus 1999). Cho bảng 7 7× và các quân cờ có một trong ba loại sau: 3 1× , 1 1× và hình chữ L gồm 3 ô. Người thứ nhất có vô hạn quân 3 1× và một quân hình chữ L, trong khi người thứ hai chỉ có duy nhất một quân 1 1× . Chứng minh rằng

a) Nếu cho người thứ hai ñi trước, anh ta có thể ñặt quân cờ của mình vào một ô nào ñó sao cho người thứ nhất không thể phủ kín phần còn lại của bảng.

b) Nếu cho người thứ nhất thêm một quân hình chữ L thì bất kể người thứ hai ñặt quân cờ của mình ở ñâu thì người thứ hai cũng phủ kính ñược phần còn lại của bàn cờ.

Page 132: Kỷ yếu hội thảo các trường chuyên dh db bắc bộ lần III

=========================================================== 134

HỘI CÁC TRƯỜNG THTP CHUYÊN KHU VỰC DUYÊN HẢI VÀ ðỒNG BẰNG BẮC BỘ

Hội thảo khoa học môn Toán học lần thứ III - 2010

Bài tập 9. Xét bảng 9 9× . Ở ô ( );p q ta viết số: ( )9 1 .p q− + Thực hiện thuật toán sau:

mỗi lần lấy ra một hình vuông 4 4× và tăng ñồng thời các số trong các ô của hình vuông này lên một ñơn vị. Chứng minh rằng tại mọi thời ñiểm, ước số chung lớn nhất của tất cả các số trong bảng luôn bằng 1.

Bài tập 10. Chia góc vuông Oxy thành lưới ô vuông ñơn vị. Các hàng và các cột ñược ñánh thứ tự từ dưới lên, từ trái sang phải. Ban ñầu, ñặt vào ô ( )1;1 một viên bi. Thực hiện

thuật toán sau: mỗi lần lấy ra khỏi góc viên bi nằm ở ô ( );p q nào ñó mà tại các ô ( )1;p q+

và ( ); 1p q + không có bi, ñồng thời thêm vào hai ô nói trên mỗi ô một viên bi. Hỏi có nhận

ñược hay không trạng thái mà a) Các ô ( ) ( ) ( ) ( )1;1 , 1;2 , 2;1 , 2;2 ñều không có bi?

b) Các ô ( ) ( ) ( ) ( ) ( ) ( )1;1 , 1;2 , 2;1 , 2;2 , 1;3 , 3;2 ñều không có bi?